You are on page 1of 416

. What gauge needle would be appropriate for a subcutaneous injection?

25 gauge needle is preferred for subcutaneous injections.

How should an artificial eye be cleaned?

with soap and water.

. What is the normal fetal heart rate?

120 and 160 beats per minute.

. On average, how much weight does a mother gain during pregnancy?

25 and 30 pounds during pregnancy.

What is the best antidote for magnesium sulfate toxicity

magnesium sulfate toxicity is calcium glutonate.

. When should a pregnant patient be tested for alpha protein serum levels

18 to 20 weeks into gestation.

. Which of the following is NOT a factor that causes the urine pH to be less than 4.5?

A diet high in protein, a fever, and metabolic acidosis can all cause the urine pH to drop below 4.5.

t what age should solids be introduced into an infant's diet?

4 and 6 months

What condition is characterized by involuntary twitching movements of the muscles, particularly rolling of the tongue?

The classic symptom of tardive dyskinesia is involuntary spasms of the facial muscles, especially rolling of the tongue.

Which foods should be avoided while on MAOI antidepressants?

Foods containing tyramine should be avoided while on MAOI antidepressants.

What kind of antacids are known to cause diarrhea?

Magnesium-based antacids are notorious for causing diarrhea

What is the best initial treatment for hypercalcemia?

Saline and furomeside should be administered as soon as possible during cases of hypercalcemia.

What is the most common transfusion reaction?

fever

What is another name for primal adrenal insufficiency?

Addison's disease is also known as primal adrenal insufficiency.

Pulse pressure (pp) is considered the

Difference between the systolic and diastolic pressure.

maternal estrogen has been transfered to the neonate what would the nurse see

enlarged breast tissue

molar pg

increased HCG levels

meconium in the amnotic fluid is nl for

breach position

restraints

should be removed q 2hrs and left off for 5 min for ROM and skin checks

cool mist tent

decrease resp tract edema

a kid with celiac how can you monitor the effectivness of tx

monitor stools

moro reflex

5 months

vacular lesions that ooze and form crust on face and extrimites impetigo direct contact wash hands!

post op cleft lip

clean suture line after every feeding w/sterile solution

cf diet

high cal high protein

pancrilipase

with meals and snacks

iron food

PB, green veggies and rasins

fontanels

should be closed by 18 months

PDA administer

indomethocin

car seat

4 or 40lbs

mo solids till

6 months

to prevent OM

place baby in upright position while feeding

preschool child

give dolls and medical stuff to play with

fetal heart tones

at 11 weeks with doppler

chylamydia

tx w/ zithromax

ectopic pg sx

abd pain vag pain and positve hcg

false labor contractions

felt in abd, irregular and relived by walking

epidural

be sure to hydrate before r/t the anestetic can cause hypotension

apgar

hr, resp, reflex, irratiblity

going into 2nd stage of labor

uncontrolable urge push

common adverse effect of phototherapy

watery stools

rooting reflex

kid turns head to the side

cns depressant lathargy is s s/e. mag sulfate Tox = flushing reflex depression , decreased urinary output, depresed rep.

folic acid

egg yolks

variable decels

change position

external monitor

lie any comfy position but try to stay off back

mild preeclampsia

htn, edema, proteinurea

placenta previa

soft relaxed non tender uterus

rh neg mom rh pos baby

give mom rhogam with in 72 hrs

to prepare for amnio

asl pt to void, assess fetal heart rate, monitor maternal vs

premature labor

provide adequate hydration

hypogycemia in the neonate

lathargy is the inital sign

neonate wt loss

loose 10 % of birth wt in 3-4 days then nl

breast feed

q 1-3 hrs

placenta previa

painless vag bleed increase r/f after 35

ace inhibitors

inhibit k secretion

DI

give vasopressin sub q or intranassaly

mi

hypotension rapid pluse and cp

post op cataract

don't sleep on effected side

predinsone side effects

wt gain hypertension and insomnia

mefanide sulfate

topical abx for burns

neostigmine

for mg to give 45 60 min before meals to improve chewing

type 1 hypogly event

admin 15-20 fast acting CHO

acute viral hepatitis

bedrest

bladder spasms and urge to void with foley after turp

check for clot by ns irrigation at 40-60 gtts q min amount instilled should be the amount with drawn if patent

nsaids

w/food to prevent gi upset

autonomic dysreflexia

monitor foley patancy can precipitate condition

asending or transverse colonoscopy

liquid or simi liqued stools

myasthenia gravis

asses resp system and muscle weakness

long term predinsone

osteoporosis

ca

premeno 1000 post meno 1500 mg q d

rule of 9's

trunk post and ant and legs make up 18% . head neck and arms 9%. perineum 1%

common allergies

berries, nuts and eggs

stoma irrigation

elevate bsg 18-24 inches above stoma, 500-1000ml lukewarm h20 if cramping stop irrigation hold breath then resume

hip replacement sitting

limit flexion to 90 degrees

tens machine

blocks painful stimuli from traveling to small nerve endings

p thyroidectomy

tetany may occur monitor for s/s of muscle twitching tingling on numbness

gcs

3 indicates coma

chest tube is disconnected

place end in sterile water to prevent air from entering chest tube

crutches

all weight on the hands

bed bath water temp

110-115

bacterium responsible for tetnus

colstridium tetani

tpn

monitor wt q d

positive ppd

induration and swelling 5-15mm

iron def anemia sx

dyspnea tacycardia pallor and fatigue

hemmacult rules

no red meat turnups or pultry or fish for 4 days

laminectomy

logroll pt

antacids w. aluminum

monitor for constipation

A nursing assessment on a male patient indicates that he is showing evidence of increased intracranial pressure. The first nursing action is to Answer1.1= Place him in high-Fowler`s position. Attempt to have him deep breathe. As the PaCO2 increases in the cerebral tissues, blood rushes to the area and this further increases the intracranial pressure. Decreasing the PaCO2, accomplished by breathing deeper and more slowly, will decrease the intracranial blood flow, thus decreasing intracranial pressure.

= Place him on his side.

= Have him cough and deep breathe.

Question.2= A 9-year-old patient with cystic fibrosis will take pancreatic enzymes 3 times a day. The nurse will know the child`s mother needs more education on the purpose and timing of these enzymes if she says = "The purpose of the enzymes is to help digest the fat in foods." = "The enzymes should be taken prior to meals." = "They should be given following breakfast, lunch and dinner." = "They should be taken at meal times, 3 times a day." = The purpose of the pancreatic enzymes is to replace the enzymes unavailable in the child`s system that assist with the digestion of fats. Therefore, they should be taken prior to, not following, the ingestion of food.

= Formulating a care plan for a patient with chronic lymphocytic leukemia, the nurse will expect that his laboratory results will reveal increased = White blood cells (lymphocytes) increase in number, although they are immature. With this condition, there is a decreased number of red blood cells and platelets.

Lymphocytes. = Monocytes. = Platelets. = Red blood cells.

Question.4= As part of the nursing care plan for a patient with chronic lymphocytic leukemia, the nurse will formulate nursing diagnoses. A priority Nursing Diagnosis is Immature white blood cells predispose the patient to infections, so this nursing diagnosis is a priority. Fluid volume deficit may also be an important nursing diagnosis, because the patient may be prone to bleeding. It does not, however, have as high a priority as (1)

= Infection, potential for. = Alteration in tissue perfusion. = Impairment of skin integrity. = Fluid volume deficit.

The nurse is assigned a patient diagnosed with a left-sided hemopneumothorax who has a chest tube connected to a water-seal drainage system. During the shift change assessment, the nurse observes that fluid in the suction control chamber is bubbling. The intervention is to = Bubbling in the suction control chamber is expected. The tubing should always be free of kinks without dependent loops. The patient should be turned side-to-side to promote drainage. While chamber levels must be maintained, bubbling in the suction control chamber does not indicate a need for adding water.

Add water to the water-seal chamber. = Make sure that there are dependent loops in the system. = Do nothing, as this is normal. Sit the patient up to increase suction.

= The nurse is assigned to dry and wrap a newborn baby. The nurse will don gloves to complete this task because It is important to observe universal precautions before the infant`s first bath. = The infant requires protection from infection. = It is the nurse`s choice whether or not to wear gloves. = Gloves are standard procedure for care of normal newborns.

#2 With the high increase in numbers of HIV positive newborns, the Centers for Disease Control (CDC) guidelines include wearing gloves when handling newborns before their first bath. They will be covered with amniotic fluid and other secretions from the mother, so it is important to be protected from these body fluids which could harbor the HIV virus.

On the first postpartum day, a mother asks the nurse to bathe and change the baby. The best response is #3 Do you remember how you bathed your last baby?" = "It`s better if you care for the baby." "How does it feel to have a new baby to care for?" = It is important to find out how the mother feels about baby care. She may just be tired, she may be afraid, or she may have some other reason for her request. The nurse needs to assess further, so this question is the most relevant.

= "I`d be glad to."

= As the nurse is checking on a male patient with dementia, she observes that he has been incontinent and soiled his clothes. This is the second time this has happened on this shift. The most appropriate nursing intervention is to #4 = Even though the nurse may eventually have to place diapers on the patient, this is not the first intervention. An every 2-hour bathroom schedule may solve the problem because the patient cannot remember to tell the nurse when he needs to urinate.

Put the patient in adult diapers to protect him from embarrassment. = Scold the patient and tell him not to wet his pants again. = Tell the patient to ask the nurse for assistance the next time he has to go to the bathroom.

= Tell the patient the nurse will change his pants and establish a 2-hour schedule of taking him to the bathroom.

A 50-year-old patient with asthma is to receive Aminophylline. The nurse will evaluate the effectiveness of Aminophylline by observing for decreased Respiratory distress. = Rales. = Mucus production. =Pulse rate. #1 = Aminophylline causes bronchodilatation and, therefore, increased oxygenation. The patient is more able to cough up secretions and the breath sounds will become more clear. The CNS is stimulated by this drug and the pulse rate may increase.

Question.10= A very attractive young man with whom the nurse has had a nurse-patient relationship begins to make sexual advances toward her, making her very uncomfortable. The best approach is to

Ignore the advances, for the nurse knows that lack of reinforcement usually extinguishes the behavior. = Tell his doctor, who should be informed of his inappropriate behavior on the unit. = Be direct in communicating the discomfort with his advances and set limits on his behavior. = Continue the relationship, for the nurse knows ending it will reinforce his negative self-image.

=The patient needs direct feedback and clear delineation of limits to the relationship for example, "This is not acceptable behavior. If you continue, I will not talk with you."

= The nurse caring for a patient with an ileostomy will carefully assess the skin around the stoma because It is very difficult to ensure proper fit of the appliance with skin breakdown. = Digestive enzymes may cause skin breakdown. = The effluent is more solid than watery and may stick to the skin. = It takes longer to heal than a colostomy stoma. #2 Rational.11= The discharge from an ileostomy is watery and contains many digestive enzymes that have not been absorbed by the intestinal villi. These enzymes may cause skin breakdown. None of the other answers is accurate.

Question.12= Which one of the following treatment plans is most appropriate in the nursing/medical management of a 10-year-old child with congestive heart failure?

#3 Rational.12= Digoxin and diuretics are administered in order to increase the force of systolic contraction and to decrease venous volume/congestion. High-Fowler`s position is utilized to provide for maximum lung expansion. Daily weights allow for constant monitoring of fluid balance/status. Antibiotics are not administered prophylactically.

High concentrations of oxygen, strict bedrest, diuretics. Answer2.12= Oral fluids, daily weights, high-Fowler`s position. Answer3.12= High-Fowler`s position, digoxin, diuretics, daily weights. Answer4.12= Digoxin, diuretics, prophylactic antibiotics.

Question.13= The nurse will know that a patient undergoing radiation therapy understands the side effects of the therapy when he says that he schedules his meals #2 Immediately before therapy. Rational.13= Food and fluids should be taken 2 to 3 hours before or after the treatments to decrease the nausea experienced as a result of the radiotherapy.

Answer2.13=Two to 3 hours before therapy. Answer3.13= One hour before therapy. Answer4.13=Immediately after therapy.

Question.14= The most appropriate activity or toy for a 4 year old hospitalized for severe rheumatoid arthritis is

A VCR with a Disney movie or a radio. Answer2.14=An erector set or Legos. Answer3.14= A dartboard and darts or marbles. Answer4.14=A wooden puzzle or crayons and coloring book

#4 Rational.14= A puzzle or crayons would be appropriate for a 4 year old who is limited in activities by his diagnosis. An erector set is too advanced for his developmental level. A video movie or radio is too passive and will not encourage intellectual or creative development. Darts are dangerous for a 4 year old the dart may accidentally hit someone or something other than the dartboard.

Question.15= A priority nursing intervention for a patient on total parenteral nutrition (TPN) is to monitor the

#3 Rational.15= Because parenteral nutrition solutions are very high in dextrose, the pancreatic insulin-secreting ability may not be sufficient to maintain normal blood glucose levels. The patient may require insulin during all or part of the TPN course. There should be no change in the sodium levels, as these solutions are not high in sodium the patient should be in acid-base balance during the therapy.

Urine specific gravity. Answer2.15=Sodium level. Answer3.15=Blood sugar levels. Answer4.15=Urine pH.

Question.16= A 42-year-old female patient was admitted to the psychiatric unit with a diagnosis of acute depressive episode. She refuses to get out of bed, go the dining room, or participate in any of the unit`s programs. The most appropriate nursing action is to #3 Rational.16= Be positive, definite and specific about expectations. Do not give depressed patients a choice or try to convince them to get out of bed. Physically assist the patient to get up and dressed. This activity will help to mobilize her which, in turn, will help the depression to lift.

Allow her to remain in bed until she feels ready to join the other patients. Answer2.16= Suggest she may be hungry later so she had better get out of bed. Answer3.16= Tell her that the nurse will assist her out of bed and help her to dress. Answer4.16= Tell her the rules of the unit are that no patient can remain in bed.

Question.17= A patient is scheduled for laser surgery to correct his retinal detachment. Before the surgery, the nurse will position him On total bedrest with the area of detachment dependent. Answer2.17= In bed, in a prone position with his head supported by pillows. Answer3.17= On his right side with the area of detachment in upward position. Answer4.17= Up ad lib, but not turning his head. Rational.17=The retinal detachment area must be kept dependent (lowest part of the eye) in order to keep the separation between the retina and its circulatory layer, the choroid, minimal. The patient should be on bedrest.

Question.18= Postoperatively on the same day as surgery for a right hip replacement with a prosthesis, the patient may be positioned on

The patient`s back and left side. Answer2.18=The patient`s back and right side. Answer3.18= Either side. Answer4.18=The patient`s back only

#4 Rational.18= A total hip replacement requires very careful attention to position. The joint may dislocate in the early postoperative period and the nurse must maintain the affected limb in a position of abduction and external rotation. Turning to either side in the early postoperative stage would, therefore, be disallowed.

Question.19= A patient is several days postoperative following a right hip replacement with a prosthesis. Assisting the patient to sit in a chair, the nurse will use

he lowest possible chair with the back reclined. Answer2.19=A reclining chair with foot rest elevated. Answer3.19= A soft chair with arms. Answer4.19=An upright elevated chair.

#2 Rational.19= The ball in socket position is maintained by minimizing hip flexion (60 degrees or less). This is done with wheelchair and commode extenders, high chairs, and proper bed positions.

Question.20= To facilitate breathing in a child with bronchiolitis, the nursing care plan will include establishing an environment of #3 Rational.20= Cool, moist oxygen is the supportive therapy of choice for bronchiolitis. If the child is at home, warm mist from the shower may be administered for croup. Humidified oxygen is administered for pneumonia.

Humidified oxygen. Answer2.20=Warm mist with oxygen. Answer3.20= Cool, moist oxygen. Answer4.20=Oxygen therapy with no mist.

Question.21= A patient has sustained a femoral shaft fracture and is being treated with skeletal traction using balanced suspension with a Thomas splint and Pearson attachment. The goal of maintaining optimum positioning will be accomplished by

Keeping the affected knee in a position of extension. Answer2.21= Maintaining the lower leg at a 90 degree angle to the upper leg. Answer3.21= Maintaining the angle between the affected thigh and the bed. Answer4.21= Not elevating the head of the bed above ten degrees.

#3 Rational.21= It is important that the established angle between the affected thigh and the bed be maintained. The patient can usually have the head of the bed flat or elevated and the lower leg can be exercised, then rest in the Pearson attachment. If the patient migrates toward the head or foot of the bed, then the angle between the thigh and bed would be altered, so it is important that adequate countertraction be maintained.

Question.22= While inserting a Foley catheter in a female patient, the nurse will advance the catheter through the urethra a distance of 1 to 1 1/2 inches. Answer2.22=2 to 3 inches. Answer3.22=Until urine flows. Answer4.22=1 1/2 to 2 inches. #2 Rational.22=The Foley catheter should be advanced 2 to 3 inches into the female urethra.

Question.23= The nurse is caring for a patient undergoing chemotherapy for cancer. One of the goals of care is to discuss possible side effects of this therapy, one of which is Constipation. Answer2.23=Alopecia. Answer3.23= Increased appetite. Answer4.23=Weight gain.

#3 Rational.23= Alopecia, or hair loss, will probably occur caused by damage to the rapidly dwindling cells of hair follicles. Hair loss begins 2 to 3 weeks after therapy begins and continues through the course of therapy. The other side effects listed do not occur, as nausea, anorexia and diarrhea are common.

Question.24= The nurse will know a diabetic patient understands exercise and its relation to glucose when the patient tells the nurse that he eats bread and milk before, or juice or fruit during exercise activity because

Exercise stimulates pancreatic insulin production. Answer2.24= A diabetic`s muscles require more glucose during exercise. Answer3.24= Exercise enhances the passage of glucose into muscle cells. Answer4.24= The pancreas utilizes more glucose during exercise.

#3 Rational.24= Exercise promotes the passage of glucose into muscle cells, so extra complex carbohydrates are essential before or during regular exercise. To prevent precipitous hypoglycemia, the IDDM patient should inject insulin into nonactive areas, such as the abdomen.

Question.25= Which of the following behaviors would indicate improvement in the coping behavior of a patient who has the diagnosis of depression?

Sleeping frequently during the day. Answer2.25=Going to the dining room to eat. Answer3.25= Turning off the television to have it quiet. Answer4.25=Initiating interaction with another patient.

#4 Rational.25= If a patient initiates interaction with another patient, it indicates he is not totally absorbed in himself, too depressed to initiate a conversation, or too preoccupied to focus on the television. The other choices do not necessarily indicate improvement in coping.

Question.26= When a patient`s hallucinations become louder, more demanding and difficult to ignore, the nurse would judge that his mental status is

mproving. =Showing more evidence of paranoia. = Remaining the same. =Deteriorating.

Rational.26=The more demanding and absorbing hallucinations (hearing voices) becomes, the more the patient`s condition may be deteriorating. Secondarily, this may indicate increased paranoia, but not necessarily since paranoid schizophrenia is only one form of this condition.

Question.27= A 3-day-old infant is admitted to the pediatric floor from the nursery with a diagnosis of esophageal atresia type III. His mother comes to visit her baby and says to the nurse, "I feel as though I`ve done something wrong to make my child sick." The most appropriate response would be to #1 Rational.27= The nurse acknowledges the mother`s feelings, but tries to show that they are not based on fact.

can understand your feelings, but remember that this is a congenital defect you did not cause." "It does no good to feel that way. Your child is sick and needs you. You should spend your time caring for him." = "Don`t be silly, your child was born with this. You`ve done the best you can." = "A lot of mothers feel guilty when their child is sick."

Question.28= A 43-year-old female patient had a subtotal gastrectomy several weeks ago. She has been readmitted with a diagnosis of pernicious anemia. Taking her nursing history, the nurse expects to find that her diet has probably not included #2 Rational.28= Patients who have pernicious anemia following a subtotal gastrectomy do not have enough intrinsic factor to utilize vitamin B12 in foods and must be given injections or sublingual doses of vitamin B12.

Eating enough foods high in iron. Answer2.28=Taking her vitamin B12. = Eating enough high vitamin B12 foods.

=Taking her iron supplements.

Question.29= A patient who is hospitalized with chronic congestive heart failure has been placed on telemetry. The nurse should know that the purpose of this type of monitoring is to

dentify ventricular arrhythmias resulting from hypoxia. = Identify ventricular arrhythmias resulting from hyperkalemia. = Assess atrial ectopic beats resulting from hypoxia. = Assess ectopic beats resulting from hyperkalemia.

#1 Rational.29=When the lungs are filled with fluid, oxygenation is not as efficient as it should be. Hypoxia can cause ventricular arrhythmias, a dangerous condition. These are identified easily by the use of telemetry.

Question.30= A neighbor asks the nurse to look at her 3-year-old child`s rash. Her face, neck and chest are covered with a maculopapular rash. She appears feverish and her nose and eyes are "running." There are small red spots with bluish-white centers on the mucosa of her mouth. The nurse recognizes that these manifestations are most likely caused by

Measles (Rubeola). = Impetigo (Staphylococcus). = Scarlet fever (Beta hemolytic streptococcus, Group A). = German measles (Rubella).

#1 Rational.30= Rubeola is a highly contagious virus. It is more severe than rubella or roseola because of the complications. The virus is transmitted by a cough or sneeze.

Question.31= In each prenatal visit, the assessment technique that evaluates appropriate fetal growth is Measurement of fundal height. =A nonstress test. = Maternal weight gain. =Blood pressure reading.

#1 Rational.31= The fundus height changes throughout pregnancy. The height of the fundus is at the level of the umbilicus by 28 weeks and near the xiphoid process by 38 weeks.

Question.32= A direct Coombs` test is done on the cord blood of a newborn to detect the presence of #2 Rational.32= The direct Coombs` test is done on cord blood to detect the presence of maternal antibodies attached to the neonate`s red blood cells.

Antigens coating the mother`s red blood cells. =Antibodies coating the baby`s red blood cells. = Maternal red cells in the fetal circulation. =Fetal red cells in the maternal serum.

Question.33= Persons with personality disorders tend to be manipulators. Which principle is it important for the nurse to know in planning the care of such a person? #3 Rational.33=Because a person with this disorder tends to manage his or her life through manipulation of others, when it doesn`t work, their anxiety level goes up. The nurse should never allow the patient to manipulate her. Answers (2) and (4) are not true.

The nurse should allow herself to be manipulated so as to not raise the patient`s anxiety. = The establishment of a nurse-patient relationship will decrease the patient`s manipulations. = When the patient`s manipulations are not successful, his anxiety will increase. = The nurse should appeal to the patient`s sense of loyalty in adhering to the rules of the community.

Question.34= A patient is aware that the biopsy report has just returned. As the nurse is changing his dressing he says, "Do I have cancer?" The most appropriate response at this time is #1 Rational.34= This response allows the patient to discuss fears about cancer and does not close off communication, as the other three responses do.

"Would you like to discuss your concerns about the results?" = "Did your physician tell you the results?" = "I`ll call your doctor and ask him." = "We do not know yet; the report has not returned."

Question.35= A patient with rheumatoid arthritis has been on aspirin gr. xx TID and prednisone 10 mg BID for the last 2 years. The most important assessment question related to the patient`s drug therapy is to ask her if she experiences

Blurred vision. =Decreased appetite. = Tarry stools. =Headaches.

#3 Rational.35= Aspirin impedes clotting by blocking prostaglandin synthesis which can lead to bleeding. A side effect of prednisone is gastric irritation, also leading to bleeding. Tarry stools indicate bleeding in the upper GI system.

Question.36= A 40-year-old male patient has the diagnosis of myasthenia gravis; the disease is adequately controlled on Mestinon. The patient tells the nurse that sometimes he gets very tired when he eats. Nursing instructions should be to

Eat several small meals each day. =Eat foods rich in vitamin B6. = Take Mestinon 30 minutes before meals. =Try pureed foods until this symptom decreases.

Rational.36= Myasthenia is a disease characterized by marked weakness and abnormal fatigue of voluntary muscles. A deficiency of acetylcholine results in a defect in transmission of nerve impulses. Mestinon, an anticholinesterase medication, increases levels of acetylcholine. Taking the medication before meals will assist in mastication.

Question.37= One afternoon the nurse finds a paraplegic patient sitting in her wheelchair crying. When the nurse asks if she can help, the patient says, "Go away, no one can help." The best nursing response is #1 Rational.37= The best response is to acknowledge she told the nurse to go away but to leave communication open for a later time. Other responses put pressure on the patient or close off communication.

"I understand that you don`t want to talk right now. I will come back and perhaps we can talk." = "You must be having problems." = "Many people feel sad in the hospital. You`ll be home soon." = "It`s better if you talk about it. Maybe I can help."

Question.38= An adult male patient is being discharged after his peptic ulcer has been controlled on a medical regimen. One medication he is taking is sucralfate (Carafate). The nurse asks if he understands when he should take the medication. The correct answer would be

#2 Rational.38= Carafate is a mucosal protective agent with antipepsin activity. It is taken on an empty stomach, 1 hour before eating, and at bedtime.

Just before meals.=One hour before meals and at bedtime. = In early morning. =With every meal.

Question.39= A new mother is told that on admission to the nursery an injection of vitamin K is given routinely to normal newborns. The nurse explains that vitamin K is given because

ewborns have an increased ability to clot blood. = Vitamin K promotes a normal clotting time. = Vitamin K prevents hypoglycemia. = Newborns need to be protected from hyperbilirubinemia.

#2 Rational.39=Vitamin K is used to promote a normal clotting time until bacteria, necessary for vitamin K synthesis, is present in the colon. Vitamin K does not protect the newborn from hyperbilirubinemia, nor does it play a role in blood glucose regulation.

Question.40= A 60-year-old male patient has been diagnosed with heart failure and is started on Digoxin 0.25 mgm PO QD and Lasix 40 mgm PO BID. After 2 days, he develops an irregular pulse rate of 64. The nurse would expect the physician to order #4 Rational.40=Potassium loss secondary to Lasix administration may cause arrhythmias and potentiate the risk of digitalis toxicity. Therefore, potassium is often ordered when Lasix and digitalis are prescribed. Calcium gluconate. =Magnesium sulfate. = Atropine sulfate. =Potassium chloride.

Question.41= When a patient experiences a severe anaphylactic reaction to a medication, the nurse`s initial action is to Prepare equipment for intubation. =Start an IV following standard orders. = Place the patient in a supine position. =Assess the patient`s vital signs. #3 Rational.41= Initially, the shock position is necessary to maintain vital signs. The other interventions may be carried out, but will follow the initial action.

Question.42= The most appropriate needle used for administering an intramuscular injection into the deltoid muscle of an adult patient is a #1 Rational.42=A 23 or 25 gauge 5/8" needle is the most appropriate size when using the deltoid muscle.

25 gauge, 5/8" needle. =22 gauge, 1" needle. = 25 gauge, 1 1/2" needle. =22 gauge, 1 1/2" needle.

Question.43= Which of the following statements best explains why premature infants are more likely to develop an infection than full term newborns? #1 Rational.43= Antibody formation is immature in the premature infant. Immaturity of the liver is responsible for hyperbilirubinemia but is not directly related to the infant acquiring an infection. White blood cells may be ineffective, but that is not necessarily related to full-term birth.

Premature infants receive few antibodies from the mother, because antibodies pass across the placenta during the last month of pregnancy. = Liver enzymes are immature in the premature infant. = Antibody formation does not mature until 6 weeks after birth of a full-term infant. = The white blood cell defense system is only effective at full term.

Question.44= A premature infant has the diagnosis of respiratory distress syndrome (RDS). The nurse understands that this disorder may be caused by Low concentration of oxygen. =The inability to produce surfactant. = Cold stress. =A genetic defect. #2Rational.44=RDS often occurs in premature infants due to decreased surfactant production in the lungs. A high concentration of oxygen can also result in this condition.

Question.45= A 70-year-old female Alzheimer`s patient has been wandering in and out of other patients` rooms. To deal with this behavior, the most therapeutic nursing action will be to

Lock the other patients` rooms. =Restrain the patient in her room. = Confine the patient to the dayroom. =Tell the patient she shouldn`t do tha

#3 Rational.45= The most therapeutic action is to keep the patient with others in the dayroom, not alone in her room. Telling her not to go into others` rooms will not change her behavior because Alzheimer`s patients have little short-term memory. Locking rooms may be perceived as punishment by the other patients.

Question.46= A male patient with bronchospasms has prednisone prescribed. The nurse`s teaching instructions before he is discharged from the hospital will include taking prednisone #1 Rational.46= Prednisone may cause a stress ulcer, so it should be taken with milk to protect the lining of the stomach. Iron is given with orange juice to increase assimilation.

With milk. =With orange juice. = Before meals. =After meals.

Question.47= A patient comes to the walk-in mental health clinic where the LVN is assigned to do intake histories. To determine the presence of depression, the primary sign or symptom the nurse will assess for is

Mood with affect of sadness. =Episodic euphoria. = Loss of reality focus. =Negative behavior.

#1 Rational.47= The distinguishing quality of depression is mood, where the affect is sadness or gloom. Negative behavior could be present, but also accompanies other diagnoses. Loss of reality is seen in schizophrenia, and euphoria is a symptom of manic episode.

Question.48= The physician orders a nasogastric tube for a female patient. As the nurse prepares to insert the NG tube, the patient begins to cry silently. The best way for the nurse to respond to this behavior is to say #3Rational.48= Crying is not a normal response to an NG tube insertion, so the nurse needs to find out what has occurred in the patient`s past to evoke such a response.

Don`t worry, I will give you pain medicine if you need it." = "It is all right to cry." = "Can you talk about why you are crying?" = "The insertion of the tube will not be painful."

Question.49= A patient scheduled for surgery has had preoperative teaching. An indication that the teaching was successful would be that the patient #2Rational.49= The person who understands something can explain it to someone else. The other responses are typical of stress responses and/or denial.

Does not question what the nurse said. = Appears relaxed and can verbalize what the nurse taught. = Says he understands and wants to be alone now. = Appears to be resting quietly.

Question.50= After a male patient returns from surgery following small bowel obstruction, he complains of pain and asks for medication. The first nursing action is to

Determine the location and severity of the pain. =Administer the medication that is ordered. = Give him half the ordered dosage. =Check with the physician.

#1Rational.50=Because the pain might be caused from his position on the operating table or the position of his arm with the IV, it may be relieved without medication by actions such as repositioning or giving a back rub. The patient should receive medication if he needs it, but medication should not be used in place of good nursing care. The first dose of narcotic following surgery may be only one half the ordered dose, but an assessment of the pain should be done first.

Question.51= A female patient has been admitted with a diagnosis of compulsive disorder. In talking with her one day, she tearfully asks the nurse, "Do you think I`m crazy?" The nurse`s best response would be #4 Rational.51=This is the only response that opens communication. The nurse should encourage the patient to express what she is thinking and feeling in order to assist her to deal with her fears.

That depends on your definition of crazy." ="You`re upset. Let`s talk about something else." = "I think you`re upset, but not crazy." ="Are you concerned about being crazy?"

Question.52= The physician has ordered IV fluids at 100 mL/hr. Which one of the following parameters is the most important indicator of fluid balance? #4 Rational.52= Hourly urine output is the best indicator of fluid balance. Vital signs will change however, they are slower to respond to fluid changes than alterations in urine output.

Adequate intake measurement. =Quality of skin turgor. = Consistent vital signs. =Hourly urine output.

Question.53= The nurse assigned to a woman in labor has orders to begin an oxytocin (Pitocin) drip. After 20 minutes of the Pitocin infusion, the nurse observes a contraction that does not relax after 90 seconds. The nurse`s first action is to

Start oxygen by mask. =Stop the Pitocin drip. = Notify the physician. =Turn the patient on her left side.

#2Rational.53= If a contraction lasts longer than 90 seconds, the safe correct action is to turn off the Pitocin. Prolonged contractions can result in a ruptured uterus. The nurse may then administer oxygen and call the physician.

Question.54= When the nurse is caring for a patient in labor, which one of the following signs indicates that the placenta has detached from the uterine wall? #4Rational.54= As the placenta is expelled and moves toward the cervix, the cord will precede it and appear to get longer.

The uterus falls below the level of the symphysis pubis. s = There is a noticeable relaxation in the abdominal wall. = The mother complains of pain low in her back. = The cord lengthens outside of the vagina.

Question.55= A 48-year-old patient was admitted 2 days ago with the diagnosis of acute interstitial pancreatitis. The nurse will evaluate for the most common and fatal complication of severe acute pancreatitis, which is

Severe hyperglycemia. =Hypovolemia. = Electrolyte imbalance. =Infection.

#2Rational.55= This condition causes large amounts of serous fluid to leak from the blood channels into the peritoneum, which decreases the intravascular volume. Hypovolemia can occur. Enzyme release may also damage the walls of the blood vessels and precipitate hemorrhage.

Question.56= In counseling a patient, which measure will promote the most comfort during a Herpes Simplex Virus, type 2 (HSV-2) outbreak? Keep the lesions clean and dry. =Apply Acyclovir as ordered. = Take sitz baths 3 to 4 times a day. =Apply a local anesthetic or systemic analgesia.

#2 Rational.56=Acyclovir applied to the lesion will result in earlier remission and also provide more comfort. There is no known antibiotic for the virus. Acyclovir applied to the lesions shortens the episode but does not cure it.

Question.57= A male patient is admitted who is semicomatose, dyspneic, and weak. His admitting diagnosis is HIV immune depression. The statement that best describes the source of this disease is a/an HIV is caused by a retrovirus with a different life cycle from that of a normal virus. It may be dormant for years before affecting the helper T-cells and causing immunosuppression.

Mutated virus that kills helper T-cells. = Autoimmune disease that is caused by the release of an antigen. = Virus that invades the immune system. = Retrovirus that attaches to the helper T-cells.

Question.58= A female patient, age 58, has been hospitalized for 2 days with chronic congestive heart failure. While administering morning care, the nurse observes that the patient frequently removes her oxygen mask even though she is dyspneic. The appropriate nursing intervention is to Increase the liter flow of O2 to maximum level. =Change from O2 mask to O2 cannula. = Change O2 administration to a Venturi mask. =Tighten the strap on the O2 mask.

#2Rational.58= Patients often feel that they cannot breathe when experiencing pulmonary edema. A mask may increase this feeling. A cannula is often better tolerated and thus should be used in this case.

Question.59= A female patient has been admitted to the coronary care unit with a diagnosis of myocardial infarction. She is placed on a cardiac monitor and has an IV of D5W at a "keep open" rate. Initially, the nurse will assess for

Chest pain characteristics. =Apical pulse. = Blood pressure. D =Heart sounds.

#1 Rational.59= The presence of chest pain may cause the pulse rate to increase and blood pressure to drop. It can also increase patient anxiety. The nurse`s priority is to assess the patient for pain. The patient is on a cardiac monitor therefore, an apical pulse is not the priority action. Once her comfort has been established, apical pulse, blood pressure, and heart sound determinations are appropriate.

Question.60= A nurse colleague working on the pediatric unit is newly married and wishes to become pregnant. Which of the following tests would it be important for her to have as soon as possible? Lipoprotein test. =Papanicolaou (PAP) smear. = Serologic test for Hepatitis A. =Rubella Viral Serologic test.

#4Rational.60= If the woman is planning to become pregnant, exposure to the rubella virus is important to detect. If a woman is exposed and she is not immune, the fetus may have congenital abnormalities at birth. A Pap smear detects cancer.

Question.61= Of the following interventions, which action should the nurse take first if the cord has prolapsed in a woman in the final stage of labor? #1Rational.61= The first action is to move the presenting part off the cord to allow oxygen to flow via the cord. Other methods that can be used to accomplish this are Trendelenburg or knee-chest position. Turning to the left side (1) will not help (2) and (3) are wasting valuable time.

Pushing upward on the presenting part with a sterile-gloved hand. = Reporting the fetal heart rate to the physician immediately. = Replacing the cord very slowly with a sterile-gloved hand. = Turning the patient onto her left side.

Question.62= A mother in labor arrives at the emergency room. An initial assessment indicates that crowning is occurring. The first nursing action is to #2Rational.62= The first action is to support the perineum to prevent tears and rapid delivery. The nurse would ask the patient to take short breaths (following instructions) but to prevent pushing the nurse will also send someone for a physician.

sk the patient to push according to the nurse`s instructions. Apply gentle perineal pressure to prevent rapid expulsion of the head. = Notify the charge nurse or physician. = Instruct the patient to take short shallow breaths to improve fetal oxygenation.

Question.63= A patient in labor is experiencing severe back pain. The best source of relief for the mother would be for the nurse to Divert her attention to the TV. =Apply counterpressure to her sacrum. = Have her walk around. =Give her a narcotic analgesic. #2Rational.63=Applying counterpressure will help to release the pain. Moving the patient into knee-chest position is also good for back pain.

Question.64= At the well-baby clinic, a mother tells the nurse she is worried about her 1 month old "catching something" before she gets her shots. What would be the most appropriate response? #4Rational.64= The mother can be reassured by knowing that passive immunity for diphtheria, tetanus and pertussis are all transferred from mother to infant during fetal life. There is very little functioning maternal antibody in the infant after age 6 months.

Protection from your antibodies will last until she is at least 1 year old." = "She has passive immunity to tetanus and pertussis but not to diphtheria so she needs to be protected from it by early shots." = "She has active immunity from you so she won`t be affected by any of the usual childhood diseases." = "The baby is protected for the first few months of life so you need not worry."

Question.65= Before administering an immunization to an infant, it is important that the nurse remember that a primary contraindication for immunization is #1 Rational.65=Children with active infections such as impetigo should not be immunized until the infection has passed.

An active infection. =Cystic fibrosis. = Congenital heart disease. =Failure to thrive.

Question.66= A 41-year-old male patient has had recurrent sharp flank pain, nausea, and vomiting for 24 hours. He is admitted to the hospital for a genitourinary work-up. Which of the following orders would be considered a priority?

Temperature every 4 hours. =Strain all urine. = Administer an antiemetic every 4 hours. =Accurate intake and output records.

#2Rational.66= The patient has symptoms indicative of a urinary calculus therefore, it is important to strain all the urine in order to detect if the stone has passed.

Question.67= On a home visit to a male patient who has had multiple sclerosis for 20 years, the nurse reinforces the nursing care the wife provides to her husband. Which of the following measures would be most appropriate to include in the teaching sessions?

Instruction in weight control. = Exercises that promote muscle strengthening and decrease tremors. = Importance of regular bowel and bladder evacuation. = Side effects of routine medications.

#3Rational.67= Bowel and bladder retention or incontinence is a major problem of patients with multiple sclerosis therefore, establishing a good routine for evacuation is essential.

Question.68= The nurse observes a patient walking down the hall carrying a knife in his hand, and saying in an angry voice, "Get out of my way. Don`t try to stop me." The nurse`s most appropriate response would be #1Rational.68= The nurse`s first priority is to give the patient space and protect herself. Moving in on the patient or showing resistance may agitate him more. She should not move too close to the patient until there is sufficient manpower to swiftly and effectively handle the situation.

Move away from him and find an area of safety, or put a piece of furniture between herself and the patient while yelling for assistance. = Hold her ground and quietly ask him to give her the knife while holding out her hand. = "You appear angry. Perhaps it would help to talk about it." = "Knives are not allowed on the unit. I want you to give it to me immediately!"

Question.82= Collecting physical data on an infant at the well-baby clinic, the nurse will observe which of the following signs in an infant with congenital hip dysplasia? #4 Rational.82= Abduction is limited in the affected leg. Assessment will also reveal asymmetrical gluteal folds, an absent femoral pulse when the affected leg is abducted, and a hip "click" on abduction.

Limited adduction of the affected leg. = Symmetrical gluteal folds. = Femoral pulse when the hip is flexed and the leg is abducted. Limited abduction of the affected leg.

Question.83= A male patient is unable to sleep. He is pacing the floor, head down, and wringing his hands. The nurse recognizes that he is anxious. What is the most appropriate intervention? #1 Rational.83= This is a more comprehensive answer and includes the others. Sleeping medication should be avoided if at all possible or unless absolutely necessary, because it only temporarily helps suppress the patient`s feelings.

Let him know you are interested and willing to listen. = Explore with him the alternatives to pacing the floor. = Give him his PRN sleeping medication. = Encourage him to talk about his behavior.

Question.84= A pregnant patient has been advised to increase her iron intake. The nurse will know she understands the dietary teaching if she says that she should take the iron supplement With vitamin C or juice. =With milk. = With her other prenatal supplements. =At mealtime.

#1 Rational.84= It is most important to take iron with vitamin C, such as orange juice, because this vitamin helps the body assimilate the iron. Iron should be taken on an empty stomach to increase assimilation. Eggs or milk products interfere with assimilation.

Question.85= In a 2 year old with increased intracranial pressure, which one of the following signs would be cause for alarm? Diminishing sunset sign.= Absence of nystagmus. = Equal pupils that react to light. =Increasing lethargy and drowsiness.

Rational.85=Increasing lethargy and drowsiness, although not specific in themselves, are signs that would lead the nurse to investigate further since level of consciousness is an excellent indicator of increased intracranial pressure. The other answers-diminishing sunset signs, equal pupils, and absence of nystagmus-are all normal signs.

Question.86= A patient`s chart states he has a stage III gluteal pressure ulcer. When assessing the patient, the nurse would expect to find a(n) Reddened, nonblanching area of intact skin. =Area of abraded skin. = Open wound with loss of subcutaneous tissue. =Deep crater with visible muscle tissue. #3Rational.86= Standardized pressure ulcer staging systems progress from stage I (red nonblanchable, intact skin) to stage IV (deep crater with visible underlying structures such as bone, muscle, tendons).

Question.87= When assessing the following patient care situations, which one would require the nurse to wear goggles? Changing a dressing. Administering an IM injection. = Catheterizing the urinary bladder. =Emptying a Foley drainage collection bag. #4Rational.87=Goggles should be worn when there is a risk of contact with body fluids through spattering or splashing.

Question.88= The nurse is taking the history of a patient who is scheduled for diagnostic tests, including an MRI. Which reported condition is of greatest importance to know about when the nurse is preparing the patient for an MRI? #3 Angina. =Iodine allergy. = Claustrophobia. =Hypertension.

Question.91= Which intervention is most useful when communicating with an aphasic patient? Use correct medical terminology when teaching or explaining. = Ask open questions to obtain information. = Repeat the same word until the patient understands. = Provide frequent praise and encouragement.

#4Rational.91= Aphasia leads to feelings of frustration and discouragement. Patients require frequent praise for their efforts and encouragement to continue to try to communicate. Aphasic patients can best provide information by responding to closed questions that require answers such as Yes or No. If the patient does not understand a word, use a different word rather than repeating. Use clear, simple, consistent language to teach or explain avoid complex medical terminology.

Question.92= Which medication would the nurse expect to be ordered for an alert patient who is experiencing severe headache after a head injury Morphine. =Acetaminophen (Tylenol). = Ibuprofen (Motrin). =Aspirin.

#2 Rational.92= After a head injury, patients should not receive medication that might cause drowsiness or depress the central nervous system (such as morphine or ibuprofen) or that may interfere with blood clotting (such as aspirin or ibuprofen). Acetaminophen does not have these undesirable effects.

Question.94= A patient is NPO and is receiving continuous nasogastric tube feedings. Select the correct nursing action related to administering medication through the patient`s nasogastric tube.

Flush the tube with water after each medication. = Stop the tube feeding for 30 minutes before and after medication instillation. = Add the medications to the tube feeding solution. = Mix the medications together and slowly instill as a bolus by gravity.

#1 Rational.94= Each medication should be instilled separately and followed by 10 mL of water to prevent obstruction of the tube. Medications are not added to the feeding solution because of possible incompatibilities, difficulty maintaining the medication schedule, and delay in medication absorption because of the slow flow of continuous tube feedings. Continuous tube feedings are interrupted only while the medication is being administered.

Question.95= The nurse is assigned a patient who is potentially suicidal. Of the following nursing objectives, which one is the most important? #4Rational.95= Because it is unrealistic to observe a patient every minute, the environment must be kept safe for patient protection. Answer (2) is important, but not the most important.

Involve the patient in activities with others to mobilize him. = Recognize a continued desire to commit suicide. = Observe the patient closely at all times. = Provide a safe environment to protect the patient.

Question.96= The major advantage of monitoring the fetal heart rate electronically is that this method akes less nursing time to use. =Records the actual heart beat. = Determines beat-to-beat variabilities. =Causes less discomfort to the mother.

#3Rational.96= A continuous record of the fetal heart rate and its relationship to uterine contractions is done through fetal monitoring to show beat-to-beat variability. Fetal monitoring can be done by both internal and external devices.

Question.97= A male patient has just had a cataract operation without a lens implant. In doing discharge teaching, the nurse will instruct the patient`s wife to #2Rational.97= Without a lens, the eye cannot accommodate. It is difficult to judge distance and climb stairs when the eyes cannot accommodate.

Keep the eye dressing on for 1 week. = Allow him to walk upstairs only with assistance. = Feed him soft foods for several days to prevent facial movement. = Have her husband remain in bed for 3 days.

Question.98= The nurse has just inserted a nasogastric tube for drainage into a young male patient and must check placement. The most accurate method is to

Insert air into the tubing and with a stethoscope, listen for a "whish" sound. = Aspirate the stomach contents and test with litmus paper. = Place the tip of the nasogastric tube in a glass of water and observe for bubbling. = Send the patient to x-ray for an abdominal film as ordered.

#2Rational.98= Checking the aspirate with litmus paper indicates tube placement. An acidic response means the tube is in the stomach. Many instructors continue to teach the air insertion technique however, this is NOT the currently accepted method. An x-ray check will be used if tube feedings are to be administered.

Question.99= A 23-year-old female patient was in an automobile accident and is now a paraplegic. The orders are for an intermittent urinary catheterization program and diet as tolerated. A priority assessment should be to observe for #4 Urinary retention. Answer2.99=Bowel evacuation. Answer3.99= Weight gain. Answer4.99=Bladder distention.

Question.101= A female patient has been in the second stage of renal failure for several months. Her condition has deteriorated to uremia, and she is now being admitted to the hospital. Checking her lab values, the nurse will expect to see

BUN 180, Creatinine 18, Urine specific gravity 1.010. Answer2.101= BUN 35, Creatinine 16, Urine specific gravity 1.035. = BUN 10, Creatinine 12, Urine specific gravity 1.020. = BUN 100, , Urine specific gravity 1.005.

#1Rational.101= While many factors affect the BUN, creatinine is a very specific indicator of renal function. Both the BUN and creatinine are elevated in renal failure, frequently with a 10:1 to 20:1 ratio. Specific gravity that stabilizes at 1.010 indicates the kidney has lost ability to concentrate or dilute urine.

Question.102= A patient with a radioactive implant moves in bed and the implant becomes dislodged. The priority nursing intervention is to

Put on a lead apron and gloves and place the implant in a covered container. = Wait until a radiation team can be called to recover the implant. = Leave the room as quickly as possible and report to the head nurse. = Use long-handled tongs and place the implant in a lead box kept in the patient`s room.

#4

Question.103= A 54-year-old female patient is 3 days postoperative following abdominal surgery. The lab results indicate that her white blood cell count is 8,000/cu mm. After analyzing this lab report, the nursing action is to Rational.103= This is a normal cell count (the range is 4500 to 11,000/cu mm) so the nurse would do nothing except note that it is normal.

equest that the lab repeat the test, as results don`t make sense. = Contact her physician so he will be aware of the high abnormal count. = Do nothing because this is a normal count. = Contact the patient`s physician so he will be aware of the abnormal low count

Question.108= When evaluating a patient`s understanding of a low potassium diet, the nurse knows that he understands if he says that he will avoid Rational.108=Raw apples are high in potassium, while white-enriched and French bread, dry cereal, and pasta are foods low in potassium.

Pasta. =Dry cereal. =French bread. =Raw apples.

Question.109= Irrigating a nasogastric tube should be carried out using which one of the following protocols? #2Rational.109= Gentle pressure is necessary when irrigating a nasogastric tube to prevent damage to the stomach wall. Normal saline is recommended to prevent electrolyte loss through gastric drainage.

Instill 30 mL sterile saline, forcefully if necessary, and allow fluid to flow into basin for return. = Gently instill 20 mL normal saline and then allow fluid to flow into basin for return. Gently instill 20 mL normal saline and then withdraw solution. = Instill 30 mL sterile water and then withdraw solution.

Question.110= A patient is scheduled for a voiding cystogram. Several hours before the test, physician`s orders will instruct the nurse to #2Rational.110= Forcing fluids ensures a continuous flow of urine to provide adequate urine output for specimen collection and, also, to prevent multiplication of bacteria that may be introduced during the procedure.

Medicate with urinary antiseptics. =Force fluids. = Maintain NPO =Administer bowel preparation.

Question.111= Following a cystoscopy, the nurse will evaluate the patient`s condition for the complication of Burning on urination. =Difficulty voiding. Development of cold chills. =Pink-tinged urine. 3 Rational.111= Cold chills could indicate the spread of infection throughout the urinary tract. `

Question.114= Collecting growth and development data on the social development of a 1-month-old male infant, the nurse knows that it should include

Responding to "No, no." = Turning his head to a familiar noise. = Actively following movements of familiar persons with his eyes. = Discriminating between family and strangers.

#3Rational.114= Actively following movements would occur at 1 month. Responding to "No" and turning the head in response to noise begins at 4 months, and discrimination between family and strangers appears at 5 months of age. This is considered intellectual or cognitive development.

Question.118= Following dialysis of a patient, the nurse expects to find an improvement in the blood values of Hypokalemia. =Low hemoglobin. = High serum creatinine levels. =Hypocalcemia. #3

Question.119= A 3-month-old infant has unrepaired Tetralogy of Fallot. Which of the following signs and symptoms would the infant be expected to exhibit? achycardia, hypertension, decreased femoral pulses. = Hypotension, bradycardia, dyspnea. = Circumoral cyanosis, hypoxic spells, feeding fatigue. = Cyanosis, tachypnea, hypertension in upper extremities. #3Rational.119= Cyanosis, hypoxia, and feeding difficulties are classic manifestations of cyanotic heart disease.

Question.120= An adult patient with a tentative diagnosis of antisocial personality refuses to participate in unit activities. When he is on the unit, he makes fun of the other patients. Considering the patient`s diagnosis and behavior, which one of the following nursing plans would be most effective for the nursing staff to follow?

et the patient know the rules on the unit. Answer2.120= Allow the patient to isolate himself so that he does not upset the other patients. Answer3.120= Confer with the patient, his psychiatrist and the staff about the patient`s lack of participation on the unit. = Require the patient to participate in activities.

3 Rational.120=In dealing with manipulative behavior, it is important that all members of the team as well as the patient are clear about expectations. (1) pits the nurse against the patient (2) is incorrect because it is not therapeutic to isolate a patient and (4) is nontherapeutic because the patient is not involved in the treatment.

Question.121= A 56-year-old patient admitted to the hospital with angina pectoris is scheduled to have a cardiac catheterization. While filling out a menu for the following day, the patient says to the nurse, "I always get a rash when I eat shellfish." In reading the patient`s chart, the nurse sees no such notation. Following safety protocol, the initial nursing intervention should be to #1

sk the nurse in charge to communicate the patient`s reaction to shellfish to the physician. Answer2.121= Ask the patient if there are any other foods that cause such a reaction. Answer3.121= Notify the dietitian of the patient`s reaction and request a "no shellfish" diet. Answer4.121= Place a note on the chart regarding the patient`s reaction to shellfish.

Question.122= A few hours after a heart catheterization, the nurse notes that the patient`s blood pressure has decreased from 136/72 to 110/64 mmHg. The patient has been voiding large amounts of urine. The nurse should recognize that this blood pressure change is probably due to

The patient going into cardiac failure. Answer2.122= A reaction to the dye used during the procedure. Answer3.122= Insufficient fluid intake for the past 12 hours. Answer4.122= Lost fluids because of the diuretic effect of the dye.

#4Rational.122= The dye used during the cardiac catheterization is hyperosmolar and, therefore, acts as a diuretic. As fluid volume drops, the arterial blood pressure drops.

Question.123= A patient is admitted to the labor room with contractions that are about 4 to 5 minutes apart and 30 seconds long. The couple have attended Lamaze childbirth classes. During transition, the nurse should support a breathing pattern that is

Slow, deep abdominal pattern. Answer2.123= Shallow chest breathing at 20 to 30 times a minute. Answer3.123= Shallow chest breathing at 40 to 60 times a minute. Answer4.123= Slow, deep chest breathing.

Rational.123=Shallow chest breathing, slightly faster than a normal rate, is the pattern used most in transition. Slow breathing, while effective for some women during transition, is usually most effective in earlier labor. Supporting a more rapid rate of 40 to 60 times a minute is not physiologically sound and could lead to hyperventilation.

= The nurse is assigned to administer two insulin solutions, NPH and regular insulin. The appropriate procedure of combining the insulin solutions in one syringe is to

Answer1.125= Inject air and withdraw NPH, then withdraw regular insulin into the same syringe.

Answer2.125= Inject air into NPH bottle, then inject air and withdraw regular insulin, then withdraw appropriate amount of NPH into the same syringe.

Rational.125=Regular insulin is withdrawn into the syringe after injecting the prescribed amount of air into the NPH bottle. The last step is to withdraw the NPH insulin into the same syringe.

Answer3.125= Withdraw NPH and regular insulin into separate syringes and give two injections.

Answer4.125= Inject air and withdraw regular insulin, then inject air and withdraw appropriate amount of NPH into the same syringe

A 21-year-old patient is described as bizarre, withdrawn, talking and laughing to herself. Orders include chlorpromazine (Thorazine) 200 mg, PO BID. If refused, 100 mg IM may be administered q 4 hours PRN for agitation. Benztropine mesylate, MSD (Cogentin) 5 mg BID is also prescribed. Assessing the patient, the nurse finds her screaming and banging her hands on the wall. She refuses oral medication. The best nursing action would be to

Answer1.126= Stay with the patient and get assistance to give her the IM injection of Thorazine.

Answer2.126= Put the patient in restraints until she calms down, then administer Thorazine.

#1 Rational.126=The patient`s behavior indicates that she needs Thorazine. To decrease the amount of external stimuli, the nurse must provide limits because the patient cannot staying with her will be reassuring.

Answer3.126= Assign another nurse to administer both Thorazine and Cogentin.

Answer4.126= Tell the patient firmly to stop hitting her hands or she will hurt hersel

Question.127= The nurse finds a patient on the psychiatric unit sitting quietly, staring out the window, and laughing inappropriately. The nurse should recognize that the patient is probably

Getting ready to "act out."

3Rational.127= Inappropriate laughter in psychotic behavior is generally caused by visual or auditory hallucinations.

Answer2.127=Laughing instead of crying about the situation. Answer3.127= Hallucinating. Answer4.127=Seeing a funny scene.

Question.128= In the last few months, a 56-year-old patient has noticed that he has been "slowing up" in all of his movements and that his speech has become indistinct. The physician makes a diagnosis of Parkinson`s disease. When teaching the patient about Parkinson`s disease, the nurse should include encouraging the patient to

aintain employment as long as possible. Answer2.128=Retire and take part in limited social activities. Answer3.128= Immediately decrease his work schedule. Answer4.128=Work only when he feels up to it.

1Rational.128=It is very important to keep patients with Parkinson`s disease actively involved in work and social situations. They experience altered body images and tend to withdraw from society. When this happens, the complications of immobility increase. As complications increase, they tend to decrease patients` life spans.

Question.129= A 28-year-old primipara is admitted to the hospital in early labor. She is comfortable and visiting with her husband when her water breaks. Which of the following nursing assessments should be completed first? 3Rational.129=When the water breaks, there is a possibility of prolapsed cord, which must be ruled out immediately. This is the first nursing assessment. Following this intervention, the nurse would check the fetal heart rate.

Monitor the fetal heart rate. Answer2.129=Attach the external fetal monitor. Answer3.129= Check for prolapsed cord. Answer4.129=Note color and quantity of fluid.

Question.135= A postsurgical patient develops bacterial pneumonia. When assessing her condition, which of the following signs or symptoms would the nurse expect to be present Rational.135= Bacterial pneumonia causes an increase in temperature, so the nurse would know that it may be present when the patient`s temperature is above 37.7 degrees C (100 degrees F). This type of pneumonia would not result in hypertension, irregular pulse or even shallow, irregular respirations. The primary symptoms, in addition to a high temperature, are rapid onset, constant cough, pleuritic pain, anxiety and dyspnea.

Irregular pulse.

Answer2.135=Hypertension.

Answer3.135= Temperature above 37.7 degrees C (100 degrees F).

Answer4.135=Shallow, irregular respirations.

Question.136= Before a patient who has gout is discharged from the hospital, it is important to evaluate his knowledge of dietary management. Which one of the following diet choices would indicate to the nurse that he understands his dietary restrictions?

Crab cakes, rice and peas.

Rational.136= Steak is the best choice because foods highest in purine include shellfish, liver, chicken, beans, and various vegetables. The appropriate diet will include high carbohydrates with calorie control.

Answer2.136=Antipasto salad, rice and asparagus.

Answer3.136= Liver, a potato and spinach.

Answer4.136=Steak, baked potato and green salad.

Question.137= A 3-year-old child is brought to an emergency department by his mother because she could not get him to wake up completely after his nap. He is semiconscious and has a low-grade fever. The physician suspects lead poisoning. The nurse should expect that the child will be treated with

Calcium disodium edetate (EDTA). Answer2.137=Syrup of ipecac. Answer3.137= Activated charcoal.

Rational.137=Calcium disodium edetate (EDTA) is a chelating agent which promotes the excretion of lead from the body. It is given IV or deep IM for children who cannot tolerate IV.

Answer4.137=Erythromycin.

Question.138= The nurse should know that if a patient with heart failure develops a ventricular arrhythmia, physician`s orders will be to administer Rational.138= Lidocaine hydrochloride, USP, is the medication of choice because it depresses ventricular irritability. Inderal is contraindicated, as it is a beta-adrenergic blocking agent--it also depresses cardiac function. For a patient who already has a compromised cardiac status, this could be fatal. Morphine sulfate, USP, reduces anxiety, but will not prevent arrhythmias. Digoxin is used to strengthen ventricular contraction.

Morphine sulfate, USP. Answer2.138=Lidocaine hydrochloride, USP. Answer3.138= Digoxin (Lanoxin). Answer4.138=Propranolol hydrochloride (Inderal).

Question.142= A 26-year-old primigravida who is 27 weeks pregnant is admitted to the hospital with pre-eclampsia. The physician prescribes magnesium sulfate therapy. The nurse will place the patient in a private room because

A quiet, darkened room is important to reduce external stimuli.

#1Rational.142= An important aspect of the treatment for pre-eclampsia is absolute quiet, and only a private room will accomplish this objective.

Answer2.142= A rigid regimen is an important aspect of eclamptic care. Answer3.142= It will allow her husband to be with her. Answer4.142= She would be disturbed if she were placed in a room with another woman in active labor.

Question.143= An adult patient with moderate diabetes mellitus, Type I, has been admitted for a work-up for peptic ulcer disease. He is scheduled for an upper GI series at 8:00 AM. The most important nursing intervention related to his intermediate-acting insulin dose will be to

Administer his usual dose of intermediate-acting insulin before the test.

Answer2.143= Explain that he will not receive his insulin until after the test is completed. Answer3.143= Administer extra insulin to withstand the stress of the test. Answer4.143= Administer regular insulin before the test and explain that his breakfast will be served after the

Rational.143=The patient is taking intermediate-acting insulin, as he is a moderate diabetic. He will most likely receive insulin and then be fed breakfast when he returns after the test. Therefore, (1) is the most correct. Extra insulin would not be administered.

Question.144= When completing an assessment on a 9-year-old child, the nurse knows that a primary developmental task of a 9 year old, according to Erikson, is Rational.144=From age 6 to 12, Erikson has theorized that the main developmental task is industry versus inferiority. Trust (1) is resolved in the infant stage of development initiative (2) versus guilt is the task of preschoolers, age 3 to 6, and identity (4) is the task to be resolved at puberty and adolescence.

Trust. Answer2.144=Initiative. Answer3.144= Industry. Answer4.144=Identity.

Question.145= The nurse has a full work load and must reassign some of her patients to the nursing assistant. The most appropriate patient to reassign is a(n) Rational.145=The most appropriate patient would be the one with the CVA diagnosis. This patient would have been hospitalized for 2 days, so the initial assessment is completed. This condition does not demand immediate assessment or intervention, as does the colostomy patient (assessing for hemorrhage, vital signs, etc.), the oncology patient (determining the effectiveness of the pain protocol), and the new patient with suspected pancreatitis (who needs a complete assessment).

Patient just returning from the recovery room following colostomy surgery.

Answer2.145= CVA patient who has been hospitalized for 2 days.

Answer3.145= Newly admitted patient with suspected pancreatitis. Answer4.145= Oncology patient who is in severe pain controlled by epidural analgesia.

Question.146= When assessing a 2-month-old infant`s reflexes, the nurse should understand that at this age the infant should exhibit 4 Rational.146=The palmar grasp, elicited by placing a finger in the infant`s palm, disappears at about 4 months of age, so it would still be present while assessing the 2 month old. A positive Babinski`s reflex is present until 12 to 18 months of age. In adults, a positive Babinski`s reflex is indicative of disruption of the pyramidal tract. The tonic neck reflex disappears at 4 months, as does the rooting reflex while the infant is awake.

negative Babinski`s reflex. Answer2.146=An absent tonic neck reflex. Answer3.146= An absent rooting reflex. Answer4.146=A palmar grasp.

Question.147= A 57-year-old patient is admitted to the nursing unit for an endocrine work-up. The tentative diagnosis is Cushing`s syndrome, etiology unknown. The patient says to the nurse, "I look so awful! I just can`t stand having my friends see me like this!" Which of the following responses would be best?

I know how you feel, but they will get used to it very soon." Rational.147=The signs and symptoms of Cushing`s syndrome are all caused by an excessive steroid production. When this condition is normalized, these signs will disappear. This is a factual, therefore reassuring, statement.

Answer2.147= "The way you look is a result of your disease. When it is controlled, you will return to your normal appearance."

Answer3.147= "It is best not to worry because the stress will make your symptoms worse."

Answer4.147= "That is a problem, and you should ask your physician what can be done about it."

A nurse is assigned to the oncology unit and understands that serum laboratory changes are associated with radiation therapy. The nurse will focus the patient assessment on

Answer1.148= Checking the need for subcutaneous injections of vitamin K. Rational.148= Patients undergoing radiotherapy may have a decreased white blood cell count and should be observed closely for infections.

Answer2.148= Checking lab tests for low hemoglobin.

Answer3.148= Monitoring changes in electrolytes.

Answer4.148= Observing for signs of infection.

Question.149= After 1 month of hospitalization, a schizophrenic patient states that voices are telling him he will die tonight, and he asks the nurse if this is true. This question should indicate to the nurse that the patient 3 Rational.149=The patient exhibits the awareness to question his symptom, thus he is improving. The nurse can reinforce this strength, then stay with him and help him discuss his fears. (4) is incorrect as his prognosis cannot be calculated. (1) and (2) are not substantiated by the data.

Is not improving.

Answer2.149= Will begin to enter the manic phase of his illness.

Answer3.149= Is improving but needs reassurance from the nurse. Answer4.149= Has a poor prognosis.

Question.150= An adult patient with a gunshot wound to the chest is brought to the emergency department. A left-sided pneumothorax occurred, and the physician inserted two chest tubes, one into the right anterior lobe and one into the right lower lobe. The water-seal drainage system is connected to a walled-in suction. To assist in chest drainage, the nurse should place the patient in Rational.150=Positioning a patient on the left side will assist in drainage and semi-Fowler`s will assist in breathing. The patient can usually be turned to both sides and the back.

igh Fowler`s position. Answer2.150=Semi-Fowler`s position, on his left side. Answer3.150= Supine position. Answer4.150=Low-Fowler`s position, on his right side.

Question.151= A patient turns abruptly in bed and dislodges his chest tube. The nurse`s first intervention is to mmediately get a sterile dressing and apply over the opening. Answer2.151= Obtain a sterile towel and place it over the opening. Answer3.151= Place a sterile Vaseline gauze over the opening. Answer4.151= Seal off the opening with the nurse`s hand. Rational.151=The nurse must seal off the opening immediately therefore, a hand is the closest thing available and should be placed over the opening. If air is sucked back into the chest cavity, the lung could collapse again.

Question.152= An 11-year-old patient with the diagnosis of acute rheumatic fever will have a care plan that includes the most important nursing measure of Sufficient vitamins for tissue repair. Answer2.152=Adherence to bedrest regimen. Answer3.152= Breathing exercises to increase oxygen exchange. Answer4.152=Isolation for prevention of infection. C Rational.152= The most important nursing care would be to maintain bedrest so that the heart is not overtaxed and the body has sufficient rest time to repair the damage caused by the systemic inflammatory disease. Warm soaks are given to reduce joint pain, and a good diet is necessary for repair of any organ, but in this instance they are not the most important components of nursing care. Normally, isolation is not required.

Question.171= A newborn passes meconium after the first day and the mother is concerned. The nurse can explain to the new mother that infant meconium is normally

Rational.171= Meconium is normally passed during the first 24 hours for all babies. It is not a transitional stool (3) which is partly meconium and partly milk, usually passed from the second to the fifth day.

Question.172= If there is a physician`s order to irrigate the bladder, which one of the following nursing measures will ensure patency?

Answer1.172= Irrigate with 20 mL of normal saline to establish patency.

Question.174= The most important nursing intervention for patients on IPPB therapy is to

Rational.174= The detection of alterations in vital signs could be an indication of nebulizer medication side-effects so this intervention is the most important.

Question.182= A 29 year old is in the labor room and has been there for 6 hours. It is the change of shift and, when the nurse is receiving report, a nurse`s aide rushes in and says that the patient is delivering. As the nurse enters the room, the baby`s head is crowning. The first nursing action is to

Rational.182= At this stage, the most important action is to support the baby`s head. The nurse would not try to hold back or stop the delivery, but allow it to occur naturally. If possible, send someone else to notify the physician. Instruct the mother to pant, not push, to avoid rapid delivery.

Question.183= Following an emergency delivery, the nurse is alone with the mother when the baby is finally born. After suctioning the baby`s mouth with a bulb syringe, the nurse realizes the newborn has not yet begun to breathe. The first nursing action is to #2 Rational.183= The first action is to hold the baby so that the head is in a dependent position. This will allow mucus and fluid to drain out. Then, if the baby does not breathe on his own, action (3) should be done. The nurse would NOT slap the baby on its back.

nitiate infant CPR.

Answer2.183= Hold the baby upside down by his heels so that mucus will drain out. Answer3.183= Gently rub the baby`s back or soles of his feet. Answer4.183= Sharply slap the baby`s back to stimulate breathing.

Question.184= A 21-year-old schizophrenic woman does not relate to staff or other patients on the psychiatric unit. She isolates herself in a part of the day room. The staff nurse wishes to establish communication with her. The best plan would be to approach the patient

Introduce yourself and ask if you can stay with her.

Answer2.184= Nonverbally because she is nonverbal. Answer3.184= Introduce yourself and tell her that you are going to sit with her a few moments. Answer4.184= Use touch to get her attention, and ask her if she would like to work with you.

3Rational.184= The most therapeutic response would be to introduce yourself and sit quietly a few moments making NO demands on her by asking questions. A nonverbal approach would be threatening. The nurse would not use touch with a schizophrenic (potentially paranoid) patient.

Question.185= A 25-year-old female with AB negative blood is para 0, gravida 1. She has aborted after 10 weeks and is going to be kept in the hospital 1 night for observation. Considering her Rh blood type, what action would the nurse expect to be performed

85= The patient would be given an injection of RhoGAM within 72 hours Rational.185=RhoGAM must be administered within 72 hours to an Rh-negative female who has not been sensitized. Because this is her first pregnancy, this patient should have the RhoGAM within 72 hours.

Answer3.186= High susceptibility to infection. Question.186= A 50-year-old patient has received the diagnosis of agranulocytosis. The nurse understands that one of the most serious consequences of agranulocytosis is Rational.186= This blood disorder is characterized by profound neutropenia and can lead to severe infection which could be fatal. The symptoms would be chills, fever, sore throat, and lethargy.

ou`ve done some good work here. I hope you`re able to follow through on it." Question.187= A psychiatric patient rapidly improves and is scheduled to be discharged tomorrow. Which of the following responses demonstrates that the nurse has a good understanding of termination of a relationship? Rational.187= Termination is an on-going process, so encouraging the patient to follow through with what was learned is a good response. The nurse would NOT encourage the patient to "stop by" because the relationship is terminated.

Answer3.193= Telling the patient that a lag of 7 to 10 days can be expected between the initiation of lithium therapy and the control of manic symptoms. Question.193= A patient is receiving lithium carbonate for manic behavior. Administration of this medication should be guided by Rational.193= There will be 7 to 10 days before the patient will experience a decrease in the manic symptoms. A therapeutic dose is 300 mg TID regular blood studies must be continued throughout drug therapy muscle weakness is an expected side effect and does not indicate toxicity.

Question.194= A 24-year-old male patient with no feeling or sensation in his lower extremities is in spinal shock. The nurse will be able to recognize that his condition is improving when

Answer3.194= Hyperreflexia occurs. Rational.194= Reflex activity begins to return below the level of injury because of automatic activity inherent in nervous tissue.

Answer1.195= High calorie, high protein, low fat. Question.195= The type of diet that would be indicated for a child with cystic fibrosis is Rational.195=High calorie and protein intake will support growth and weight gain. A low fat diet is encouraged because the fat is poorly digested. Pancreatic enzymes are replaced and given with meals to assist with the digestion and absorption of fat.

1cm above the level of the ischial spines. Question.198= After a vaginal examination, the patient`s obstetrician tells her that she is at -1 station. When she asks the nurse what this means, the nurse tells her that station -1 means the presenting part of the fetus is Rational.198= Station is the degree to which the presenting part has descended into the pelvis-the relationship between the presenting part and the ischial spine. The fetus moves from above to below the level of the ischial spines.

Left or right occiput anterior. Question.199= In assessing the baby`s position for delivery, the nurse knows that the most favorable position for delivery of an infant is a Rational.199= Left or right occiput anterior (LOA and ROA) are the most favorable positions for delivery. Breech, where the buttocks or lower extremities are the presenting part, is a difficult delivery position and a transverse lie necessitates delivery by C-section.

Answer4.204= Ankle and calf circumference and leg length after the patient has been lying down. Question.204= When measuring a patient for elastic hose, the nurse will measure the Rational.204= The leg length and ankle and calf circumferences should be measured after the patient has been lying down. This causes the peripheral edema to be minimal and ensures that the hose fit snugly to offer maximum support.

Fluid retention, mood changes, anxiety. Question.207= A patient has physician`s orders to take chemotherapeutic hormonal agents. The nurse would prepare the patient to anticipate which of the following side effects Rational.207= Fluid retention, mood changes, and anxiety are common side effects. They are a direct result of using androgens or estrogens as chemotherapeutic agents.

Question.209= The physician has just completed a liver biopsy. Immediately following the procedure, the nurse will position the patient

Rational.209=Placing the patient on his right side will allow pressure to be placed on the puncture site, thus promoting hemostasis and preventing hemorrhage. The other positions will not be effective in achieving these goals.

Question.210= The nurse collects the following information when taking a nursing history from a postmenopausal patient. Analyzing the data reveals a risk factor for the development of osteoporosis when the patient

as been on prednisone (Deltasone) for 3 months. Rational.210= Glucocorticoids, such as prednisone, promote protein catabolism and are a known risk factor for the development of osteoporosis.

Question.212= The nurse is at the bedside when a 9 year old has a seizure shortly after admission. The first action during the seizure is to

Answer3.212= Protect the child from injury by removing objects from the bed.

Question.214= While explaining the side effects of oral contraceptives to a teenage patient, the nurse will tell her that

Answer4.214= Side effects may appear but these usually disappear within the first to third cycle.

Question.216= A 65-year-old male patient with Parkinson`s disease is being treated with L-Dopa. The nurse will know he understands the teaching principles when he says that he avoids foods rich in

Rational.216= Foods rich in B6 block the desired effects of L-Dopa therefore, they need to be omitted from the diet. Examples of foods to be avoided include meat, especially organ meats, whole-grain cereals, peanuts, and wheat germ.

Question.220= A 20-year-old patient is admitted to a hospital with a diagnosis of acute schizophrenia. He is becoming more withdrawn and suspicious of other patients, and he constantly tries to argue with the nursing staff that several of the patients are "out to get him." The best nursing approach to this behavior is to

Answer3.220= Avoid disagreeing with him and get him involved with an activity.

Question.222= A 50-year-old female patient is scheduled for a cholecystectomy. Following surgery, she has a T-tube in place and is returned to the surgical unit. Which position will ensure optimal functioning

Rational.222=Initially a low-Fowler`s position, then a semi-Fowler`s position is encouraged, but not high-Fowler`s. The objective is to facilitate drainage, as well as allow a position of comfort for the patient.

Question.223= A patient is admitted with a tentative diagnosis of metabolic acidosis. Assessing for the clinical manifestations most representative of this condition would include

headache, diarrhea, =Metabolic acidosis results from conditions such as diabetes mellitus and diarrhea

Question.230= A female patient has orders to take lithium regularly after being discharged from the hospital. Her serum level has been regulated at 1.4 mEq/L and she seems to be doing well. Which of the following is the most important discharge information to impart to the patient and her family

Answer2.230= Drowsiness, tremors and slurred speech are early indications of lithium toxicity.

Rational.230= Most important is that the patient and family be aware of lithium toxicity these symptoms are indicators of this condition. Answer

Answer1.236= Administering pilocarpine eye drops. Question.236= The preoperative nursing care plan for a patient scheduled for an iridectomy will include Rational.236=Pilocarpine acts directly on the myoneural junction it constricts the pupils and forces the iris away from the trabecular, allowing fluid to escape. Cycloplegic drops are given preoperatively with a cataract to paralyze the ciliary muscle, and postoperatively to relax the ciliary muscle with an iridectomy.

Answer2.237=Turn the patient onto her left side.

Question.237= A patient is lying comfortably on her back 30 minutes after receiving an epidural injection. The nurse checks fetal heart tones and the rate is 100 per minute. The first nursing action is to

Rational.237= This form of regional anesthesia may cause transitory fetal bradycardia. The first nursing action is to turn the patient onto her left side, shifting the weight of the fetus off the inferior vena cava. If the condition does not change after repositioning, then administer oxygen, as ordered. The next two actions would be to check the fetal heart rate and the mother`s blood pressure.

Question.239= A 2-month-old female infant develops an ear infection. While administering ear drops, the appropriate technique is to place the baby

Rational.239=The appropriate technique is to pull the ear auricle down and back so that the ear drops fall into the ear canal. The infant may be on her side or back with her head turned for this procedure. This question asks for technique, so answer (1) is more appropriate. If the nurse is administering ear drops to an adult, the procedure is to pull the ear auricle up and back.

Question.241= One of the primary goals of treatment for a patient with acute glomerulonephritis would be to encourage

Rational.241= Bedrest during the acute phase will protect the kidneys from added stress. Activity may increase urinary abnormalities as well as facilitate diuresis. When the kidneys recover, fluid balance and mild hypertension will be alleviated. The diet should be low, not high, protein to protect the kidneys from processing protein waste products.

Question.247= During a typical emergency delivery, which of the following principles best explains why the nurse will not cut the cord

Rational.247= The rationale for not cutting the cord following an emergency delivery is that lack of sterility could lead to infection. Hemorrhage might also occur, but not from the placenta. Answers (1) and (4) are not relevant principles to explain why the cord should not be cut.

Question.250= A nursing goal to be included in the care plan for a patient with chest tubes in place will be to

Answer2.250= Keep the bottles below bed level to prevent backflow.

Question.251= A patient has recently been diagnosed with tuberculosis. The wife of the patient is concerned that she will contract the disease. The nurse can teach the patient`s wife that the most important preventive method would be

Rational.251= Those people infected with the tubercle bacillus without the disease and those at high risk for developing the disease should receive a drug regimen of isoniazid and vitamin B6 as prophylaxis. Measures to prevent the spread of the disease should also be implemented but, depending on the relationship, not living with her husband or keeping him in isolation are not the usual methods.

Question.252= A 2-year-old patient has a fracture of his right femur. Observing whether Bryant`s traction is properly assembled, the nurse will expect to see the

Rational.252= Bryant`s traction is a form of skin traction and, therefore, does not require a pin insertion. Moleskin is frequently used as the stabilizing material for traction application. The weights must hang freely from the crib to maintain alignment and decrease the fracture.

Answer4.252= Weights attached to skin traction and hung freely from the crib.

Question.254= A patient with the diagnosis of schizophrenia tells the nurse that there is nothing wrong with her-it`s the fault of her family that she is in the hospital and that they are out to get her. This defense mechanism is called

Rational.254= The patient is using projection when she blames her family, saying they are out to get her. Projection is placing the blame for one`s difficulties on others it is an indicator of paranoia.

Answer2.255= Having difficulty telling the difference between her own wishes and fears and what is real. Question.255= The nurse observes a schizophrenic female patient sitting, staring into space, occasionally saying something like "Is the world coming to an end?" The nurse`s understanding of a comment like this is that the patient is

Rational.255= A central problem in schizophrenia is difficulty in reality-testing, where the patient often cannot tell the difference between what is real and what is not. This statement is not necessarily paranoid ideation, so answer (

Question.259= For a diagnosis of viral pneumonia in a 6 month old, the nursing diagnosis is fluid deficit. The best method of ensuring a proper fluid balance during the acute stage of the infant`s illness is to administer

Rational.259= One of the dangers of high temperatures in young children is dehydration. It is critical to restore fluid balance, so IV infusion is the preferred treatment. Dextrose and water is a hypotonic solution that causes cells to expand or increase in size. It is the fluid replacement of choice for diarrhea and dehydration.

Question.261= The nursing staff of a psychiatric unit planned an all-day outing. Many of the patients were on large doses of phenothiazines. A nursing action important to implement prior to the outing is to

Rational.261=Patients taking phenothiazines are sensitive to the sun (photophobia). The correct answer is to have these patients put on sunscreen and wear hats when they are in the sun. Answer

Question.263= A 23-year old female patient, following an automobile accident, was placed in skeletal traction for a fractured femur. In the assessment of this patient, one of the nurse`s first concerns will be to assess for

Rational.263= Proper alignment is critical to maintain skeletal traction in counterbalance or correct pull. It is also important to assess for pressure points or skin excoriation, but these observations would be continual as long as the traction is in place.

Question.268= Dilantin is ordered to control a 10 year old`s seizures. While teaching her mother about the medication, one of the side effects to emphasize is

Rational.268= The primary side effects of the drug, Dilantin, are bleeding gums and gum hypertrophy, rash and GI symptoms. Answer (3) is the preferable choice because it will involve teaching the patient to use a soft toothbrush, brush frequently, and eat a diet high in vitamins and minerals to protect the gums.

Question.274= During the postoperative period following abdominal surgery, the patient will require airway suctioning. The primary rationale for keeping the suction catheter in the airway for only 10 to 15 seconds is to prevent

Rational.274= Suctioning longer than this length of time will remove excess oxygen and this may lead to hypoxemia. Another technique of preventing hypoxemia is to administer oxygen 1 to 2 minutes before suctioning.

Question.276= There is an order to perform the crede procedure on a male patient every 6 hours. "Crede" is a term used for a

Rational.276= Crede is a French term that refers to the manual expression of urine, made necessary because of a hypotonic bladder. This method will help initiate bladder retraining.

Question.277= A patient has a chest tube connected to two-bottle water-seal suction. The purpose of water in the second bottle is to

Rational.277= The end of the drainage tube is kept under water this water seals the tube so air cannot enter and be drawn back into the pleural space.

Question.279= Immediate postoperative nursing interventions for an above-the-knee amputation will include

Rational.279= The possibility exists that the patient could hemorrhage from the stump. Therefore, it is safe nursing care to have a tourniquet at the bedside.

Question.280= The nurse will help a child with acute glomerulonephritis follow a diet regimen of

Rational.280= A diet with restricted potassium and protein is necessary for all children who demonstrate some degree of renal failure.

Question.282= A child is admitted to the hospital with marked symptoms of nephrosis. The following information should be included in the admission nurse`s notes. Which information is most pertinent in terms of the child`s condition

Rational.282= It is important to note the degree and extent of generalized edema that occurs with nephrosis. The condition is characterized by severe proteinuria which results in hypoalbuminuria leading to the shift of fluid from the intravascular to the extracellular compartment.

Question.288= The instructions to a patient just learning to use a four-point crutch-walking gait would be to move the

Answer3.288= Right crutch, then the left foot.

Question.291= A 1-month-old female infant has been readmitted to the hospital for a cleft lip repair. In planning for her preoperative care, the nurse will feed her in a/an

Rational.291= An upright or sitting position is important when using a soft, large-holed, cross-cut or special nipple. The nipple should be placed on the opposite side from the cleft.

Question.294= An adult male patient is admitted to the hospital for a cardiac work-up. During the initial physical assessment, the nurse listens for bronchovesicular breath sounds. These sounds can normally be heard over the

Rational.294= These hollow, muffled breath sounds can be heard over the bronchial area below the clavicle.

Question.296= A male patient has a tracheostomy and requires suctioning. The nurse knows that signs of hypoxia may occur during this procedure. A nursing action to prevent hypoxia is to

Answer3.296= Hyperinflate the lungs with 100% oxygen prior to and following suctioning. Rational.296= Hyperinflation of lungs with oxygen before and after suctioning prevents potential cardiac complications due to a sudden drop in blood oxygen levels.

Question.299= A 24-year-old married female suspects that she is pregnant. She states that her last menstrual period started on August 9, 2005. She asks, "If I am pregnant, when will my baby be due?" According to Nagele`s rule, the due date would be

Rational.299=Nagele`s rule is to count back 3 months from the first day of the last menstrual period and add 7 days. Answer1.299= May 16,2006.

Question.301= A 1 1/2-year-old is brought to the hospital with a diagnosis of pneumonia. His temperature is 38.8C (102F), respiratory rate is 40 and he appears lethargic. A tentative diagnosis of pneumonia is made. Evaluating his condition, an indication that he is improving is a respiratory rate of

Rational.301=A normal respiratory rate for a 1 1/2 year old is 30 breaths/min. This rate would indicate that the child`s respiratory condition is improving. The usual respiratory rate for a 14 year old is 18 breaths/min. A respiratory rate of 20 is usual for a 10 year old. A respiratory rate of 50 is abnormally high.

Question.302= A new mother-to-be asks why her doctor told her to discontinue taking all unnecessary drugs during pregnancy. Based on the understanding of the relationship between drugs and the fetus, the nurse`s reply would be

Rational.302= All drugs may be expected to cross the placental barrier and are especially damaging during the first 8 weeks, when fetal organogenesis is taking place. Drugs taken later in pregnancy may also affect the fetus, but their effects may not be known for years. Even drugs taken during labor may have a depressive effect upon the CNS of the fetus and may take several days to wear off. Nicotine from smoking may cause low birth weight infants as well as congenital defects.

Question.303= Adequate nutrition is essential during early pregnancy for optimum fetal development. The nurse recommends a daily diet that would include

Rational.303= The diet must include at least one fruit or vegetable high in vitamin C, and should include a total of four fruits and vegetables. Pregnancy requires the addition of 300 calories a day over regular caloric intake, and 1500 calories a day would be inadequate. The recommended calories for someone aged 28 are 2300 a day. New research indicates that sodium is essential, so a low sodium diet is not recommended.

Question.306= A cerebral arteriogram is performed on the patient. When he returns from the operating room, the nurse observes that he may be having a reaction to the dye. The sign or symptom that suggests this complication is

Rational.306= Numbness of the extremities is a symptom of delayed reaction to the dye. Respiratory distress is a frequent early sign of anaphylactic shock. The release of histamine causes major vascular and bronchial symptoms resulting in anaphylaxis.

Question.307= The patient goal for fluid intake following abdominal surgery for an inguinal hernia would be

Rational.307= 2000 to 3000 mL/day would be fluid maintenance postsurgery. The body will require additional fluids over the minimum due to fluid loss and the recovery process after surgery.

Question.309= A 26-year-old primigravida is mildly pre-eclamptic and will be followed on an outpatient basis. Which of the following signs or symptoms should the nurse expect to observe if the eclampsia is becoming more severe

Rational.309=Edema, proteinuria and hypertension are the three cardinal signs of pre-eclampsia. Normal urine output or oliguria occurs rather than polyuria.

Question.311= The RN has discussed the baby`s newborn status with the mother and asks the practical nurse to reinforce it. She explains to the mother that the insulin level in newborns of diabetic mothers

Rational.311=Insulin levels are increased in these infants because the mother`s glucose readily crosses the placenta and stimulates the fetal pancreas to secrete increased levels of insulin. The fetal insulin does not cross the placenta.

Question.315= A patient is being prepared for an oral cholecystogram. Before the dinner meal the practical nurse instructs the nursing assistant to

Rational.315= Diarrhea is a very common response to the dye tablets. A dinner of tea and toast is usually given to the patient. Each dye tablet is given 5 minutes apart, usually with 1 glass of water following each tablet. The number of tablets prescribed will vary, because it is based on the weight of the patient.

Question.316= A 7-year-old male complains of pain and limited movement in his left hip. The physician suspects Legg-Perthes disease. At this stage of the disease, the major goal of treatment is aimed at

Rational.316= Legg-Perthes disease affects the femoral epiphysis in which aseptic necrosis occurs. Pressure on the necrotic femur can cause permanent damage.

Answer4.316=Preventing pressure on the head of the femur.

Answer3.318= "I understand you are concerned. The staff will see that you are safe." Question.318= A 34-year-old male patient with a diagnosis of schizophrenia, paranoid type, barges into the dayroom yelling, "The President is on my side. If you bother me, I`ll send him after you!" The nurse could most effectively respond by saying Rational.318= The patient`s grandiose attacking statements probably reflect his feelings of fear and his anger at being afraid. Reassurance that he will be safe is important. His fear should be respected but not necessarily confronted since this might increase his anxiety. Confrontation would probably escalate his aggressiveness and add to his defensiveness. In his present state, a probing question would be threatening and inappropriate, although it might be useful later.

Question.319= A male patient was admitted with a diagnosis of subdural hematoma and transferred to the ICU in a semicomatose state. If the patient goes into a coma, the nurse would be likely to observe

Rational.319= An indication of a comatose state is increased blood pressure and slowing respirations. Aphasia is also a result of increased pressure however, due to a decreased level of consciousness which is sometimes present, this is not always an accurate indicator.

Question.320= A male patient is taken to surgery for evacuation of a subdural hematoma. Immediately after the evacuation, the priority nursing assessment is to observe

Rational.320= All of the nursing interventions listed would be carried out for the patient however, the most important one is to prevent cerebral hypoxia (which contributes to cerebral edema) by maintaining a patent airway. The acid-base imbalance and hypoxia are often mistaken for signs of increased intracranial pressure, leading to unnecessary surgical intervention. A patent airway will establish adequate oxygenation and prevent carbon dioxide build-up.

Question.323= A female patient has sustained burns of her right arm, right chest, face, and neck. She has just been admitted to the burn unit. Her weight on admission is 50 kg. Using the rule of nines, the estimate of the extent of her burns is

Rational.323=The extent of the burns is 27 percent, calculated by adding the head = 9 percent (face and neck each equal 4 1/2 percent), arm = 9 percent, and chest = 9 percent.

Question.324= To promote adequate nutrition, a burn patient`s diet after the first week of hospitalization should include

Rational.324= A diet high in carbohydrates is essential to allow the protein to be spared for tissue regeneration. High protein is also needed for tissue repair.

Question.325= Completing a general assessment on a 6 month year old, the nurse knows that this age infant is able to

Answer1.325= Sit for a short time, look toward sounds, and begin babbling.

Question.327= Each newborn receives an Apgar score shortly after birth. The nurse understands that the purpose of Apgar scoring is to

Rational.327=The purpose of Apgar scoring is to determine the viability of the infant. Apgar scoring is the evaluation of five vital signs: heart rate, respiratory rate, muscle tone, reflex irritability, and color. Scores of 0, 1 or 2 are given to each vital sign for a total of 10. A score of 7 to 10 is considered vigorous.

Question.333= The nurse has completed a postoperative assessment for hypoparathyroidism following a thyroidectomy. If the symptoms of this condition are present, the nurse would check with the RN and expect to administer

Rational.333=Signs of hypoparathyroidism following a thyroidectomy are evident in an acute attack of tetany. The drug of choice is calcium gluconate to counter the low calcium level.

Question.334= A patient is admitted to the hospital with an obstruction just proximal to the old ileostomy stoma. The nurse will monitor for a major complication which is most likely to be

Rational.334= Due to the extreme loss of fluids from the high colon interruption, fluid and electrolyte imbalance is the most common complication. The lower colon reabsorbs a major portion of the fluid, whereas the upper colon does not have this function. A great potassium loss also occurs, because it is found in large amounts in the upper colon.

Question.335= The nurse will know that a patient with an ileostomy understands dietary restrictions when she indicates that she does not include which one of the following foods in her diet

Rational.335= Corn may cause obstruction of the ileostomy and thus should be avoided.

Question.338= A 54-year-old male patient with a history of cirrhosis from alcohol abuse has been admitted for bleeding esophageal varices. While the patient is on bedrest, he is in a semi-Fowler`s position. The major objective for using this position is to

Rational.338= Any position that impedes respirations by the pressure of abdominal contents on the diaphragm should be avoided therefore, the best position for this patient is semi-Fowler`s position, which increases effectiveness of breathing.

Question.339= The nurse is assisting the physician to insert a Sengstaken-Blakemore tube. Prior to insertion the nursing action is to

Rational.339= In order to prevent the trauma of reinsertion, the balloons must be checked for leaks before insertion. This is done by inflating them and placing them in water to observe for bubbles.

Question.340= A child with the diagnosis of phenylketonuria (PKU) may not eat which of the following foods

Rational.340= The accumulation of phenylalanine, an amino acid breakdown of protein, is toxic to brain tissue. Therefore, any foods high in protein must be restricted. These include meat, fish, legumes, lima beans, milk products, etc.

Question.342= A female patient is being treated with intracavitary irradiation using a Cesium implant. The nursing care plan should include

Rational.342= A low-residue diet helps to prevent frequent bowel movements, a major side effect of radiation therapy. Absolute bedrest with logrolling from side-to-side is allowed. The head of the bed can be raised no more than a 45 degree angle.

Question.343= When a patient with advanced cirrhosis selects a snack, the choice that indicates understanding of the dietary requirements is

Rational.343= Carbohydrates are one of the mainstays of the cirrhotic patient`s diet. The liver can metabolize only very small amounts of protein, so usually only 40 to 50 grams of protein is allowed per day (normal diet is 60 to 80 grams per day). The banana is the only nonprotein choice.

Question.346= Which of the following assessments of the patient with an abdominal aortic aneurysm requires immediate reporting to the RN or physician

Rational.346=The increase in blood pressure could cause a rupture of the aneurysm, so this finding should be reported immediately. Palpation of a pulsating mass and a bruit are normal findings. Moderate anxiety over the surgery is also normal.

Question.347= A patient is having a prolonged labor and there is no progress past a dilation of 8 cm. Her physician decides to do a cesarean delivery. The new mother and her partner express their disappointment that they will not have a natural childbirth. The best response is to say

Most couples who have an unplanned cesarean birth feel cheated and disappointed.

Question.354= When using nasotracheal suction to clear a patient`s airway of excessive secretions, the nurse would

Rational.354= To prevent trauma to the mucous membranes lining the airway, suction should be applied intermittently and only while withdrawing the catheter. The catheter should be lubricated with a water-soluble substance to prevent lipoidal pneumonia. Suctioning attempts should be limited to 10 seconds or less to prevent hypoxia.

Question.356= When a patient has lung cancer that has been classified as Stage 1 , NO, MO, the nurse knows that this indicates that the patient has a tumor

Rational.356= "" means the tumor is less than 3 cm in diameter without invasion proximal to a lobar bronchus. "NO" means there is no demonstrable metastasis to regional lymph nodes and "MO" indicates no distant metastasis.

Question.357= The parents of a 4 month old noticed that many bruises were forming on their son`s knees, buttocks and thighs. The blood tests reveal that he has classic hemophilia. The nurse understands that hemophilia is

Rational.357= Hemophilia is a sex-linked recessive disorder. The asymptomatic mother transmits the disorder to the son on the X chromosome.

Question.362= A drug commonly administered to reduce the extrapyramidal side effects of phenothiazines used in the treatment of schizophrenia is

Rational.362=Cogentin or Artane are the antiparkinson drugs usually prescribed for reducing extrapyramidal effects caused by phenothiazines. Benadryl is also commonly used and has fewer side effects. The other answers are incorrect-Niamid and Ritalin are antidepressant drugs and Valium is classified as an antianxiety drug.

Question.363= A 2 year old has eaten half a bottle of his grandmother`s ferrous sulfate tablets. When the mother calls the clinic, the nurse will tell the mother to

Rational.363= Contact either the poison control center or the emergency department first and follow their instructions. In this case they will probably advise giving the child water to dilute the ferrous sulfate tablets and syrup of ipecac to induce vomiting. Because some poisons will be damaging if vomited, the center would not always advise the mother to give syrup of ipecac. Then the nurse would instruct the mother to bring the child to the hospital.

Question.364= A 58-year-old patient with a diagnosis of schizophrenia, chronic undifferentiated type is taking 400 mg of chlorpromazine (Thorazine) TID. The nurse notices on the morning rounds that he is drooling and flapping when he walks. The best nursing action would be to

Rational.364= The patient is experiencing side effects to phenothiazines, but these are not life-threatening and can be brought to the attention of the RN and physician within an appropriate amount of time. (4) is incorrect. Thorazine is effective at certain blood levels, and holding the drug would lower the blood level. If possible, it is preferable to check with the physician before holding a drug.

Question.365= A 65-year-old female patient has suffered a cerebral vascular accident (CVA)-left hemisphere lesion. The most appropriate method of communicating with her is to

Rational.365= For a left hemisphere lesion, the best method of communication is to pantomime what you are communicating while speaking in a normal tone of voice. Pantomime will confuse a person who has suffered a right hemisphere lesion. Before communicating, however, the nurse will assess the patient`s ability to understand speech. It is also important to give feedback as you communicate.

Question.367= A young patient, following a motorcycle accident, sustained spinal cord injury with respiratory function impairment. The cord segments involved with maintaining respiratory function are

Rational.367= Nervous control for the diaphragm (phrenic nerve) exists at the level of or of the spinal cord.

Question.368= The nurse is assigned a patient with the diagnosis of portal hypertension. The most important assessment with this condition is for the complication of

Rational.368= GI bleeding is a very common complication associated with portal hypertension. Obstruction of portal circulation leads to increased collateral circulation which can result in bleeding tendencies.

Question.369= An elderly patient with organic brain syndrome suffers from insomnia and asks the nurse for something to help him sleep. The physician will not order barbiturates for this patient because they cause

Rational.369= In organic brain disorder, barbiturates commonly cause delirium, confusion, and paradoxical excitement, thus they should not be ordered for patients with organic brain disorder.

Question.370= A diabetic patient takes 22 units of NPH insulin at 7:30 AM each day. Evaluating the effects of insulin, at which time during the day will the nurse assess her for signs of restlessness, memory lapses and headache

Rational.370= Intermediate insulin peaks from 8 to 12 hours after injection. 3:30 PM is the most appropriate time to assess for signs of insulin reaction.

Question.374= A patient is to receive 2500 mL of IV fluid over 24 hours. The IV tubing delivers 15 gtts per 1 mL. How many drops per minute should the patient receive

Rational.374= First, determine the number of mL to give the patient each hour by dividing the total volume by the total number of hours: 2500 mL divided by 24 hours yields a desired hourly amount of 104 mL. One method for determining the minute rate uses the formula of: Volume divided by Minutes multiplied by gtts/mL equals gtts/minute. 104 divided by 60 multiplied by 15 gtts/mL equals 26 gtts/ minute.

Question.378= The highest priority goal in the care of a newborn with tracheo-esophageal fistula (TEF) and esophageal atresia is to

Rational.378= The anatomical malformation in this anomaly threatens the newborn`s airway. Maintaining a patent airway is the highest priority in any situation in which the airway is threatened.

Question.385= A patient, age 60, is admitted to the hospital for a possible low intestinal obstruction. His preoperative work-up indicates vital signs of BP 100/70, P 88, R 18, and temperature of 96.4(F. Listening to bowel sounds, the nurse would expect to find

Answer3.385= Hyperactive, high-pitched sounds.

Question.386= A patient, age 68, has an external shunt placed in preparation for hemodialysis. The nursing care plan will include

Rational.386= Shunts should be inspected several times each day for presence of possible clotting. Dark spots will quickly be followed by separation of the sera and cells if clotting becomes complete. When dark spots appear, patients should be instructed to immediately seek treatment for declotting.

Question.387= A patient sustained a fracture 3 days ago. When the following blood studies are returned, they are all elevated. Which elevation would be considered a normal finding following a fracture

Rational.387=Bone is rich in alkaline phosphatase and blood levels normally increase following a fracture and during fracture healing. Elevation of the other blood studies should alert the nurse to the need for further assessment of the patient, because of the probable presence of an illness.

Question.388= A patient with a fractured right leg has been in Buck`s extension traction for a week. The nurse checking the patient finds that he is now unable to dorsiflex his right foot. The nurse will notify the charge nurse because there is a possibility that the

Rational.388= Dorsiflexion of the foot requires an intact peroneal nerve. Compression from any part of the traction apparatus along the lateral surface of the leg just below the knee can exert pressure on the peroneal nerve and impede its function.

Question.389= During visiting hours, a patient the nurse is caring for becomes very agitated and angry with his visitor. The most effective nursing approach to this patient is to

Rational.389= This approach would help decrease the patient`s anxiety and assist him in gaining insights.

Question.391= Using Leopold`s maneuvers, the nurse palpates the presence of a firm round prominence over the pubic symphysis, a smooth convex structure on the patient`s right side, irregular structures on the left side, and a soft roundness in the fundus. The nurse would conclude that the fetal position is

Rational.391= The head is down, the back is on the right side, legs on left, and fetus` buttocks in the fundus indicate the position of ROA.

Question.392= A patient who developed cerebral edema following a head injury is given mannitol (Osmitrol) intravenously. The outcome that most clearly indicates the drug has achieved its desired therapeutic effect is when the

Rational.392= Mannitol is given to reduce cerebral edema by promoting the movement of water from the tissues into the plasma followed by its excretion through the kidneys. The patient`s level of awareness is the most sensitive indicator of the effects of increased intracranial pressure. Improvement in the level of awareness, therefore, indicates a therapeutic response to the mannitol. The increased urinary output is simply a means through which the desired therapeutic effect is achieved. The absence of seizures does not indicate a therapeutic response to mannitol. Slowing of respirations may indicate increased cerebral edema.

Question.393= When a patient with a diagnosis of manic episode returns to the clinic to have lithium blood levels checked, her lithium level is only slightly higher than the previous week but she complains of blurred vision and ataxia. The first intervention is to

Rational.393=These are symptoms of toxicity and the nurse must withhold the next dose. The nurse would then notify the physician. The patient needs to maintain a normal fluid level to prevent toxicity, but this may not be the cause of her symptoms.

Question.395= A quadriplegic patient tells the nurse that he believes he is experiencing an episode of autonomic hyperreflexia (dysreflexia). The first nursing intervention is to

Rational.395= Blood pressure can become dangerously elevated during an episode of dysreflexia and can cause cerebral and retinal hemorrhages. Elevating the head will help prevent these complications and should be the nurse`s first action. Identifying the precipitant is useful in terminating the episode by removing the noxious stimulus which provoked the exaggerated autonomic response. A full bladder may precipitate dysreflexia and emptying the bladder would be appropriate if it was the precipitant. The blood pressure and pulse should be monitored throughout the episode of dysreflexia.

Question.396= A 3 month old with a diagnosis of chalasia is admitted to the hospital. He has had severe weight loss because of frequent vomiting. To minimize vomiting, the nurse would place the infant

Rational.396= The greater curvature of the stomach is toward the left side so the right side position affords less pressure. Elevation of the head would lessen the tendency to vomit.

interaction with othes Question.398= Which of the following behaviors would indicate improvement in coping for a patient Rational.398= If a patient can initiate interaction with another patient, it indicates he is not totally absorbed in himself, too depressed to initiate, or too absorbed in delusions or hallucinations to interact.

Question.400= An elderly patient with the diagnosis of COPD has been admitted to the hospital. In teaching the patient about his diet, which diet choice would indicate that a patient with COPD understands nutritional needs

Rational.400= Carbohydrate metabolism produces carbon dioxide which increases the blood levels of carbon dioxide. High protein prevents muscle wasting and helps preserve the strength of muscles, including the muscles of respiration. Calorie and energy needs are met by increasing the fat intake.

Question.410= A patient has an arteriovenous fistula as an access site for hemodialysis. Which assessment finding indicates that the fistula is patent

Rational.410= The flow of blood through a patent arteriovenous fistula produces turbulence manifested by a bruit audible when the fistula is auscultated.

Question.422= A male patient is becoming increasingly angry and verbally abusive. The first nursing intervention is to

Rational.422= The first intervention would be to calmly set limits to defuse the situation. The nurse would not place the patient in restraints until he is totally out of control nor would the nurse summon a male staff member initially -a sudden involvement of others could escalate the situation. After the initial intervention, the patient could be sent to his room.

Question.423= Teaching a new mother the principles of breast feeding, the nurse will know the mother understands how to care for her breasts when she says

Rational.423= The breasts need to be dried after feeding. No soap should be used because it removes natural oils. The baby should not be pulled off the breast because it will make the breast sore. The baby should not be nursed more than every 2 hours.

Question.425= The nurse in a well-baby clinic reminds a mother that at 4 months of age the infant should receive immunizations that include

Rational.425= At 4 months, most children receive DTaP, (diphtheria, tetanus and pertussis toxoid) inactivated oral polio vaccine hemophilus influenzae type b Pneumococcal vaccines and hepatitis B, if not given earlier. MMR (measles, mumps and rubella) is given at 15 months and 11 to 12 years. This is recommended by the Schedule Committee on Infectious Disease from the American Academy of Pediatrics.

Question.431= The nurse is assigned to a patient who is to have a Miller-Abbott tube inserted. Assisting the physician with this procedure, the nurse will position the patient in

Rational.431= High-Fowler`s with neck flexed is the position necessary for tube insertion, as it is inserted through the nose into the intestine.

Question.454= When performing naso-oral suctioning, the correct nursing action is to

Rational.454=Clean, rather than sterile, gloves can be used for naso-oral suctioning and the nurse would insert the catheter through the nares without applying suction. Suctioning is limited to 10 seconds to prevent excessive removal of oxygen. Suction on the catheter is released intermittently during the procedure and when being withdrawn.

Question.458= A patient makes a suicide attempt on the evening shift. The staff intervenes in time to prevent harm. In assessing the situation, the most important rationale for the staff to discuss regarding the incident is that

Rational.458= Even though all of the reasons are important and should not be ignored, the most important task for the staff is to assess the patient`s behavior and to identify cues that might indicate an impending suicide attempt.

Question.460= A manic patient is on lithium. The nurse will instruct her to report signs of lithium toxicity, which will not include

Rational.460= Diarrhea is associated with lithium toxicity. Some common symptoms of lithium toxicity are diarrhea, vomiting, nausea, drowsiness, muscle weakness, ataxia, blurred vision, confusion, dizziness, and restlessness.

Question.462= A baby is born with a myelomeningocele. The best position for this baby before surgery is

Rational.462= The prone Trendelenburg position is preferred to minimize tension on the fragile meningeal sac. In addition, feces and urine are less likely to come into contact with the defect.

Question.463= Assessing for the classic signs of basilar skull fracture, the nurse will observe for blood behind the tympanic membrane and

Rational.463= Cerebrospinal fluid leakage needs to be monitored closely. If leakage persists beyond a week, surgical repair may be necessary. Pain in the back of the head and retinal hemorrhage are not associated with a basilar skull fracture.

Question.465= An infant is hospitalized with a diagnosis of meningitis. A late sign of meningitis that the nurse should immediately report is

Rational.465= Because the infant`s fontanels are open, opisthotonos, a sign of increased intracranial pressure, appears late in the course of the illness. Hypothermia is a decompensator adaptation to many illnesses. "Sunset" sign of the eyes is found in uncorrected hydrocephalus. The fontanels continue to bulge as the meningitis remains unresolved.

Question.467= When an infant is one day old he has surgery for reduction of a myelomeningocele. Which nursing intervention is critical during the postoperative period?

Rational.467= Infants with myelomeningocele are prone to develop hydrocephalus, which is sometimes noted before surgery. When surgery is performed soon after birth, it is a critical nursing responsibility to observe for signs of increased intracranial pressure. Passive ROM, manual expression of urine, and observation of stool are important nursing activities, but the priority is assessment for increasing head circumference.

Question.468= If untreated, a child who has contracted diptheria may die because of

Rational.468= In severe cases persons with diptheria may develop a membranous lesion that covers the tonsils and can spread to cover the soft and hard palates and the posterior portion of the pharynx. The individual is unable to exchange air through the membrane and death results.

Question.472= A patient who is 6 weeks pregnant complains of a thick, white vaginal discharge. She denies having any itching of the vulva, blood-tinged mucus, or foul order with the discharge. The nursing intervention is to

Rational.472= Vaginal secretions during pregnancy are thick, white, and acidic (pH 3.5 to 6.0). The increased acidity is due to the presence of Lactobacillus acidophilus. An increase in leukocytes in the vaginal discharge results in leukorrhea. No culture or testing is needed.

Question.485= Elements of cause of action in negligence/malpractice include

Rational.485= Breach of duty, foreseeability, causation, injury, and damages are all constituents of malpractice and negligence. The level of damages is not related to negligence/malpractice (1). Negligence/malpractice injuries can be physical or psychological (2). The permanence of an injury doesn`t dictate negligence/malpractice (4).

Question.493= A female patient experiences swelling from her left shoulder to her fingertips within 24 hours after her left modified radical mastectomy with node dissection. The nurse explains to the patient that this results most directly from

Rational.493= When axillary nodes are removed and biopsied, the lymphatic system loses important sites of waste and fluid returns to the venous system. The resultant swelling can resolve without incident, or become severe and warrant intervention. Elevating her left (operative) arm on pillows decreases the swelling gravity assists the flow. Mastectomy is not usually offered to a woman who has widespread breast cancer. Also the nurse would be stating a medical diagnosis that could severely alarm the patient. With incisional drains in place postoperatively, it is unlikely that drainage from the arm would be impeded enough to allow blood and fluid collection under the incision. However, the drains should be routinely checked for patency.

Question.494= During preparation for a cardiac catheterization, instructions include information about contrast medium injections. The nurse will tell the patient to expect to experience

Rational.494= Nausea, warmth, facial flushing, and a salty taste in the mouth are all expected sensations when an iodine-based contrast medium is injected. Severe dyspnea could indicate a beginning anaphylactic reaction or perforation of the vessels, heart, and/or lungs by catheter. Generalized itching often heralds an allergic reaction to the dye. The patient who is cool and clammy may be experiencing hypovolemic shock or neurogenic shock or anxiety.

Question.507= The nurse observes 210 mL of bloody drainage in the chest tube collection chamber of a patient who underwent cardiac surgery two days earlier. Which of the following actions should the nurse take?

Rational.507= Bloody chest tube drainage is expected in the postoperative phase of cardiac surgery and no immediate intervention is indicated. Drainage in excess of 500 mL in the first 24 hours postoperative could indicate hemorrhage. Suction should never be increased without a physicians order (1). The chest drainage system should not be elevated above the level of the patient`s heart or the insertion site (2). Drainage in excess of 500 mL for the first 24 hours postoperative should be reported (3).

Question.508= The nurse is caring for a newly hospitalized patient identified with locked-in syndrome secondary to amyotrophic lateral sclerosis (ALS). Which of the following interventions would be appropriate to establish a form of communication?

Rational.508= Locked-in syndrome occurs in persons with a paralyzing illness and prevents them from communicating through writing, gestures, or speech. Because voluntary eye closure is spared, using eye blinks in order to communicate may prove useful. patients with locked-in-syndrome are unable to write. patients experience paralysis with locked-in-syndrome.

Question.511= When the nurse is caring for a patient with a new endotracheal tube, the most important action is to

Rational.511= Auscultate the patient`s lungs immediately after the endotracheal tube has been inserted and every hour thereafter. Oral hygiene, repositioning, and arterial blood gas results are not the highest priorities.

Question.512= Which of these assessment findings would indicate a tension pneumothorax?

Rational.512= A tension pneumothorax occurs when air is trapped within the pleural space. Mediastinal shift can develop as pressure increases, causing the layrnx and trachea to deviate away from the injury. Wheezing on the affected side is not a symptom of pneumothorax, since the lung is collapsed therefore, lung sounds would be distant or absent

Question.516= Which of the following ECG monitor rhythm-strips would indicate the body is compensating for heart failure?

Rational.516= Sinus tachycardia is a hallmark sign of heart failure and results from sympathetic stimulation. Atrial fibrillation may cause heart failure (1). Supraventricular tachycardia is an atrial dysrhythmia, not a common cause of heart failure

Question.517= Nursing care for a 6 month old who had a lumbar puncture performed to rule out bacterial meningitis should include

Rational.517= Rest, quiet environment, and fluids should be promoted postprocedure, and the child should be positioned flat or slightly elevated. Restraints can make the child more agitated and restless.

Question.518= A patient admitted for shortness of breath demonstrates arterial blood gas reults of pH 7.47; PCO2 33; and HCO3 26. A nurse should recognize these results as indicative of

Answer3.518= Uncompensated respiratory alkalosis.

Question.519= Interventions in the plan of care for a neonate who is admitted for sepsis should include

Rational.519= Treatment for neonatal sepsis includes: 1) obtaining blood, urine, and cerebral-spinal fluids for culture 2) initiating antibiotic therapy after cultures are obtained 3) providing respiratory support after blood gas determinations are drawn and 4) instituting measures for fever reduction which do not influence lab data.

Question.521= When evaluating the reliability of PKU blood test findings, the nurse should be confident with the validity of the results if the blood was acquired

Rational.521= The Guthrie test should not be done until 24 hours after feeding has been started with formula or breast milk. It takes time for the metabolites from feeding to accumulate, hence the need to wait 24 hours.

Question.523= When sending a specimen to the laboratory for ABG analysis, information that should be on the laboratory slip includes

Answer1.523= O2 running at 3 L/minute when the blood was drawn.

Question.528= A toddler is admitted to the pediatric unit with a diagnosis of near drowning. The nurse`s initial plan of care on admission should primarily focus on

Rational.528= With all near drownings, airway and breathing are the first priorities and can lead to subsequent cardiovascular, neurologic, and renal injuries if oxygenation is not maintained.

Question.529= A toddler is scheduled to receive the measles-mumps-rubella (MMR) and inactivated polio (IPV) vaccines. The nurse should not administer the MMR vaccine if the child

Rational.529= MMR is a live vaccine which should not be given if immunosuppressive therapy is currently being administered. A mild illness or HIV positivity is not a contraindication for MMR immunization.

Question.531= A patient with a bipolar disorder who is slightly manic is to start on lithium therapy. The patient and the nurse have discussed the drug`s actions. Which of these statements by the patient indicates he understood the discussion?

Rational.531= When lithium is used to treat mania, the patient may be aware of a slowing down of the thought processes. Diarrhea is a sign of lithium toxicity.

Question.532= When a patient taking lithium has a fever, it is extremely important for the nurse to be sure that the patient

Rational.532= Dehydration caused by excessive heat or fever can cause a buildup of lithium due to sodium loss, and toxicity may then occur. The priority of care is to prevent lithium toxicity due to sodium and fluid loss (1). Sunlight is not an issue in the person taking lithium (2). A nonstimulating environment does not address lithium toxicity (4).

Question.534= If a patient who is addicted to a benzodiazepine, such as diazepam (Valium), were to withdraw abruptly from the drug, the nurse should expect the patient to have

Rational.534= When a patient becomes addicted and develops a tolerance to Valium, the seizure threshold of the brain is raised. Abrupt cessation lowers the threshold and causes seizures.

Question.535= A patient who has been on long-term antipsychotic drug therapy shows early signs of tardive dyskinesia. The nurse`s action is based on the understanding that

Answer2.535= The drug may be discontinued. Rational.535= At this time, the only known treatment for tardive dyskinesia is to discontinue the antipsychotic drug causing the symptoms. Antiparkinsonian medications are not effective in treating tardive dyskinesia (1). Increasing the dosage may mask symptoms initially, but they will reemerge

Question.536= A patient who is taking an antipsychotic drug complains of beginning symptoms of dystonia. The nurse`s action is to

Answer2.536= Obtain an order for benztropine mesylate (Cogentin). Rational.536= Dystonia requires immediate treatment. It is easily treated with an antiparkinsonian drug, such as Cogentin.

Question.537= A female patient is started on chlorpromazine (Thorazine) therapy. The patient has received instructions regarding the drug`s side effects. Which of these comments by the patient indicates that she understood the instructions?

Rational.537= Photosensitivity may occur with Thorazine therapy staying out of the sun and wearing protective clothing are the best ways to deal with this problem. Thorazine does not decrease the desire for sex. It may cause impotence in men, or irregular menses (1). Weight gain is a common side effect of chlorpromazine (2). Sedation is a common side effect of chlorpromazine (4).

Question.547= A therapeutic nursing strategy to assist parents whose infant has just died is to

Rational.547= Alerting all personnel to a patient`s condition can help minimize pain related to thoughtless remarks. A symbol or a flower placed on the patient`s door alerts other staff to the special needs of the grieving. Parents often feel isolated and health care personnel tend to avoid these parents due to feelings of awkwardness (1). Health care personnel do not need to express sadness, only sensitivity to the parents` loss

Question.548= A patient is admitted to the labor and delivery unit in active labor. She gives birth to a 37-week stillborn baby. Immediately following the birth, the patient accuses the physician of "making a mistake" and "making a sick joke." She requests the nurse to bring her baby to her because she can hear him crying. The nurse`s best action is to

Rational.548= Actually seeing and holding the dead infant may help with the grieving process.

Ask her if she wants to hold the baby

Question.552= Which of the following statements would the nurse use when trying to get the demented patient, who is angrily arguing about where he is, to come to dinner?

Answer4.552= "You may not want to eat now, but it is dinnertime."

Question.554= When working with a depressed patient, the nurse should focus the initial nurse-patient contacts on

Rational.554= Establishing trust is essential to assist a person with depression.

Question.560= The rationale for using antipsychotic drugs with schizophrenia is to relieve

Rational.560= Antipsychotic medications work directly to stop hallucinations in a schizophrenic patient and are also helpful in eliminating or minimizing paranoid ideation.

Question.561= A 20-year-old patient on a psychiatric unit tells the nurse, "You are the only one who listens to me. The other nurses on this unit hate me." This person is attempting to create

Answer3.561= Splitting of the staff. Rational.561= The patient who expresses this is usually trying to manipulate the nurse in a manner that will put one staff member up against another. The patient is not attempting to create a fantasy world (1). The patient may be trying to create disorder and confusion, but it is accomplished through splittinging

When talking with a suicidal patient, it is important to establish

does he have a plan

Question.566= A 45-year-old woman is demanding of everyone and shows no concern for anyone but herself. Erickson would note that her stage of development is

Answer3.566= Generativity versus stagnation.

Question.567= Which of the following interventions is most appropriate when one is working with the aggressive, agitated person?

Rational.567= The highest priority of nursing diagnoses for the patient who is agitated and acting aggressively is high risk for injury to self and others. By creating a calm environment, the patient is provided with the opportunity to regain control. The patient is not violent at this time, so calling in extra staff isn`t appropriate and may exacerbate the situation (1). Setting limits is appropriate as it helps the patient regain control, but a more important priority is to remove persons at risk for injury

Question.568= When teaching a patient who is taking lithium to treat a bipolar disorder, the nurse would note that a way to prevent lithium toxicity is to

Rational.568= Lithium is excreted by the kidneys, and increased intake will ensure more efficient excretion. Exercise and hygiene aren`t methods to prevent lithium toxicity

Question.571= An African-American mother asks how to assess for jaundice in her new baby. Which of the following is the MOST appropriate answer for the nurse to offer?

Rational.571= Jaundice usually begins in the head and moves down the body and can be detected by blanching the neonate`s nose, palms of the hands, or soles of the feet. The sclera of the eyes will also be yellow tinted. Neonates of dark skin or olive complexion may appear to be jaundiced when in actuality they are not.

Question.577= Which of the following assessment findings on a 34 week gestation infant who is 4 days old would it be a priority to preport to the physician?

Answer2.577= Distended abdomen. Rational.577= Preterm infants are at risk for necrotizing enterocolitis (NEC), a condition with 50% mortality. Assessment of increasing abdominal distention or absent bowel sounds, vomiting, blood in the stools, and signs of infection are vital to report. Treatment includes antibiotics, discontinuation of oral feeding, administration of parenteral nutrition, and possibly surgery.

Question.579= Which of the following comments by a patient with pregnancy-induced hypertension should alert the nurse to potential problems?

Rational.579= Preeclampsia may progress to eclampsia, the severe phase of PIH. In severe cases, patients may experience frontal or occipital headache that is not relieved by analgesics.

Question.580= As part of the assessment of a patient suspected to have pregnancy-induced hypertension (PIH), the nurse will

Rational.580= Brisk deep tendon reflexes may suggest cerebral irritability secondary to decreased circulation and edema associated with preeclampsia.

Question.581= A patient with preeclampsia complains of epigastric pain. The nursing intervention is to

Rational.581= A sign of impending eclampsia is right upper quadrant or epigastric pain. Early detection and management of preeclampsia are associated with reducing risks and progression of this condition.

Question.584= During an infant tube feeding the best position for the infant is to

Rational.584= Parents should be encouraged to hold the infant during tube feedings. If regurgitation has been a problem, the infant may be positioned on his right side.

Question.586= A postpartal patient is complaining of shortness of breath and chest pain, and exhibiting tachypnea and tachycardia. The priority nursing intervention is to

Rational.586= Deep vein thrombasis (DVT) and pulmonary embolism are possible complications occurring in the postpartal period. Signs and symptoms of pulmonary embolism include sudden, sharp chest pain, syncope, tachycardia, rales, and tachypnea. Treatment depends upon rapid assessment and notification of the physician of the patient status.

Question.590= A newly married couple have decided to practice natural family planning. They ask the nurse about the Billings method. The nurse explains that the Billings method is performed by

Rational.590= The Billings method, also called the cervical mucus method or the ovulation method, depends on the characteristic changes in the cervical mucus at the time of ovulation.

Question.591= When teaching patients about improving their chances for fertilization, which of the following time periods should be highlighted postovulation?

Rational.591= The total critical time for fertilization is 24 to 48 hours, 12 to 24 hours before ovulation and 12 to 24 hours after ovulation.

Question.592= The nurse is teaching the expectant mother how the production of progesterone and estrogen prepare the uterus for implantation of the fertilized ovum. Which pituitary hormone would the nurse emphasize?

Rational.592= LH is released by the anterior pituitary, stimulating ovulation and the development of the corpus luteum, preparing the endometrium for implantation.

Question.594= Betamethasone is ordered and given to a patient diagnosed with preterm labor at 34 weeks. In evaluating the effects of this medication, the nurse would note that it is given to

Rational.594= Betamethasone may be given to the mother to hasten fetal maturity by stimulating development of lecithin when membranes are ruptured and premature labor cannot be arrested. The incidence and severity of respiratory distress syndrome has been found to be reduced if glucocorticoids (e.g., betamethasone) are administered to the mother at least 24 to 48 hours before birth."

Question.601= When performing a physical exam on a newborn infant, the nurse notes a circular swelling with bluish discoloration around the top of the baby`s head. This is recorded as

Rational.601= Caput succedaneum is a generalized, easily identifiable edematous area of the scalp.

Question.605= When instructing a pregnant patient about average weight gain, the nurse would reinforce that the ideal weight gain during pregnancy is

Answer4.605= 25 to 35 pounds.

Question.607= Assessing a newly admitted patient to the ER, an early sign of shock would be a

Rational.607= A rise in the pulse rate is one of the earliest signs of shock. A blood pressure of 110/70 and a respiratory rate of 24 are normal and do not reflect shock (2) (3). The temperature of 101degree F is not associated directly with a sign of shock but may indicate many other problems (4).

Question.611= When teaching a patient the proper use of a walker, the nurse would direct the patient to

Rational.611= The walker should be advanced 6 inches (15 cm) in front of the patient before the patient moves toward it. Picking the walker up and only using it periodically would negate the purpose of the walker (2). The patient should be taught to advance the weaker leg first (3). The arms should be slightly flexed at a 30-degree angle at the elbow. A 90-degree flexion is unsafe and makes the walker too high for the patient (4).

Question.613= Assuming a normal meal schedule, the patient who takes isophane insulin suspension (NPH) SC at 6 AM is most likely to demonstrate symptoms of hypoglycemia at

Rational.613= This intermediate-acting insulin has a peak action of 6 to 12 hours. If the patient is given this product at 6 AM, the window for a reaction would be highest between 12 PM and 6 PM. Seven AM is too short a time period for intermediate insulin (1). Nine AM would be more consistent with the peak effects of regular insulin (

Question.614= An order reads "Infuse 1000 mL D5W over 10 hours." The nurse knows that the drop factor is 15gtt/mL. You would infuse ________________________drops per minute.

Rational.614=The answer is 25 drops per minute. To determine the drip rate, first note that 1000 mL administered over 10 hours equals 100 mL per hour. 15 gtt/mL x 100 mL = 1500 gtt per hour. 1500 gtt/hour divided by 60 min = 25 gtt/min.

Question.616= A male patient is being treated for ruptured esophageal varices with a Sengstaken-Blakemore tube. His vital signs have been stable and the suction port is draining scant amounts of drainage. He suddenly becomes acutely dyspneic and oximetry reveals an O2 sat of 74%. The nurse`s immediate action is to

Rational.616= Sudden dyspnea and poor oxygen saturation may be signs that the esophageal balloon has slipped. If this is the case, the esophageal balloon should be cut immediately, as it is occluding the patient`s airway. Releasing the gastric balloon will not improve airway function in the person who is experiencing airway obstruction as a result of movement of the inflated esphageal balloon (2). Increasing suction or irrigating the gastric balloon will not improve the airway function in the person who is experiencing airway obstruction as a result of movement of the inflated esophageal balloon

Question.621= Following the application of skeletal traction, a patient complains of "the traction pulling too much." The nurse`s response is to

Rational.621= Continuous or intermittent traction is maintained as prescribed. The patient may realign herself, but the weights must remain in place. Never lift or remove the weights to care for the patient or to assist the patient in moving up in bed (2). Caution the patient`s family not to lift or remove the weights. Administering analgesics is appropriate, but the nurse should assist the patient to realign her body first (3). Do not remove any weights when caring for a patient in skeletal traction

Question.623= A patient with a fractured tibia is being evaluated in the orthopedic clinic for possible compartmental syndrome. If the patient were experiencing this complication, she would be likely to complain of

Rational.623= The signs and symptoms of compartment syndrome include pain, pallor, pulselessness, paresthesia, and paralysis.

Question.624= A postoperative patient returns from the PACU with two chest tubes inserted in his right chest. The upper chest tube serves the purpose of

Rational.624= An upper chest tube is used to remove air. Two chest tubes may be inserted on one side: an anterior-superior tube for the removal of air, and a posterior-inferior tube for removal of fluid. The lower chest tube is inserted to remove fluid which may possibly have some clots (1). No chest tube will prevent clots (2). Milking a chest tube (this procedure is not allowed in many facilities) helps to facilitate patency of the tube, but tube placement doesn`t facilitate "milking" (4).

Question.626= A nurse is caring for a patient who has a tracheostomy tube and requires frequent suctioning. When performing tracheal suctioning, the nurse should limit the procedure to periods of

Answer4.626= 5 to 10 seconds.

Question.627= The nurse is reviewing with the staff a care plan for a patient with myasthenia gravis. She explains that the main objective of therapy for a patient with myasthenia gravis is to

Rational.627= Medical management of the patient with myasthenia gravis is primarily accomplished by giving anticholinesterase medication to maintain an effective balance of acetylcholine and cholinesterase at the neuromuscular junction. Stimulating synaptic terminals to produce ACTH or giving ACTH are not effective treatments (1) (4). Supportive care would be inadequate, because medication is available to treat this condition (2).

Question.631= After consultation with the physician, a 71-year-old female is weaned from mechanical ventilation. After the weaning trial has been in progress for an hour, the patient`s ABGs and vital signs are blood pressure 140/78, pulse 94 (sinus tachycardia), respiratory rate 22, Sp0 with subscript((2)) .98, pH 7.23, Pa0 with subscript((2)) 91, PaCo with subscript((2)) 59, HCO with subscript((3)) 24. Based on the above information, the nurse interprets the ABGs as indicating

Rational.631= Respiratory acidosis is present when the pH is decreased and the PaCO with subscript((2)) is increased. No compensation is present because the kidney hasn`t started to retain HCO with subscript((3)). The normal HCO with subscript((3)) does not indicate a compensatory mechanism (1). Acidosis is present, but not compensatory alkalosis or uncompensated alkalosis (2) (4).

Question.632= A patient receiving radiation therapy complains about the dark black marking placed on her skin. What is the most appropriate comment by the nurse regarding these markings?

Rational.632= Skin markings are used as a marker for treatment areas and should not be washed away. They are not permanent or caused by the radiation

Question.635= A patient with a pituitary tumor is transferred to a surgical floor after having a transphenoidal hypophsectomy. The nurse caring for this patient is aware that which of the following procedures would be contraindicated in the immediate postoperative period?

Rational.635= Frequent mouth care should be given, but the teeth are not to be brushed, because this may damage the gingival suture line. Advancing the diet as tolerated, observing for signs of meningitis, and maintaining the bed at a 30-degree angle are appropriate interventions.

Question.637= The nurse is caring for a patient who is admitted for Addisonian crisis. Which of the following symptoms would the nurse expect the patient to exhibit?

Rational.637= The patient experiencing an Addisonian crisis experiences the extremes of hypotension, hyponatremia, dehydration, and hyperkalemia. The other options are not clinical manifestations of a sudden life-threatening Addisonian crisis.

Question.638= A common complication resulting from bacterial meningitis is

Rational.638= Most complications from meningitis involve damage to the cranial nerves. The most common complication is hearing loss.

Question.641= In children, staring, inattentiveness, or daydreaming may be signs of

Rational.641= Absence seizures result in brief loss of consciousness. They are usually very brief (5 to 10 seconds) and do not include the dramatic symptoms (e.g., falling, rigidity, pallor) characteristic of tonic-clonic seizures.

Question.647= To decrease neurologic sequelae, children with spinal cord injuries that result in motor deficits are given

Rational.647= Methylprednisolone is administered in high doses. The drug must be started within 8 hours of the injury to achieve maximum anti-inflammatory effect.

Question.648= Digoxin overdose is more common when the child`s

Rational.648= Digoxin overdose is more common when serum potassium levels are low. Many children with heart disease receive diuretics that cause potassium to be excreted in higher amounts.

Question.649= Children with Kawasaki may disease often receive aspirin primarily for

Rational.649= Children with Kawasaki disease develop thrombocytosis, which places them at risk of thromboembolism. Aspirin is given until the platelet count returns to normal.

Question.651= Recombinant human deoxyribonulease (rhDNase) is an aerosolized medication administered to children with cystic fibrosis for the purpose of

Rational.651= This medication helps to thin and liquify tenacious mucus in the bronchioles and maintain airway patency.

Question.654= The nurse checks the young patient in Bryant`s traction. Proper body alignment is being maintained if the legs are perpendicular to the trunk and the buttocks are

Rational.654= To maintain the prescribed balance of pull and countertraction in Bryant`s traction, the child`s buttocks must not be allowed to rest on the mattress. The angle of the hips should be 90 degrees.

Question.658= A characteristic lesion that occurs in children with measles (rubeola) is

Rational.658= Koplik spots appear in the mouth about 2 days before and after onset of the rash. They are small, irregular, bluish-white spots on a red background.

Question.661= Which of the following abnormalities is a sex chromosome defect?

Rational.661= Klinefelter syndrome is the most common of all sex chromosome abnormalities occurring in about 1 in 800 - 1000 live male births. It is caused by the presence of one or more extra X chromosomes.

Question.664= When administering IV medications to children, it is recommended that IV tubing be flushed after an infusion because

Rational.664= A significant amount of the medication may still remain in the distal tubing. For example, if the tubing held 15 mL and the medication were mixed in 30 mL of IV fluid, 50% of the medication would remain in the tubing if not flushed through.

Question.667= Giving an infant formula that is mixed with too little water can cause

Rational.667= Improper mixing of the formula introduces a high solute intake without adequate water, which is a cause of hypernatremia.

Question.670= Neonates are at higher risk than are older children for loss of fluid in the urine because

Rational.670= Neonates have a limited ability to concentrate or dilute urine thus they are unable to conserve or excrete fluid in response to fluid shifts.

Question.678= A 37-year-old G VII, P VI gave birth in the LDRP room one hour ago. She was straight catheterized for 950 mL of clear yellow urine prior to birth. Her labor lasted 8 hours. Since birth, her vital signs have been stable, fundus firm and 1 cm above the umbilicus with moderate, bright, red vaginal bleeding. Which of the following factors would the nurse assess for because it would predispose the patient to pospartum hemorrhage?

Rational.678= Postpartum hemorrage is defined as blood loss of 500 mL or more after birth. Risk factors for early postpartum hemorrhage include uterine atony, trauma, and lacerations. Grand multiparity (5 or more births) is a predisposing factor for uterine atony as well as previous uterine atony, overdistention of the uterus, and precipitous labor. Postpartum hemorrhage may also occur later, or 24 hours after birth.

Question.682= A patient requests information about the cause of megaloblastic anemia during pregnancy. The nurse should respond

Rational.682= During pregnancy, increased maternal red blood cell production and fetal demands can result in folic acid or folacin deficiency. Folic acid is a co-enzyme necessary for the synthesis of nucleic acids. Nucleic acid and nucleoprotein synthesis are required for the production of red blood cells.

Question.683= The nurse obtains the following assessment data for a female, 12 weeks pregnant with her third child: P = 112 (resting), R = 20, T = 99degreeF, profuse sweating, pronounced palmar erythema. Based on this data, the nurse could anticipate the physician ordering a

Rational.683= Symptoms of hyperthyroidism indicate the need for further diagnostic assessment. These symptoms include tachycardia, weakness, increased appetite, heat intolerance, sweating, enlarged thyroid, exophthalmos, nervousness, weight loss, and the tremors. She may also exhibit pronounced palmar erythema and resting pulse rate greater than 100 beats per minute.

Question.687= A patient receiving thyroid medication for hypothyroidism asks the nurse whether she can breast-feed. The nurse should respond

Rational.687= A woman receiving thyroid replacement therapy can still breast-feed, as only minimal amounts of thyroid hormone are passed to the infant through the breast milk.

Question.688= A patient is scheduled for a culdocentesis. The nurse explains that the test

Rational.688= Culdocentesis may be used to confirm ectopic pregnancy, especially when ultrasound is not available. The physician inserts a needle through the posterior wall into the cul-de-sac and aspirates fluid. Aspiration of nonclotting blood may indicate a ruptured ectopic pregnancy.

Question.691= A new mother asks if amniocentesis can help detect a fetus with spina bifida (myelomeningocele). The nurse should respond that yes, during the fourth month, assessment of the amniotic fluid would indicate a nervous system defect if

Rational.691= Between the 14th and 16th weeks of pregnancy, abnormally high levels of a-fetoprotein in the amniotic fluid indicate a neural tube defect. Although the extent of the defect cannot be determined, an amniocentesis should be performed before the 18th week because, after this time, a-fetoprotein levels decrease. In addition, there would still be time for an induced abortion.

Question.692= The nurse assessing a newborn, initiates the Moro reflex by

Rational.692= The Moro, or startle, reflex indicates the newborn`s sense of balance. A sudden stimulus such as jarring the infant or making a loud noise should elicit the reflex. The baby should be lying quietly in the supine position to provide the best response. Gentle handling of the infant and the crib will not elicit the reflex.

Question.693= After his arrival from the delivery room, the infant`s initial temperature is 36degreeC (96.8degreeF). Nursing interventions should include recording the temperature, dressing and wrapping him, and then placing him in a (an)

Rational.693= Immediately after birth the baby`s temperature is the same or slightly higher than the mother`s. Because he is wet, suddenly exposed to a dry, cool environment, and has a large body surface for weight, he loses heat via evaporation, conduction, convection, and radiation. His immature hypothalamus cannot initiate mechanisms such as shivering to raise his body temperature. Placing him in a heated crib under a radiant warmer should raise his body temperature to 36.6degreeC (97.8degreeF) in 2 - 4 hours.

Question.694= A baby is suspected to have brachial plexus paralysis of the left arm. Which observation would the nurse make during the newborn assessment to confirm the diagnosis?

Rational.694= A normal tonic neck reflex should reveal that when the head is turned to one side, the arm on the same side is adducted and extended in a fencing position. The infant with brachial plexus palsy has flaccid paralysis of the affected arm, which does not extend when the tonic neck reflex is elicted. Brachial plexus is not a congenital defect, so the affected arm is not longer, but may appear that way because of the limpness in that arm (1). There is no resistance of the arm when adducted (3). When the infant has limited use of the affected arm or asymmetric Moro reflex, a fractured clavicle is suspected (4).

Question.695= When the nurse elicits a baby Moro reflex, his arms and legs adduct and extend simultaneously. His arms then adduct in an embracing position, and his legs flex with his soles turned inward as he starts to cry. The nursing intervention is to

Rational.695= A normal Moro reflex should be symmetrical and complete. An incomplete or absent Moro reflex indicates brain trauma. An asymmetrical reflex reveals a muscle or peripheral nerve injury, as in Erb-Duchenne paralysis. The response was symmetrical and complete (1). The response was normal, not hyperactive (3). Since the response was symmetrical, no weakness or paralysis is suggested (4).

Question.698= A 6-year-old child is admitted with a diagnosis of asthma. An early sign of respiratory distress would be

Rational.698= An increased pulse rate indicates respiratory distress and a decrease in the oxygen content of the blood.

Question.700= A young child has a diagnosis of nephrosis. Which of the following medications would the nurse plan to discuss with his parents?

Rational.700= The use of glucocorticoids has been found to have a therapeutic value in treating nephrosis.

Question.702= The clinic nurse is teaching the parents of a newborn about the immunization schedule they will follow. The immunizations that should be started after the first birthday or at 15 months are

Rational.702= The MMR is recommended to be given at 15 months. The presence of the maternal antibody to measles in the infant`s blood is sufficiently reduced by 12 months, so the vaccine is effective if given after the first birthday.

Question.706= When monitoring a 10-year-old child who has undergone a cardiac catheterization, which of the following signs would have the highest priority for continued monitoring?

Rational.706= Cardiac catheterization is a diagnostic procedure but not without risks, one of which is transient arrhythmias.

Question.709= A young patient with cyanotic heart disease would present with

Rational.709= The child with a cyanotic heart defect is more likely to have frequent respiratory infections because of lowered resistance and poor cardiac function.

Question.710= The nursing priority when caring for a child in the early phase of Guillain-Barre syndrome is

Rational.710= Guillain-Barre syndrome results in muscle weakness that begins in the legs and spreads to the trunk, chest, neck, face, and head. The child may experience weakness of the respiratory muscles, resulting in inadequate ventilation that may necessitate intubation and mechanical ventilation.

Question.713= Follow-up evaluation of children who have recovered from meningitis is important because they

Rational.713= The child is at risk of developing complications from meningitis. The more common sequelae include hearing loss, attention deficits, seizures, developmental delay, and septic arthritis.

Question.725= Because drug therapy is often not successful, the treatment of choice for children with aplastic anemia is

Rational.725= Aplastic anemia results from the failure of the bone marrow to produce adequate numbers of blood cells. A bone marrow transplant from a compatible sibling or family member donor is the treatment of choice.

Question.729= Discharge instructions for a child who has had rheumatic fever need to emphasize compliance with

Rational.729= On discharge, a daily low-dose antibiotic is prescribed or a monthly long-acting antibiotic injection is given. This prophylactic treatment may be indefinite, and the family must understand the importance of preventing future strep infections, and that heart damage can result from recurrent rheumatic fever.

Question.731= A child with thalassemia minor or thalassemia trait would exhibit which of the following symptoms?

Rational.731= Thalassemia minor or thalassemia trait produces a mild microcytic anemia.

Question.732= Which of the following locations would place a child with sickle cell anemia at greatest risk for a sickle cell crisis?

Rational.732= Hypoxia resulting from general anesthesia is a major surgical risk. Sickle cell crisis could begin during surgery, placing this patient at high risk. Special precautions must be taken before any surgery. Therefore, it is necessary that children with sickle cell anemia be appropriately identified prior to surgery.

Question.742= An infant is 2 months old and had repair to a cleft lip 10 hours ago. He is now crying. The most important nursing intervention would be to

Rational.742= Crying increases tension on the suture line. The nurse should anticipate the infant`s needs and provide immediate comfort by having his mother hold him.

Question.747= A safety precaution when caring for a child with a tracheostomy is to

Rational.747= A clean tube of the same size should be immediately ready for insertion in case the tracheostomy tube should fall out. A second tube of the next smaller size should also be available in case there is difficulty inserting the same-size tube.

Question.748= A child with cystic fibrosis is at risk for a deficiency in which vitamins?

Rational.748= Vitamins A and D are fat soluble. A child with cystic fibrosis may have difficulty digesting fats - so these vitamins will not be assimilated - because of the absence of pancreatic enzymes. Children are given water soluble forms of vitamins A, D, E, and K.

Question.750= To determine unresponsiveness in an infant, the nursing action is to

Rational.750= Unresponsiveness is determined by gently tapping the infant and speaking loudly enough to get a response.

Question.751= Infants develop bronchopulmonary dysplasia as a result of

high pressure vent Rational.751= Bronchopulmonary dysplasia is a condition that results from the treatment given to infants with severe respiratory problems.

Question.755= The young patient has just been admitted for evaluation. The nurse reports that the physical assessment reveals a finding that may indicate coarctation of the aorta, which is

Rational.755= Coarctation is a narrowing that occurs in the descending aorta. Blood flow to the legs is decreased, and flow to the arms and head is increased. The femoral pulses are weak, and the radial pulses are full. The blood pressure is higher in the arms, lower in the legs.

Question.756= The pediatric nurse is teaching inservice to a new RN staff member. She knows the RN understands accepted skill procedures when she says the dorsogluteal muscle can be used for injections for children at the age of

Rational.756= The dorsogluteal muscle develops with walking and should not be used for injections until the child has been walking at least 1 year. However, because these muscles are poorly developed, they are not the ideal choice for a child younger than 5 years old.

Question.757= A child with cerebral palsy needs a diet

Rational.757= The child with cerebral palsy requires a high-calorie diet because of increased muscle activity and feeding problems. Many children with cerebral palsy have difficulty chewing and swallowing. Offering high-calorie soft foods in small frequent feedings is often helpful.

Question.758= A child has been diagnosed with pulmonary stenosis. The nurse is teaching the parents about their child`s diagnosis and tells them that Pulmonic stenosis causes

Rational.758= Stenosis (narrowing) of the pulmonary artery valve makes it more difficult for blood to enter the pulmonary artery, causing increased pressure within the right ventricle. This leads to right ventricular hypertrophy, because the ventricle must pump harder to move the blood into the artery. r ventricle affected

Question.760= An infant has been diagnosed with patent ductus arteriosus. The nurse teaches the parents that symptoms resulting from this condition are directly related to the

Rational.760= After birth, blood in the aorta is under higher pressure, causing blood to flow through the ductus into the pulmonary artery. This extra blood entering the pulmonary system can lead to increased pulmonary vascular congestion.

Question.768= The needle size that the nurse would use to give an intramuscular (IM) injection to a newborn is a

Rational.768= You would use a thin (25-gauge), short (0.5-inch) needle for giving an IM injection to a newborn.

Question.786= Which of the following data place a patient at risk for developing pregnancy-induced hypertension (PIH)?

Rational.786= Primigravid women or women pregnant for the first time after an abortion are more susceptible to PIH than are multiparous women.

Question.789= A patient with hyperemesis gravidarum is at risk for having a (an)

Answer1.789= Infant who is growth retarded.

Question.792= The nurse should assess the small-for-gestational-age (SGA) neonate for

Rational.792= Small-for-gestational age neonates frequently develop hypoglycemia because of low glycogen stores.

Question.798= A male patient is being treated for exacerbation of his chronic obstructive pulmonary disease (COPD). Oxygen would be administered at 2 liters via nasal cannula because

Rational.798= Precise amounts of oxygen delivered to COPD patients is critical and depends on the hypoxic drive of the patient. Too much oxygen can depress respirations and cause hypoventilation. The size of the lungs is not directly related to the hypoxic drive (1). The opposite would occur in this patient if given high concentrations of oxygen (2). The statement about his lungs and chest diamter is true, but does not explain why he responds to hypoxia (3).

Question.799= Which of the following interventions would the nurse plan to do for the patient demonstrating symptoms of Addisonian crisis?

Rational.799= Addisonian crisis is a life-threatening complication of Addison`s disease (hypocorticism). Cortisone replacement can be life-saving. The patient is likely to be severly dehydrated and hypovolemic. IV saline will be run rapidly in bolus amounts (

Question.804= Several days following surgery to remove a thyroid tumor, a female patient begins to complain of muscle twitching in her legs. The most likely cause of this is

Rational.804= Accidental parathyroidectomy after removal of the thyroid gland results in a hypocalcemic condition manifested by GI complaints (vomiting and diarrhea), convulsions, and tetany.

Question.807= While walking with a 79-year-old dementia patient, the nurse shows him a car key. She asks the patient to name the object, but he is unable to. This inability to describe the key is known as

Rational.807= The failure to recognize previously known objects is called agnosia. This condition can be seen in several types of nervous system disorders such as Alzheimer`s dementia and cerebral vascular accidents. Apraxia is the inability to carry out a series of actions (1). Aphasia is a disorder in communication (2). Agraphia is the inability to write (3).

Question.811= Which of the following tests would the nurse employ when evaluating the hearing of a patient?

Rational.811= The Weber test is conducted by using a tuning fork to measure whether the patient is able to hear sound conducted to both ears. If there are conductive or sensorineural losses, it will lateralize to one of the ears. The Trendelenburg test is used to check vascular competency in the lower leg (1). The Snellen test determines vision (3). The Allen test assesses for radial and ulnar patency at the level of the wrist (4).

Question.816= A patient with carcinoma of the thyroid suddenly develops symptoms of pheochromocytoma. Which of the following symptoms would the nurse consider a top priority when planning nursing interventions?

Rational.816= Severe, life-threatening hypertension can result from excess catecholamine secretion.

Question.817= The nurse is caring for a patient with aldosteronism. The nurse should anticipate which of the following symptoms?

Rational.817= Hypertension and hypokalemia are common signs of aldosteronism.

Question.831= The nurse is preparing to administer a blood transfusion in the emergency room. After ordering the blood, the nurse`s first action would be to

Rational.831= The patient and the family should be supported prior to and during the procedure, and anxieties should be allayed by the nurse. Rechecking the order and reviewing the typing and crossmatching are done by two nurses as part of the pretransfusion checklist (1) (3). It is seldom necessary to contact the laboratory, because information on the blood bag label is sufficient to determine that the lab sent the proper unit (4).

Question.836= The patient who has had an MI hopes to resume sexual intercourse with his partner. The nurse determines that the patient understands the instructions given for resuming sexual intercourse on discharge from the hospital when he says he will

Rational.836= The strain on the recovering heart should be avoided. After a heavy meal, blood flow is diverted to the GI tract. Increased workload increases the risk of underperfusion of the coronary arteries. The patient should lie on his or her back while the partner kneels to take some of the weight from the patient (1). Answer (2) is not an appropriate recommendation for this patient. The patient recovering from an MI without complications may usually resume sexual activity in about 5 to 8 weeks. One index of readiness is the ability to walk up two flights of stairs without becoming dyspneic (4).

Question.837= While developing a care plan for a patient receiving intermittent pelvic traction (to relieve lower back pain due to sciatica), the nurse would include

Rational.837= Pelvic traction is skin traction that can be manipulated by the patient. It can be intermittent and the patient may not require it, except during specific times. The patient`s pelvis should partially rest on the bed for support (2). The patient`s knees should be slightly bent to relieve pressure on the lower back (3). Pelvic traction does not involve pins inserted into any extremity, as is the case in skeletal traction (4).

Question.838= A 45-year-old male patient comes to the clinic for evaluation. He is receiving continuous ambulatory peritoneal dialysis. For the past three cycles, he has retained between 250 and 400 mL with each cycle. He has no urine output and has not moved his bowels in 3 days. The effluent is clear. The nurse should

Rational.838= The physician should be notified When the CAPD patient has a retention of more than 250 mL of fluid, as glucose absorption may become a problem. The physician should also be notified if the outflow is 500 mL greater than the amount that was instilled. Dialysate solution concentrations should not be changed without a physician`s order (1). The patient may be constipated, but the overall priority at this time should be to notify the physician (3). The next cycle should not be started until after discussing this problem with the patient`s physician (4).

Question.842= An elderly patient is being transferred to the rehabilitation unit. The nurse explains that the focus of his care will be "tertiary prevention," or

Rational.842= Tertiary prevention care includes restorative and rehabilitation activities to obtain optimal level of functioning. Screenings and saftey education are examples of primary prevention. Illness care, such as first aid, is an example of secondary prevention.

Question.847= A 45-year-old female patient refuses to get out of bed and resists the nurse`s attempts to assist her to sit up in a chair. If the nurse attempts to remove the patient from her bed without the patient`s approval, the nurse could be charged with

Rational.847= Battery is defined as the offensive touching of the body of an individual. It is not necessary to have caused harm. Assault is to place someone in fear of harm. Negligence is conduct that falls below the standard of practice. A tort is a civil wrongdoing.

Question.851= Which of the following disorders would the nurse classify as a disease that is spread through fomite transmission?

Rational.851= Formites are nonliving objects that transmit infection. Examples include the spread of ringworm of the scalp via contaminated combs, hats, or the upper back portion of bus seats. Tuberculosis is an organism spread through airborne droplets. Lyme disease and Rock Mountain spotted fever are spread through infected ticks.

Question.857= Screening for elevated blood lipid levels is an example of what level of prevention?

Rational.857= Early diagnosis and prompt treatment are part of secondary prevention. Primary prevention is aimed at health promotion and protection against illness. Tertiary prevention is aimed at rehabilitation and return of the patient to maximum level of functioning.

Question.859= Conducting a diabetes screening clinic for well adults is an example of

Rational.859= Secondary prevention includes screening techniques and treatment of early stages of disease.

Question.863= The method most likely to be used to reduce the heart rate in a child experiencing supraventricular tachycardia is to

Rational.863= Applying ice to face or iced saline solution to the face causes vagal stimulation that may reduce the heart rate. An older child can perform the Valsalba maneuver to increase intrathoracic and venous pressures, thus slowing the heart rate. Other activities will not reduce the heart rate.

Question.865= When a child is diagnosed with cystic fibrosis, the news is often met with disbelief by the parents because

Rational.865= Cystic fibrosis is an inherited autosomal recessive disorder, which means that both parents must carry the gene in order for a child to have the disease. In past generations most children with cystic fibrosis died very young, often without ever having been diagnosed. Adults are often unaware that they are carriers of this disorder.

Question.869= A 4-year-old child has had chronic otitis media. Parent education after myringotomy and insertion of tympanostomy tubes would include

Rational.869= The tubes allow air and fluid to flow from the middle to the outer ear and also allow water to flow in the opposite direction. To avoid introducing bacteria directly into the middle ear, the child should wear earplugs when swimming because submerging or diving places the child at increased risk for otitis media. Noise has no effect on tube placement and tubes fall out spontaneously.

Question.871= Which of the following symptoms may be an indication of respiratory alkalosis in a young child?

Rational.871= Respiratory alkalosis leads to stimulation of the nervous system. The clinical signs include nervousness tingling in fingers, toes, and around the mouth and tetany.

Question.876= In counseling the parents of a child with idiopathic thrombocytopenic purpura (ITP) who has not responded to steroids and immunoglobulins, the nurse will reinforce the physician`s plan that the next treatment would be a

Rational.876= In ITP the platelets are destroyed faster than they are produced by the bone marrow. The spleen is the organ that destroys the platelets. Splenectomy is reserved for those patients in whom ITP has persisted for 1 year or longer. Transfusions and transplantation are not indicated in the treatment of ITP.

Question.877= To avoid risk of transmission of the human immunodeficiency virus (HIV), children with mild hemophilia are often treated with

Rational.877= DDAVP is a synthetic drug that temporarily increases the activity for factor VIII two- to threefold. The chance of HIV transmission is eliminated, because the drug is synthetic and thus contains no human blood components. Factor VIII is not used to treat mild hemophilia and cryoprecipate is no longer recommended.

Question.883= A patient is being assessed for suicidal risk. Which of these factors in the patient`s history places the patient at risk for suicide?

Rational.883= Suicide tends to run in families. It may be that because one family member committed suicide, suicide can be viewed by other family members as an "appropriate" coping mechanism. The other answers do not place a person at risk for suicide.

Question.885= The nurse and a patient who resides on a psychiatric unit are discussing the idea of contracting. The patient has a history of violent behavior. The nurse will know that the patient needs further instruction about the concept of contracting if the patient makes which of these statements?

Rational.885= Contracting is a system whereby the nurse, staff, and patient together decide how aggressive behavior will be handled.

Question.894= On the psychiatric unit, the nurse is talking to a patient who consistently claims that his food is being poisoned. The nurse should recognize this symptom as

Rational.894= Delusions are fixed beliefs maintained despite experience and evidence to the contrary. Hallucinations are a distorted misrepresentation of reality, channeled through the patient`s senses (2). Projection is putting feelings that are unacceptable to the self onto another person or object (3). Dissociation describes the abrupt disengagement with the present and loss of contact with reality (4).

Question.895= A patient who has been diagnosed as having schizophrenia is readmitted to the hospital. It is the patient`s third admission in 3 years. The patient`s mother says to the nurse, "I don`t understand what my son is doing wrong. Every year he has to be hospitalized." It is appropriate for the nurse`s response to be based on which of these understandings?

Rational.895= Schizophrenia is a chronic disease and various factors may lead to intermittent hospitalization for acute exacerbation of symptoms. These admissions should not be viewed as a treatment failure. Noncompliance would lead to repetitive and frequent hospitalization (2). There is no way to predict the frequency of hospitalization (3). Reevaluating treatment options can be done in the outpatient setting as well as in the hospital (4).

Question.901= A patient is diagnosed as having a schizoid personality disorder. The nurse can expect this patient to exhibit a pattern of

Rational.901= Schizoid personality disorder produces a pattern of detachment from social relationships and a restricted range of emotional expression.

Question.903= When the nurse begins to give the patient a prescribed medication, the patient says, "You`re poisoning me with those pills." Which of these responses by the nurse is most appropriate?

Answer1.903= "I`ll get a fresh package of pills so you can see me open the sealed package."Rational.903= patients who are having delusions, such as thinking that their medications are being poisoned, need to be given an appropriate degree of choice, e.g., seeing a closed medication packet and watching the nurse open the packet. The paranoid thought process isn`t clarified by attempts to explain or establish logic.

Question.905= A young woman patient on the psychiatric unit has a fluctuating mental status. She can be calm and cooperative, and yet an hour later be paranoid and combative. Her response to environmental stimuli ranges from an appropriate response to fleeting illusions. The most imperative nursing care need for the patient is to

Rational.905= Providing safety for the patient and staff is the primary concern on a hierarchy of needs. Encouraging fluid intake, maintaining skin integrity, and providing reality orientation are less important needs.

Question.908= When the nurse suspects that an individual in crisis may be having thoughts of suicide, the appropriate intervention is to

Rational.908= Asking directly about a suicide plan will help you determine the degree of risk for the patient. Not asking, inquiring about moral beliefs, and trying to persuade the patient that suicide is a poor choice are less effective measures.

Question.917= A 50-year-old Asian American woman was admitted to a psychiatric unit. She states that she does not know why she is on a psychiatric floor because she definitely is not crazy. In your initial interview, you choose an unstructured format in order to

Rational.917= An unstructured format will help establish a trusting relationship, something that must occur before she can begin to identify and work on problems. Teaching about psychiatric units will occur after you obtain more information about the patient`s problems (2). Trying to convince the patient she is in the right place will invalidate her perceptions (3). It`s important to focus on both verbal and and nonverbal behavior (4).

Question.918= As you walk onto a psychiatric unit, a 16-year-old adolescent yells at you, "You are like all the rest. You are full of !@#$!" The best response to her would be

Answer3.918= "It sounds like you are upset."

Question.924= A young woman is admitted to an acute care psychiatric unit for having attempted suicide. She is a recent graduate of a nursing program and is employed at a local hospital. You recognize that interaction with her may be more complicated because she is a nurse. The most obvious potential problem for you would be

Rational.924= Because the patient is also a nurse, identification may make it difficult to maintain well-defined boundaries. Because the patient is also a nurse does not relate to remaining objective (2). Confidentiality is an issue with all patients (3). Your relationship with the patient should be therapeutic, not professional (4).

Question.928= The drug of choice for alcohol detoxification usually is

Rational.928= Benzodiazepines are commonly used to decrease the CNS irritability that occurs during alcohol withdrawal. Barbiturates are used for benzodiazepine or sedative-hypnotic withdrawal (2). Antihistamines are used to combat sleeplessness (3). A neuroleptic would be given to treat psychotic disorders (4).

Question.929= A patient on lithium believes salt is bad for people, so she eliminates all sodium from her diet. In counseling her, the nurse should

Rational.929= Deficiency of sodium results in more lithium being reabsorbed, thus increasing risk of toxicity.

Question.930= A male patient with the diagnosis of depression is given a monoamine oxidase (MAO) inhibitor. The nurse knows that MAO enzymes destroy

Rational.930= Neurotransmitters are the targets of MAO inhibitors. Normal transmission with a MAO is partially responsible for keeping synaptic levels of the neurotransmitter low.

Question.931= A 38-year-old patient who has been diagnosed with schizophrenia comes to the nurse and states that his legs are moving like a robot. The nurse understands this to be a sign of

Rational.931= Depersonalization is often associated with an altered perception in which persons feel as if they are having an out-of-body experience. They report robot-like movements and loss of sensations in different body parts. Delusional behavior is seen in examples of delusions of grandeur ().

Question.932= An elderly gentleman is brought to in the clinic and states, "The radio is sending messages to the police that I am to be interrogated and arrested." The nurse interprets this communication to be consistent with

Rational.932= Ideas of reference are defined by the patient`s belief that everything happening around him is related to himself. Delusions of grandeur represent an exaggerated belief of self-importance (2). Hallucinations are not a delusional experience (3). A disturbance in volition represents an inability to initiate activities (4).

Question.939= Electroconvulsive therapy has been ordered for a patient, who has severe depression. What is the most important assessment to complete before the procedure?

Rational.939= patients who are scheduled to undergo ECT should have a complete history and physical examination to rule out any contraindications for therapy. Diseases such as brain tumors, seizures, and trauma to the head may be contraindications for ECT. A family history of unremitting depression does not provide information that would contraindicate ECT (2). A social history that includes lengthy bouts of sadness is not a contraindication of ECT (

Question.940= A patient is being maintained on lithium therapy for bipolar disorder. Her serum lithium level is 0.9 mEq/L. The nurse assesses this level as

Rational.940= The therapeutic range for effective ongoing management of the patient on lithium carbonate can be as low as 0.6 mEq/L or as high as 1.5 mEq/L. A value of 0.9 mEq/L is within the therapeutic range for lithium.

Question.941= While caring for a 56-year-old patient, the nurse observes a newly developed slight tremor when he is at rest. The patient is taking prescribed chlorpromazine (thorazine) as part of his management plan for treating schizophrenia. Which of the following interventions would be most appropriate?

Rational.941= The development of extrapyramidal symptoms is a side effect of this medication, and tremors are part of the constellation of symptoms. Medications are frequently prescribed to counteract and relieve these symptoms. The patient has begun to develop symptoms of side effects. The physician should be notified without waiting for other problems to occur (1). Reassurance that his is a normal response is not the correct action (3). The patient has developed side effects, and it`s important to notify the physician. The nurse should not discontinue the medication without an order (4).

Question.947= Which of the following foods should the patient taking an MAO inhibitor such as isocarboxazid (Marplan) be taught to avoid?

Rational.947= When using this category of medications, the patient should be cautioned against eating foods that contain tyramine. This includes many foods that have been fermented in some way, such as yogurt, aged cheese, sour cream, champagne, beer, pickled herring, shrimp, overripe bananas, and yeast extracts. Spinach, haddock, and tomatoes do not contain tyramine and would not be contraindicated for the person taking an MAO inhibitor.

Question.1612= The intervention that would be contraindicated when administering oxygen to a patient with chronic obstructive pulmonary disease (COPD) is

Rational.1612= Administering oxygen at 5 liters per minute is too much oxygen for a person who has chronic lung disease. It will create respiratory arrest potential because these people respond to a hypoxic drive. Oxygen dries the mucous membranes so moisture to the nose is a comfort measure properly done by the nurse. Since oxygen is a flammable gas, it is important for staff and visitors to be aware that it is being used in the room they are entering.

Question.1608= When caring for a patient receiving a sitz bath, the intervention is to

Rational.1608= To maximize healing and relaxation, the sitz bath should last from 10-20 minutes. Water temperature should be about 110 degrees F. Sitz baths are usually done 3 or 4 times per day to provide cleansing and comfort.

Question.1607= The causative agent of syphilis is

Rational.1607= The causative agent of syphilis is Treponema pallidum. Hemophilus ducreyi causes chancroid neisseria causes gonorrhea and chlamydia trachomatis causes chlamydia.

Question.1603= A patient injects his or her regular insulin at 6 AM every morning, has breakfast, and then jogs for three miles. This patient should be taught to

Rational.1603= Exercise increases insulin utilization. The patient should be prepared to treat hypoglycemia as the action of onset of regular insulin is from 30 minutes to 1 hour.

Question.1600= When administering neomycin sulfate as part of the management plan for a patient to prevent hepatic coma, the nurse recognizes that this medication acts by

Rational.1600= Neomycin works locally on the bowel to decrease or eliminate the normal bacterial flora, which then decreases the amount of ammonia produced. The impaired liver may not be able to metabolize the ammonia and hepatic failure may result.

Question.1598= Which of the following oxygen-delivery systems would the nurse use to provide the most accurate concentration of oxygen?

venturi Rational.1598= This system can deliver oxygen in the 24% to 100% FIO with subscript((2)) range with flow rates of 4 to 10 L/min. It is the most accurate of the delivery systems mentioned.

Question.1596= A female patient is recovering from an exacerbation of her chronic bronchitis. Her sputum suddenly changes from a pale-green color to a rusty-brown color. Assessment of her lungs reveals bilateral crackles that are scattered in both bases and do not clear upon coughing. This change in sputum is most likely to represent

Pneumococcal pneumonia.

Question.1593= A sudden increase in pain in a patient admitted with duodenal ulcers may indicate that which condition is developing?

Rational.1593= Acute gastric perforation is characterized by sudden, sharp, intolerable pain. Bleeding or extending ulcers do not cause pain. A pancreatic ulcer is not related to duodenal ulcers.

Question.1592= Cholinergic crisis in myasthenia gravis is due to

Rational.1592= Myasthenia gravis is medically managed with the use of anticholinesterase medications (or cholinesterase inhibitors). The dose and medication schedule should be the minimal amount needed to provide maximal improvement in the vital muscles of swallowing and breathing. Side effects of the drug, called cholinergic reactions, result from excess acetylcholine.

Question.1591= The primary effects of sepsis include

Rational.1591= Sepsis is characterized by hypotension and altered tissue perfusion. Sepsis stems from toxins produced by microorganisms. Once the organism enters the body, a set of complex humoral, cellular, and biochemical mediators are released causing epithelial damage, peripheral vasodilation, and increased capillary permeability. Secondarily, the patient would become acidotic with decreasing pH and PaO with subscript((2)) levels, followed eventually by loss of consciousness.

Question.1590= A patient is admitted to the intensive care unit with signs and symptoms of ascending paralysis and respiratory failure. The critical care nurse would investigate for a past history of

Rational.1590= The nurse would suspect Guillain-Barr syndrome, which is an acute, rapidly progressive inflammation and demyelination of nerve endings of the peripheral nervous system that predominantly affects motor funtion. Sensory and motor loss usually occur rapidly in an ascending order. Current research reports that 60-70% of patients with Guillain-Barr syndrome report a mild febrile illness, usually respiratory or gastrointestinal, 1-3 weeks before onset of symptoms. Trauma to the head or spinal cord may cause paralysis below the area of the injury.

Question.1589= The nurse is administering medications to a patient with acute myelocytic leukemia. One of her medications is allopurinol, and she questions why she is receiving this. An appropriate nursing response would be

Rational.1589= Chemotherapy may cause hyperuricemia due to rapid tumor destruction. Allopurinol is administered before and during therapy to break down urates. This is standard treatment and is being done to prevent symptoms of gout.

Question.1588= A 54-year-old man comes to the clinic for evaluation of a painless lump that has been present on the side of his neck for the past 6 weeks. Hodgkin`s disease, stage II, was subsequently diagnosed. He is to begin radiation therapy and is questioning why "an x-ray can kill this cancer." The nurse`s best response should be

Rational.1588= Therapeutic radiation uses ionizing forms of radiation that must produce cell death to be effective. This type of radiation uses high-energy beams to treat malignant cells. It is not the same as x-rays. There are side effects associated with radiation therapy and the patient will be informed about those.

Question.1587= A compensated respiratory acidosis would demonstrate which two blood gas findings?

Rational.1587= The body compensates for respiratory acidosis by increasing the HCO3 and the PaO2.

Question.1580= Following a gunshot wound to the abdomen, a 27-year-old male has a complete colectomy with creation of an ileostomy. Nursing measures that will be necessary for the patient, considering the fact that the function of the large intestine has been eliminated, will include

Rational.1580= Complications following an ileostomy include fluid and electrolyte imbalance therefore, accurate fluid intake and output records must be maintained at all times. An intact stomach and small intestine means that the patient does not need tube feedings, enzymes, or emulsifying agents.

Question.1577= As the nurse is orienting a new nurse to the unit, and she asks about treatment the patient is receiving for an infection. The new nurse tells the nurse she remembers being taught that aminoglycosides (the patient is receiving gentamicin) can cause renal failure. The patient has a normal urinary output and the following lab values: Na 142, K 4.6, Cl 103, HCO3 21, creatinine 3.2, BUN 54. She asks if this patient is at risk for developing renal failure. The nurse would answer

Rational.1577= This patient has elevated creatinine levels (normal is 0.5-1.5 mg/dL). Serum creatinine levels do not elevate until half of the nephrons are not functioning. patients with elevated creatinine are likely to have severe renal impairment. BUN values vary widely according to the dietary intake of protein, but values elevated to more than 20 mg/dL suggest renal insufficiency. Renal output is important but the creatinine level denotes renal impairment (1). The normal range for creatinine is 0.5-1.5 mg/dL (2).

Question.1574= To assess third cranial nerve function, the nurse would test the patient`s

Rational.1574= The third cranial nerve governs oculomotor responses. This includes pupil response to light. Eye movement is controlled by Cranial Nerve IV: Trochlear (2). Cranial Nerve I controls olfactory function: smell (3). Cranial Nerve VIII controls vestibulocochlear function (acoustics or hearing) (4).

Question.1573= A 41-year-old female is admitted to the unit with an exacerbation of chronic lymphocytic leukemia. She states that she has had small amounts of vaginal bleeding. Ecchymotic areas are noted on her arms and legs. Laboratory data reveal the following:

Rational.1573= A normal platelet count is 15,000-45,000/mm3. A low or below normal count places the patient at risk for bleeding. A normal WBC count is 3400-10,000/mm3. patients with a low or below normal count are at risk for infection. The other choices are partly correct, but the patient needs both protective isolation and bleeding precautions.

Question.1572= The rationale for using hypertonic solutions in burn resuscitation is

Rational.1572= Hypertonic solutions will decrease edema because they will pull water out of the cells by osmosis. The hypertonic solution has no nutritional value (1). The goal of fluid replacement in burn resuscitation is to maintain a serum sodium concentration of 140 Eq/L

Question.1571= A 72-year-old female is admitted to the unit following a fall at home. Her daughter explains that her mother attempted to stand after dinner and immediately fell. Currently she is awake but unable to move her left side. She is able to talk and is alert and oriented. Admission vital signs are blood pressure 176/100, pulse 62, respiratory rate 16, temperature 36.8 degrees C. The patient`s pupils are equal and reactive and her eye movements are normal. The patient states that she has been healthy and has never needed to see a doctor. Based on the preceding information, the nurse would continue assessing for

Rational.1571= Left-sided weakness (left hemiparesia) or paralysis (hemiplegia) indicates a stroke involving the right cerebral hemisphere, because the motor nerves cross in the medulla before entering the spinal cord and periphery. A left-sided CVA would cause right hemiparesis or hemiparalysis (1). Hemianopsia is blindness in half the visual field, resulting from damage to the optic nerve (2). Hypertonia (spastic paralysis) tends to cause fixed positions or contractures of extremities (4).

Question.1570= Brudzinski`s sign is best described by which of the following definitions?

Rational.1570= Brudzinski`s sign is positive when the nurse gently flexes the patient`s head and neck to the chest and flexion of the hips and knees results. This is an indicator of meningitis.

Question.1569= The critical care nurse should recognize that a major complication of diabetes insipidus could include

Rational.1569= Diabetes insipidus is a condition characterized by impaired renal conservation of water caused by a deficit of ADH. This causes the excretion of large volumes of diluted urine, leading to dehydration. Serum sodium levels increase (not decrease), due to decreased or absent ACTH, if water replacement does not occur (2). Serum osmolarity is increased, due to a decrease in ADH and increased water loss (3). Due to dehydration, intravascular volume is low therefore, hypotension occurs. Tachycardia results as a compensatory response (4).

Question.1567= If a patient develops a grand mal (tonic-clonic) seizure, the initial nursing action is to

Rational.1567= The first priority is to protect the patient from injury resulting from environmental hazards, such as siderails, falls, and clothing. Forcing an airway into the patient`s mouth after a seizure has begun should never be attempted. This is likely to injure the patient`s teeth and risk aspiration on tooth fragments (1). Oxygen therapy may be used, but is not the initial priority even though cyanosis is common in a grand mal seizure. It generally resolves without treatment (3). A padded tongue blade should never be inserted after a tonic-clonic seizure has begun, because it may injure the patient`s teeth, and tooth fragments may cause aspiration (4).

Question.1566= Which of the following best describes Kernig`s sign?

Rational.1566= Kernig`s sign is positive when a patient is unable to extend his or her leg when the thigh is flexed to the abdomen. This is a positive assessment for meningitis.

Question.1565= A 36-year-old male is admitted to the nursing unit with rapidly increasing symptoms of generalized weakness following an episode of "flu." He noticed that the weakness started in his arms and legs and has progressed to his upper legs, abdomen, and chest. He has difficulty taking a deep breath. Vital signs are normal and he has some complaints of shortness of breath. Based on the preceding symptoms, which condition is likely to be developing?

Rational.1565= Guillain-Barr? syndrome is an acute inflammatory process characterized by varying degrees of motor weakness and paralysis, plus sensory and motor disturbances that occur in an ascending, distal to proximal pattern. Respiratory compromise is common. The cause is obscure, but evidence indicates a cell-mediated immunological reaction. Myasthenia gravis is a demonstration of progressive paresis that is resolved, in part, by rest. The most common symptoms involve extraocular muscles of the eyes (2). Multiple sclerosis is a degenerative disease manifested by increased fatigue, stiffness of the extremities, and flexor spasms (3). ALS (amyotrophic lateral sclerosis) is a progressive degenerative disease characterized by fatigue while talking, tongue atrophy, dysphagia, dysarthria, fasciculations of the face, and weakness of the arms and hands (4).

Question.1564= The nurse, assessing a patient with head injuries monitors for signs of increasing ICP (intracranial pressure). These signs would be

Bradycardia and hypertension.

Question.1562= Assessing for hypokalemia, the nurse will observe for ECG changes of

Rational.1562= The cardiovascular effects of hypokalemia include dysrythmias (PVCs), inverted T waves, peaked P waves, and prolonged QT interval. The other choices are incorrect. A narrowed QT interval is indicative of hypokalemia (4).

Question.1560= Trousseau`s sign is a test for the electrolyte deficiency of

Rational.1560= Tetany resulting from a decrease in serum calcium can be confirmed by a positive Trousseau`s sign. Hyperphosphatemia may be present with a positive Trousseau`s sign (1). Hypercalcemia produces lethargy and weakness, not signs of carpal spasm seen with Trousseau`s

Question.1559= A nurse in an extended care facility is caring for a patient with a cuffed endotracheal tube. The nurse understands that the purpose of inflating the cuff is to

Rational.1559= Inflating the cuff creates a seal between the tube and the patient`s trachea. This forces air exchange to take place only through the ET tube. Inflating the cuff on the ET does not make suctioning easier (1). The percentage of oxygen remains the same after the cuff is inflated

Question.1557= A patient is admitted to the intensive care unit after sustaining a knife wound to the back. Assessment findings include loss of pain and temperature on the right side and loss of motor function on the left. Vital signs are stable and he is alert and oriented. No other injuries are noted. Based on this information, which type of neurological syndrome is likely to be developing?

Brown-Squard syndrome is caused by hemisection of the spinal cord. Clinical manifestations include paralysis below the level of injury on the same side of the lesion and the perception of pain and temperature is affected below the level of injury on the opposite side of the lesion. Central cord syndrome is not a known disease process

Question.1556= The primary symptoms present in cases of autonomic hyperreflexia are

Rational.1556= Autonomic hyperreflexia occurs in patients with a spinal cord lesion at or above T6. The response occurs as a result of a noxious stimulus below the level of the lesion, causing visceral lesion activity resulting from massive sympathetic response. Symptoms include hypertension, bradycardia, and profuse sweating above the level of the injury. Autonomic hyperreflexion includes bradycardia and hypertension

Question.1554= For a patient with C-3-4 injury, the nurse should first assess

Rational.1554= In spinal cord injuries that occur above C-4, the muscles responsible for respiration are paralyzed. Heart rate, motor ability, and temperature may be important, but the priority is respiration.

Question.1553= Guillain-Barr syndrome affects the neurological component of the

Rational.1553= Guillain-Barr syndrome is an acute, rapidly progressive inflammation and demyelination of nerve endings of the peripheral nervous system.

Question.1551= When assessing a patient on the neurological unit, the nurse knows that decerebrate posturing is characterized by

Rational.1551= In decerebrate posturing, the patient has a rigid and possibly arched spine, rigidly extended and pronated arms with the wrists flexed and the palms facing backward, and extended legs with plantar flexion. An abnormal flexion response is decortication (2). Decerbrate posturing does not include hyperflexion of lower extremities (3). Decerebrate posturing is a motor response (4).

Question.1549= When auscultating a patient`s lungs, which type of adventitious lung sounds is the nurse most likely to hear with left ventricular failure?

Rational.1549= Physical signs associated with left ventricular failure include bilateral basilar rales. Wheezing is associated with asthmatic or airway restrictive disease (1). Tinkling sounds aren`t a characteristic of left ventricular failure (2). Increased respiratory breath sounds in the apex is associated with lung collapse (4).

Question.1548= What is the dominant effect of ADH (antidiuretic hormone) on the kidneys?

Rational.1548= ADH regulates the osmolarity of extracellular fluids by increasing reabsorption of water from the renal tubules.

Question.1547= A 59-year-old female with a history of alcoholism is admitted to the unit after being found unresponsive. During the next 24 hours, she develops decreased blood flow to her distal extremities, manifested by discoloration of her hands and feet. The physician believes that DIC (disseminated intravascular clotting) may be taking place. The test that will help confirm that DIC is present is the

Rational.1547= A measurement of fibrinogen degradation produces greater than 45 ug/mL which helps to confirm DIC. This increased level (normal = 10 ug/mL) indicates that intravascular clots have formed and the body is attempting to dissolve them to restore microcirculation.

Question.1546= A physician orders nitroprusside and dobutamine for a patient with CHF (congestive heart failure). The goal of this type of medication regimen is to

Rational.1546= Nitroprusside is a vasodilator which dilates arteries and veins. Arterial relaxation reduces afterload, making it easier for the left ventricle to eject blood. Dobutamine is a synthetic catlecholamine that is primarily a beta receptor stimulator. Dobutamine increases the force of contraction by as much as 30-70%. This drug combination reduces afterload and improves contractility

Question.1544= The priority nursing goal for the patient in status epilepticus is to

Rational.1544= When a patient is in status epilepticus, follow the ABC (airway, breathing, circulation) after calling for help. Do not leave the patient, maintain a patent airway, and administer oxygen via nasal cannula. Medications are given after the airway and breathing are stabilized.

Question.1541= The nurse is caring for a 37-year-old female admitted with an intracerebral bleed. The nurse on the preceding shift says that a neurological examination was performed on the patient and that she had an abnormal "doll`s eyes" test. Which of the following descriptions best describes an abnormal oculocephalic response to the "doll`s eyes" test?

Answer4.1541= The eyes follow the direction of a quick turn of the head. = An injury that stimulates the neurons of the cortical gaze center causes the patient`s gaze to remain fixed in the direction of head movement. This is called a positive "doll`s eyes" test. Normal response is for eyes to move in the opposite direction of head movement.

Question.1540= Cheyne-Stokes breathing is characterized by which of the following respiratory patterns?

Answer4.1540= Short periods of apnea followed by respirations of increasing depth that then slow again to apnea.

Question.1539= The primary function of the cerebellum includes which of the following?

Rational.1539= The cerebellum plays an essential role in modifying the force of contractions of the muscles primarily responsible for movement and the relaxation of muscles opposing movement. This role is essential for smooth, coordinated movements. The cerebellum also detects loss of balance and restores it by modifying muscle contraction. Thought and personality are higher functions and occur in the frontal lobes. Sight is controlled in the occipital lobe.

Question.1538= Following a head injury sustained in an auto accident, the patient is admitted. The nurse is assessing the patient`s level of consciousness, based on her knowledge that maintenance of an awake and alert status is dependent on the proper functioning of which two cerebral structures?

Rational.1538= Consciousness relies on the active function of the cerebral cortex. This is maintained by continuous stimulation of the cerebral cortex by nerve impulses from a series of nuclei in the brain stem that together are called the reticular activating system.

Question.1537= A patient in your unit has suffered frontal head injuries from a motor vehicle accident. Which type of impairment may result from injury to the frontal lobe?

Rational.1537= The frontal lobes make up approximately one-third of the mass of the cerebral hemispheres and are thought to be concerned with the highest cognitive and intellectual functioning, personality, and motor control. Sensation is controlled by the parietal lobes, vision in the occipital lobe, and hearing in the temporal lobes.

Question.1536= A 20-year-old male is admitted to the unit following a suicide attempt after breaking up with his girlfriend. He ingested an unknown drug or drugs and is currently combative but with a reduced level of consciousness. A large-bore nasogastric tube has been inserted in an attempt to lavage his stomach. Which of the following nursing actions should be initiated at this point?

Rational.1536= patients at greatest risk for aspiration during gastric lavage are those who have an altered level of consciousness, are confused, are debilitated, or have impaired gag reflexes. Initiation of mechanical ventilation or sedation are physician actions. Maintenance of an airway takes priority over other possible nursing actions.

Question.1534= Asterixis is regarded as a sign of the developing condition of

Rational.1534= Asterixis is the irregular flapping movements of the fingers and wrists when the hands are outstretched, with the palms down, wrists bent up, and fingers spread. Jerking muscle tremors are also seen in the feet and tongue. Also called liver flap or hepatic tremor, this is the most common and reliable sign that hepatic encephalopathy is developing. Calcium disturbances and cardiac problems are not manifested in asterixis.

Question.1532= A patient with a C-6 fracture would most likely be able to perform which of the following movements?

Rational.1532= Spinal nerves originating in the cervical disks 5 through 8 control the brachial plexus. C-6 function involves good shoulder control, wrist extension, and supinators. Injury above C-5 would have additional impairment, while injury below C-6 would determine how much waist or leg movement was possible.

Question.1531= A 51-year-old female is admitted with hypotension, bradycardia, and decreased level of consciousness. Her core temperature is 35.5degree C. No history is available regarding prior medical problems. She appears to be overweight, with dry, scaly skin and puffy face and lips. Shortly after admission, she has a grand mal seizure. She is intubated and placed on mechanical ventilation. Based on the preceding information, which condition is likely to be developing?

Rational.1531= Myxedema coma is a complication associated with hypothyroidism. Signs and symptoms include hypothermia, seizures, and exacerbations of hypothyroidism. Common CHF symptoms are rales, increased respirations, and tachycardia. ARDS presents with wheezing and tachycardia. Thyroid crisis has increased heart rate and respirations.

Question.1530= A patient has just been admitted to the unit following an accident in which he sustained an injury to the temporal lobe. The nurse`s assessment will focus on disturbances in the patient`s

Rational.1530= The temporal lobes are located on the lateral aspects of the cerebral hemispheres and are primarily concerned with the perception of verbal material: auditory-receptive and hearing. Spatial orientation and taste arise from the parietal lobes. The occipital lobe is associated with vision.

Question.1529= A 48-year-old female is admitted to the unit. Her diagnosis is a possible syncopal episode. She is currently awake, nervous, and anxious. Vital signs are as follows: blood pressure 178/108 pulse 129 respiratory rate 28 temperature 39degree C (102degree F) During the initial examination, the nurse notes that she has exophthalmos and that her skin is warm and wet. Given the preceding information, the nurse will continue assessment for the possible condition of

Rational.1529= Thyroid storm is an extreme exacerbation of severe hyperthyroisism. The metabolic rate increases without regard for any body system. Body temperature will rise from as high as 102degree F (39degree C) to as high as 106degree F (41degree C). Severe tachycardia, hypertension, and tachypnea also are present. Myxedema would present with opposite symptoms.

Question.1525= The nurse is analyzing data collected from a patient in the first 24 hours post acute myocardial infarction (AMI). Which of these data suggests the onset of cardiogenic shock?

Rational.1525= Extremely close monitoring of a post-MI patient`s vital signs is indicated, as hypotension is a cardinal indicator of impending cardiogenic shock.

Question.1522= A patient has a chest catheter connected to a closed water seal drainage system. Which nursing action should be included in the plan of care?

Rational.1522= The documention of the amount, color, and consistency of the drainage every shift by the nurse should be included in the plan of care. Any significant change in the amount, color, or consistency is reported to the physician. Stripping chest tubes is now contraindicated and should not be included in the plan of care. Continuous bubbling in the water seal chamber indicates an air leak and must be evaluated to maintain proper function of the system. The collection chamber must be kept below chest level to function properly.

Question.1521= A patient with a closed head injury should be monitored for signs of increased intracranial pressure. A clinical manifestation of this condition would be

Rational.1521= Increasing systolic blood pressure with widening pulse pressure, bradycardia, and respiratory slowing are part of Cushing`s Triad, a hallmark of increased intracranial pressure

Question.1520= The assessment finding that indicates a flail chest is

Rational.1520= A flail chest usually occurs with nonpenetrating trauma and produces a paradoxical motion of the damaged area so that it moves in a direction opposite to the motion of the uninvolved chest wall.

Question.1519= The nurse evaluates a patient`s arterial blood gases and determines that the patient has a low plasma bicarbonate level. This condition is known as

Rational.1519= Metabolic acidosis occurs when there is either an increase in acids or a deficit in the base (bicarbonate). Acid-base imbalances that are respiratory in nature result from changes in the CO2 levels.

Question.1518= Which of the following common cardiac medications should the nurse anticipate administering to a patient with a sudden onset of extreme bradycardia?

Rational.1518= Atropine decreases AV conduction time, thus achieving the desired outcomes of increasing both heart rate and cardiac output in this patient. Lidocaine is indicated in the management of ventricular dysrhythmias.

Question.1511= A patient has just been diagnosed with acute pancreatitis. Hypocalcemia is often present in acute pancreatitis. Which of the following lab findings is associated with hypocalcemia?

Rational.1511= The calcium level is regulated by parathyroid hormone, which maintains an inverse relationship between calcium and phosphorous.

Question.1503= Which of the following ostomies would most need to be irrigated?

Rational.1503= The sigmoid colostomy is most likely to need irrigation, because the stool is similar to the consistancy of normal stool. The other ostomy types have less formed stool.

Question.1498= When assisting a patient from a bed to a chair for the first time postoperatively, the nurse would make sure to place the chair

Rational.1498= Placing the chair at the head of the bed, facing toward the patient`s feet is a good position for the patient to transfer to and still be able to observe the activities within the room.,

Question.1496= When planning meals with a patient who is hemodialyzed three times per week, which of the following food selections would indicate the patient understands the dietary guidelines?

Rational.1496= Apple juice is low in phosphorus, sodium, potassium, and protein. The patient in renal failure must avoid these nutrients, because the damaged kidney cannot clear them. Bananas are high in potassium. Red meat contains protein and phosphorus. Legumes contain phosphorus.

Question.1483= The nurse practice act is an example of which of the following types of law?

Rational.1483= Individual state legislative bodies enact specific laws for professional practice. Each state has its own nurse practice act.

Question.1474= The proper position in which to place a patient immediately postop until her gag reflex returns is the

Rational.1474= If the gag reflex has not been regained, place the patient on her side in order to prevent aspiration until she is able to swallow. The prone position would allow secretions to pool in the airway, predisposing the patient to aspiration.

Question.1472= Which of the following signs would indicate that a patient is having a severe reaction to the contrast material administered in an intravenous pyleogram?

Rational.1472= Severe respiratory signs of reaction to the contrast material include laryngospasm, cyanosis, laryngeal edema, and apnea.

Question.1471= An elderly gentlemman patient has a diagnosis of occlusive arterial disease. When instructing the patient regarding proper positioning in bed, the nurse would advise him to

Rational.1471= Allowing the patient to sleep in a somewhat dependent position may help prevent pain at rest. Raising the legs would decrease arterial perfusion and cause more vascular problems.

Question.1467= When assessing the lower extremities of a patient with arterial insufficiency, the nurse notices that they are reddish-blue in color below the knees. This observation will be recorded as

Rational.1467= Rubor is a reddish-blue color caused by superficial capillaries that remain dilated after injury. Cyanosis is a blue-gray color, pallor is a white color, and striae are lines that are red, then a silvery tone over time.

Question.1465= A female patient, age 57, is being evaluated in the GU clinic. Her assessment reveals paradoxical incontinence, which can be described as

Rational.1465= Overflow or paradoxical incontinence occurs with the leaking of small amounts of urine from a distended bladder in the absence of effective contractions. Urinary hesitancy is not associated with paradoxical incontinence. A decrease in the stream is associated with benign prostate hypertrophy and frequent urination is associated with an infection.

Question.1463= A patient diagnosed with congestive heart failure (CHF) is noted to have gained 4 1/2 pounds in the past 24 hours. When considering the patient is retaining fluid, the nurse would note this weight gain is consistent with

Rational.1463= The amount of fluid retained can be determined by the weight gain. The accepted ratio, one liter, is equivalent to one kilogram or 2.2 pounds. This patient had gained 4 1/2 pounds, or roughly two liters of fluid.

Question.1454= To assist the left CVA patient out of bed, the nurse would place the wheelchair

Answer1.1454= Facing the patient`s left side.

Question.1451= The proper temperature for administration of a tepid sponge bath is

90 degrees F.

Question.1447= A 77-year-old woman suffered a left-sided CVA, resulting in a right hemiparesis. To preserve her skin integrity, the nursing care plan would include

Rational.1447= The need to encourage high-protein foods is essential for proper maintenance of skin. A lack of protein leads to promotion of skin breakdown and a lack of healing ability. The position should be changed at least every two hours. The pressure areas should be massaged with each turn, at least every two hours. The daily shower would not be recommended for a person of this age because it would tend to dry skin, leading to potential cracking and breaking.

Question.1444= The nurse explains to the patient receiving pertoneal dialysis that when the dialysate is removed after the initial peritoneal dialysis exchange, it often appears

Rational.1444= The dialysate should be clear and pale yellow, although after the first few initial exchanges it may appear bloody or pink tinged. Any cloudiness or green dialysate may indicate infection.

Question.1443= To eliminate hyperglycemia in the patient who has diabetes and is on peritoneal dialysis, the nurse would

Rational.1443= Diabetics may become hyperglycemic from the dextrose in the dialysate. Insulin can be added to the dialysate, thereby eliminating the need to give the patient subcutaneous insulin when performing dialysis.The dialysate solution must contain dextrose (sugar) in order to create an osmotic gradient. Blood sugar would be tested as symptoms appeared.

Question.1439= When assessing an adult`s ECG, which of the following findings would the nurse recognize as being consistent with an MI?

Rational.1439= The area immediately surrounding the infarcted areas has injured cells and is called the zone of injury it produces ST segment elevations. An increase in the PR interval indicates a conduction delay between the SA and AV nodes. A widened QRS complex indicates a conduction delay in the ventricles and may be indicative of ventricular dysrhythmias. U waves are associated with a disturbance in the electrolyte potassium.

Question.1433= A patient with Type 1 Diabetes Mellitus is given a stat dose of insulin because his blood sugar is 525 mg/dL. After receiving the insulin, the patient begins to demonstrate dysrhythmias on the ECG monitor. Which of the following disorders would the nurse consider to be the MOST likely cause of this cardiac irregularity?

Rational.1433= The administration of regular insulin tends to drive the potassium from the extracellular fluid into the intracellular fluid, causing a loss of serum potassium. Decreased serum potassium can cause serious dysrhythmias.

Question.1432= The nurse is completing an admission assessment on a 67-year-old male patient who has a long-standing history of emphysema. Upon percussion of the lungs, the nurse expects to hear

Rational.1432= Hyperresonance is a very loud, low-pitched, boomlike sound produced in air-filled spaces, such as emphysematous lungs. Dullness is a medium-pitched, thudlike sound heard over a solid structure, such as an organ. Resonance is a loud, low-pitched, hollow sound normally heard over healthy lungs. Tympany is a loud, high-pitched, drumlike sound normally heard over a gastric air bubble.

Question.1429= The staff is developing a care plan for a patient with hypercalcemia. An action the nursing care plan will include to improve urinary problems is to

Rational.1429= Increased fluids will decrease the incidence of infection and stones both are less likely to develop if the urine is dilute.

Question.1428= Which of the following diets is most likely to be recommended for a patient experiencing hypercalcemia?

Rational.1428= An acid-ash diet will decrease the risk of urinary tract infection and renal stone development because it helps acidify urine.

Question.1424= Which of the following signs would the nurse assess for in a patient with a suspected diagnosis of Lyme disease?

Rational.1424= Early signs and symptoms of this disorder may include distinctive flat or raised red lesions, lesions that are hot to the touch, fever, malaise, fatigue, headache, chills, nausea, and lymph node enlargements.

Question.1422= Confabulating and lack of voice control in a patient with a CVA can be attributed to

Rational.1422= A right-hemisphere-damaged CVA patient lacks voice control, talks incessantly, confabulates, and reads aloud fluently without comprehension.

Question.1415= The nurse in the PACU notes a change in the patient`s respiratory rate from 18 to 36 respirations per minute. The nurse`s first action would be to

Rational.1415= If the patient has any difficulty in breathing or exhibits a change in the pattern of breathing, notify the anesthesiologist immediately.

Question.1414= A patient has just been informed that she will be started on a CAPD protocol. She wants to know more about CAPD and asks the nurse how many exchanges she would have to perform per day. The nurse explains that

Rational.1414= CAPD usually involves 3 to 4 exchanges per day of 2 liters each in a 24-hour period, with dwell times of 4 to 6 hours each. Most patients who need dialysis can use CAPD.

Question.1411= The wife of a patient on dialysis asks the nurse how chronic renal failure causes anemia. The nurse explains that

Rational.1411= Anemia in CRF occurs as a result of decreasing erythropoietin and because the uremic environment decreases the life span of the RBC.

Question.1406= A patient has undergone a transspheniodal hypophysectomy. Procedures contraindicated in the immediate postoperative period include

Rational.1406= Frequent mouth care should be given, but brushing teeth may damage the gingival suture line. Elevating the head of the bed promotes an open airway. Advancing the diet and monitoring for infection are routine nursing responsibilities.

Question.1403= The nurse is teaching the wife of a patient with Parkinson`s disease. She explains that patients with Parkinson`s have difficulty slowing down once they walk forward, and in fact, they actually speed up. This problem is described as a

Rational.1403= Propulsive gaits are usually exhibited in later disease progression. The patient`s steps become faster and faster and shorter and shorter.

Question.1397= A male patient has been admitted for treatment of end-stage renal disease. The nurse understands that the primary cause of renal failure is

Rational.1397= Diabetes mellitus is the most frequent cause of renal failure, followed by hypertension.

Question.1396= The nurse plans to prevent complications in the diuretic stage of a patient`s renal failure. Information that the nurse will use in developing this plan includes recognizing that

Rational.1396= Although renal healing is occurring during this stage, the tubule system is frequently unable to concentrate urine satisfactorily. Thus, patients may excrete large volumes of urine (but not 25 to 30 liters in a day) and lose substantial amounts of electrolytes.

Question.1395= Which of the following disorders would be of greatest concern for a patient in the oliguric stage of acute renal failure?

Rational.1395= Acute uremia results in fluid retention and electrolyte abnormalities, particularly hyperkalemia.

Question.1392= Which of the following would the nurse expect to find on the urinalysis report of a patient who has a diagnosis of hyperparathyroidism?

Rational.1392= As a result of hypercalciuria, hyperphosphaturia develops and altered renal tubule function causes unusually alkaline urine. Ketones and glucose are associated with diabetes mellitus. RBCs are associated with infection or trauma.

Question.1390= A female patient, age 49, has been admitted to the hospital with an exacerbation of rheumatoid arthritis. The nurse understands that this disease primarily affects adults between 20 and 55 years old and the incidence is

Rational.1390= Rheumatoid arthritis affects women three times as often as men. It is an autoimmune disease that is not genetically linked and is not as common in children as in adults.

Question.1388= When percussing a patient`s abdomen, the nurse knows that fluid will be revealed as a

Rational.1388= Fluid in the abdomen will pool in the flanks and will percuss with a note that is flat or dull. Tympany or hyperresonant sounds occur over air. A bruit is a swirling sound.

Question.1387= A patient is diagnosed with a left-sided heart failure. One major goal in the management of congestive heart failure is

Rational.1387= Afterload is a major concern in CHF. Ways to reduce it include the administration of vasodilating agents. These agents decrease peripheral vascular resistance (afterload) and increase cardiac output by reducing resistance. Contractility is thereby increased. Relief of anxiety is not an immediate goal of treatment.

Question.1386= To assess the peripheral pulses of a patient`s lower extremities, which of the following areas would the nurse palpate?

Answer1.1386= The popliteal, dorsalis pedis, and posterior tibial.

Question.1385= To confirm the diagnosis of left-sided CHF, the patient will undergo a cardiac catheterization. Which of the following preprocedure orders would the nurse question?

Rational.1385= Sodium should be restricted. A reduction of sodium will decrease the retention of salt and water and therefore reduce vascular volume.

Question.1378= Which of the following interventions may cause injury in a patient with rheumatoid arthritis experiencing a flare-up (exacerbation) of the disease?

Rational.1378= During periods of exacerbation, or acute inflammation, active range of motion (especially to the point of pain) can cause significant joint damage. Consequently, range of motion should be done by the nurse but not to pain or beyond. Splinting of joints and medication are useful in treating exacerabations of RA.

Question.1374= The aphasia seen in CVA patients is usually caused by damage to the

Rational.1374= Aphasia, both expressive and receptive, is seen in patients who have sustained damage to the left cerebral hemisphere. Damage to the cerebellum usually results in motor deficits.

Question.1371= A 34-year-old female is in the critical care unit with acute respiratory distress secondary to sepsis following a motor vehicle accident. She currently has a chest tube in place, set at 20 cm H2O suction. There is no bubbling in the water seal although there is bubbling in the suction control chamber. While the nurse was at lunch, one of the unit technicians says he clamped the chest tube to determine if the suction level was still at 20 cm H2O. It is still clamped when the nurse goes into the room. The patient does not complain of any symptom change. About the same time, the patient`s physician comes into the room and notices the clamped tube. He becomes very upset and says to get an immediate chest x-ray to see if a tension pneumothorax has occurred. The nursing action is to

Rational.1371= Fluctuations in the water seal show that there is patency between the pleural cavity and drainage bottle. Fluctuations will stop if lung expansion has occurred or there is kinking of the tube or clots present. Listening to the lungs would indicate if expansion has occurred, in which case no x-ray will be necessary.

Question.1366= When evaluating the effectiveness of traction, the finding that would counteract this is

Rational.1366= For traction to be successful, countertraction must be present. Weights resting on the bed would not allow for traction. Force and direction must be maintained with the ropes in the center track of the pulley.

Question.1365= When assessing the breath of a patient with advanced liver disease, the nurse notes an ammonia-like odor. This most often represents

Rational.1365= Fruity or acetone odor is indicative of ketosis, whereas an ammonia smell is associated with uremia.

Question.1361= A young patient is diagnosed with mitral stenosis. When assessing the patient, which symptoms are consistent with this condition?

Rational.1361= Mitral stenosis refers to the obstruction of the mitral orifice due to adhesions between the two mitral valve leaflets. The first symptom observed is exertional dyspnea.

Question.1358= Which of the following assessments would indicate possible neurovascular compromise in a casted extremity?

Rational.1358= Neurovascular assessment of a casted extremity involves color, sensation, and movement. Tingling would indicate a problem. Normal assessment would expect that skin is warm and pink with equally palpable pulses. Mild pain on passive motion is expected.

Question.1357= A female, age 45, is admitted to a cardiac medical unit for preoperative care prior to having a possible valve commissurotomy. Pertinent past history includes her having had a case of rheumatic fever when she was 12. She is married and has two children aged 17 and 13. The patient seems very anxious and asks the nurse, "How could a sore throat I had 30 years ago damage my heart now?" The best reply by the nurse is

Rational.1357= Rheumatic fever is a systemic, inflammatory, nonsuppurative disorder that usually occurs as a sequela to a throat infection caused by group A beta-hemoloytic streptococci. It is characterized by a diffuse proliferative and exudated inflammatory reaction in connective tissues, particularly of the heart, joints, and skin.

Question.1353= A patient diagnosed with a hypothalamic disorder is being admitted. In her assessment, the nurse will question the patient about disturbances in

Rational.1353= The hormones of the hypothalmus control many important functions. One of the major functions is temperature control

Question.1352= The nurse is caring for a patient undergoing peritoneal dialysis and notes that the dialysate has not drained. To help facilitate drainage, she will

Rational.1352= Draining problems may occur in peritoneal dialysis due to obstruction of the catheter by the omentum. If this occurs, turning the patient from side to side will change the position of the catheter.

Question.1347= The nurse continues teaching the patient with AIDS that the agent used to treat pneumocystis carinii pneumonia is

Septra (Bactrim).

Question.1346= The AIDS clinic nurse is teaching a patient about his lab profile. She explains that the cell known as the "helper cell," vital in activating the immune response, is the

Rational.1346= T4 lymphocytes, known as "helper cells," are vital in activating the immune response. T8 lymphocytes are suppressor cells and neutrophils are leukocytes.

Question.1344= The nurse is checking the patient`s pleural drainage system. She recognizes indication of an active pleural leak when she sees

Rational.1344= Constant bubbling in the water seal indicates leakage of air in the drainage system fluctuation is normal with respiration. Bubbling is expected in the suction control chamber.

Question.1338= In the patient with smoke inhalation, the most likely time period during which pulmonary complications may develop is the

Rational.1338= Any patient with smoke inhalation must be observed for at least 24 hours for respiratory complications. It may take a full 24 hours for complications to develop.

Question.1334= A 19-year-old African-American woman presents with a nontender, unilateral breast lump in the right breast, which she noticed on a breast self-exam. Her last menstrual period was normal and ended 2 days ago. She denies pain, nipple discharge, or recent trauma. The most likely diagnosis is fibrocystic breast changes versus

Rational.1334= A mass in an adolescent is almost pathognomonic for fibroadenoma, especially in African-Americans. If breast symptoms are unilateral or there is a palpable mass, refer the patient for further evaluation. Women with fibrocystic changes usually present with bilateral nodularity and increased tenderness or pain prior to menses. The nodularity may be generalized or localized in the upper outer region.

Question.1325= A pregnant woman who is not immune to rubella should be vaccinated

Rational.1325= To reduce the risk of acquiring rubella at the beginning of a future pregnancy, the woman should receive the vaccine after she gives birth. Caution her to avoid becoming pregnant for at least 3 months after receiving the vaccine.

Question.1324= The nursing plan for an infant diagnosed with necrotizing enterocolitis (INEC) should include

Rational.1324= Measuring the abdominal girth provides information on whether or not the gastric distention is resolving.

Question.1323= At what age do term infants usually regain their birth weight after the initial weight loss?

10 days.

Question.1320= When a newborn cries, the right side of his face remains motionless. When he sleeps, his right eyelid remains open. The nurse should interpret this as a

Rational.1320= A facial nerve paralysis results from pressure on the facial nerve during labor or from the use of forceps for delivery. The paralysis is usually unilateral and most apparent when the newborn cries. If the nerve has been injured, the paralysis disappears within a few days or weeks. It may take as long as several months. If the nerve has been torn, it requires neuroplasty. Facial nerve paralysis is an alteration in expected findings of the newborn assessment. It is neither a congenital anomaly nor a genetic defect.

Question.1319= In order to prevent the need for a backup method of contraception, the nurse instructs the patient starting oral contraceptives to take the first pill

Rational.1319= No backup contraceptive method is needed if the OCs are begun on the first day of menses since ovulation is inhibited for that cycle. The pill works primarily by suppressing ovulation (via suppressing release of gonadotropin-releasing hormone [GnRH], which inhibits the pituitary release of FSH and LH), causing thick cervical mucus (which makes it difficult for sperm to penetrate), and making endometrium or lining of the uterus unfavorable for implantation.

Question.1318= In addition to the current pregnancy Sandra has been pregnant three times. She lost the first pregnancy at 10 weeks; the second pregnancy produced twin girls who were born prematurely; and the last pregnancy was carried to term and her son was born at 39 weeks. All of the children are alive and well. Which of the following numeric scores accurately depicts Sandra`s reproductive history?

Rational.1318= Assessment of pregnancy history is accomplished using the pneumonic GTPAL G (gravida) T (term pregnancy) P (preterm birth) A (abortion) L (number of living children). Since Sandra has been pregnant 4 times, she is gravida 4 . She delivered her last pregnancy at term, making 1 term pregnancy. She had 1 birth (delivery) of twins, making preterm (P) births equal to 1. Her first pregnancy was a miscarriage giving her 1 (spontaneous) abortion. All of her children are currently living (twin girls and a son). In addition, some pneumonics add "M" (multiple gestations) making the pneumonic GTPALM, in which case Sandra would be 4-1-1-1-3-1.

Question.1316= A patient who is 8 weeks pregnant tells the nurse that her gums have been bleeding with every tooth brushing. The nursing intervention is to

Rational.1316= Pregnancy gingivitis may result from proliferation of localized blood vessels and softening of the gums. A focal hypertrophy of the gums, epulis, may present with erythema, swelling, and bleeding when traumatized as in teeth brushing. The teeth are not affected and the hypertrophy resolves after pregnancy as it is most likely related to elevated estrogen levels. However, the frequency of bleeding early in pregnancy requires further assessment before considering this physiologic cause.

Question.1315= Which of the following are probable signs of pregnancy that a patient might experience at some point during her gestation?

Rational.1315= Probable signs, or objective signs, include those which can be visualized on examination (Chadwick`s sign, uterine enlargements, ballottement). A positive pregnancy test suggests pregnancy by testing for the presence of HCG. However, the similarity between HCG and other hormones (such as LH) may result in cross-reactions resulting in a positive pregnancy test. Therefore, it is considered as a probable sign. Presumptive signs are subjective signs (including "morning sickness," amenorrhea, breast tenderness, and quickening). Audible fetal heart tones, palpated fetal movements, and visualization of the fetus on ultrasound are all positive signs of pregnancy.

Question.1314= A nurse determines that a patient has several presumptive indicators of pregnancy. They are called "presumptive" because they

Rational.1314= Presumptive signs of pregnancy such as amenorrhea, breast tenderness, nausea, fatigue, and urinary frequency, are subjective and are not diagnostic, only suggestive of pregnancy.

Question.1313= By the end of 12 weeks the pregnant uterus should be palpable

Rational.1313= The ascent of the uterus from the pelvis into the abdomen occurs at approximately 12 weeks. The uterus cannot be palpated when it is below the level of the symphysis pubis.

Question.1311= A patient who is 12 weeks pregnant reports that her mouth frequently feels like she has too much saliva. This condition is known as

Rational.1311= Ptyalism is the production of excessive saliva during pregnancy, characteristically noted during the first trimester. Oral hyperemia (redness) and gingivitis (gum inflammation) may occur also. Hyperemesis is persistent vomiting during pregnancy, which results in dehydration.

Question.1307= A vaginal exam at eight weeks of pregnancy would reveal which of the following signs of pregnancy?

Rational.1307= In the early weeks of pregnancy, Goodell`s sign (softening of the cervix) and Chadwick`s sign (a bluish discoloration of the cervix) both occur. Ballottement occurs in midpregnancy. The mucus glands form a mucus plug, but there is also an increase in white vaginal secretions. However, in addition the mucus secretions are acidic.

Question.1303= Shortly after entering the room of a patient with PIH, the nurse observes facial twitching followed by a tonic contraction of the patient`s entire body. The nursing intervention is to

Rational.1303= Eclampsia is marked by the occurrence of tonic-clonic seizures or vascular collapse. In the event of seizures, the following signs are observed: a prodromal facial twitching lasting only a few seconds, a tonic contraction of the entire body lasting about 20 seconds, and a convulsion lasting about a minute. Protecting the patient from injury is the priority intervention.

Question.1302= A pregnant woman is admitted to the ER after having a convulsion. The diagnosis of eclampsia is made based on the presence of

Rational.1302= Eclampsia is marked by hypertension characterized by the occurrence of tonic-clonic seizures or vascular collapse.

Question.1301= A patient with mild pre-eclampsia should be encouraged to

Rational.1301= Left lateral position decreases pressure on the vena cava and is believed to improve venous return and placental and renal perfusion.

Question.1300= A 2-week-old newborn is brought to the hospital clinic by her mother with a complaint of continuous regurgitation. A barium swallow reveals a diagnosis of gastroesophageal reflux. The nurse will instruct the mother to

Rational.1300= Prone positioning has been shown to decrease reflux and facilitate gastric emptying. Some textbooks recommend a prone position with the head of the bed elevated. Either position is currently accepted therapy.

Question.1299= The nurse is caring for a 2-week-old infant with a ventricular septal defect. In planning interventions for the nursing diagnosis "alteration in nutrition: less than body requirements related to fatigue," the nurse will suggest feeding

Rational.1299= A soft nipple helps the infant suck more easily, decreasing the energy expenditure.

Question.1297= A clinical manifestation of increased intracranial pressure in infants may be observed as

Rational.1297= The early signs and symptoms of increased ICP are often subtle and assume many patterns, such as irritability and changes in feeding.

Question.1296= Infants triple their birth weight at age ___________________ months.

Rational.1296= The answer if 12 months. During early infancy, the energy requirements for this period of rapid growth are high. By 1 year of age, infants weigh approximately 3 times their birth weight.

Question.1295= Assessing an average 2-month-old child, the nurse knows that he should be able to

Rational.1295= Although the acquisition of gross motor skills varies in its timing, infants at 2 months of age can lift their heads up to a 45 degree angle when prone.

Question.1294= Recommended treatment of children with sickle cell anemia includes which of the following?

Rational.1294= Poor splenic function predisposes the child with sickle cell disease to overwhelming infection from S. pneumoniae and H. influenzae. Therefore, recommendations for management include oral doses of prophylactic penicillin to protect from infection.

Question.1293= A 2-year-old African-American child with sickle cell anemia has his immunizations up to date. Now that he is two, which of the following vaccines should be initiated?

Rational.1293= The pneumococcal vaccine should be administered to all children with sickle cell anemia at 2 years of age. These children run a high risk of developing pneumococcal septicemia or meningitis.

Question.1292= Which of the following is the most common serious complication of sickle cell anemia in children?

Rational.1292= The most common serious complication of children with sickle cell anemia is overwhelming septicemia and meningitis due to Streptococcus pneumoniae and Haemophilus influenzae type B.

Question.1284= A child who is in a chronic state of hypoxemia would have a physiologic response of

Rational.1284= In chronic hypoxemia, the body compensates by producing more red blood cells (polycythemia). By producing more red blood cells, the oxygen-carrying capacity of the blood is, in theory, increased. This is only true if adequate amounts of iron are available for the formation of hemoglobin.

Question.1283= A 2 month old has hydrocephalis. He has been admitted for insertion of a ventriculoperitoneal shunt. After surgery the nurse observes that the baby`s anterior fontanel is elevated. The nursing intervention is to

Rational.1283= An early sign of increased intracranial pressure in an infant is a bulging fontanel when at rest. This is a serious sign after a shunt operation because it indicates that there is an obstruction to the flow of cerebrospinal fluid, and corrective surgery may be required.

Question.1282= Which of the following is not a common clinical presentation of AIDS in children?

KS

Question.1280= The first sign or symptom of an acyanotic heart defect is often

murmur

Question.1278= A 32-year-old multipara is pregnant with her third child. She has been in labor for 20 hours, and the fetal position is LOP. The patient is likely to experience

Rational.1278= Labor with a fetus in an occiput posterior position is long and difficult. Backache is common as the occiput bone presses against the mother`s sacrum.

Question.1272= A primipara, aged 24 and 39 weeks` gestation, phones the clinic and reports that she is having contractions every 15 minutes. The nurse will help her distinguish the difference between early and false labor by explaining that it is usually false labor when the patient

Rational.1272= Braxton-Hicks contractions, or false labor, are mild, intermittent, painless contractions which occur throughout the second half of pregnancy. Women often obtain relief from them by walking or lying down.

Question.1271= A 25-year-old primipara is in labor, having strong contractions occurring every 2 minutes and lasting 60-90 seconds. She feels frustrated, starts to tremble, and is afraid she is losing control. The nurse explains to her that she is moving into the

Rational.1271= The answer is the transition phase these are characteristic findings of the final or transition phase of the first stage of labor.

Question.1270= A pregnant patient, gravida 2, para 1, is admitted for generalized edema, proteinuria of +2, and an elevated blood pressure. She is started on magnesium sulfate and is receiving 2 grams/hour IV. Preparing to mix the next liter of medication, the nurse assesses her, finding absent reflexes and a respiratory rate less than 10 breaths per minute. The first nursing action is to

Rational.1270= Absent reflexes and a respiratory rate of 10 per minute or less indicate magnesium toxicity and the antidote (calcium gluconate) should be in the patient`s room ready for administration. The physician should also be notified at once.

Question.1269= When caring for a patient with cardiac disease who has just entered the second stage of labor, the nurse understands that the management of the patient will likely include

Rational.1269= The goal of management with this patient is to shorten the second stage of labor. Appropriate management could include 1) spinal anesthesia, 2) episiotomy, and 3) forceps delivery.

Question.1268= A priority goal for a patient experiencing an incomplete abortion is

maintain vs

Question.1266= A pregnant woman is admitted to the ER. She states that she is 16 weeks pregnant and has been bleeding vaginally. The nurse would differentiate between a threatened abortion and an inevitable abortion when the assessment findings include the

Rational.1266= A threatened abortion is suggested when a woman experiences vaginal spotting or bleeding early in pregnancy and no cervical dilation or effacement is present. An inevitable abortion, which cannot be prevented, is indicated when the cervix has begun to dilate, uterine contractions are uncomfortable, and vaginal bleeding increases.

Question.1254= The nurse/midwife performs a procedure in the last trimester to determine fetal position, lie, and presentation. The nurse would describe this procedure as

leopold Fetal lie, presentation, position, and engagement can be determined by abdominal palpation of the mother.

Question.1253= Samantha has been taking an oral contraceptive for 3 years. Since a recent head injury, she is also taking phenytoin (Dilantin) 100 mg three times a day for seizure control. What information would the nurse include in the patient teaching?

Rational.1253= Phenytoin can decrease the effectiveness of oral contraceptives. The oral contraceptive should be supplemented with other means of birth control.

Question.1249= The nurse in the prenatal clinic is teaching a goup of pregnant women about self-care. Instructions will include that when lying down, it is best to lie on the left side in order to

Rational.1249= The growing uterus may press on the inferior vena cava when the pregnant woman is supine. This reduces blood flow to the inferior vena cava when the pregnant woman is supine, and also reduces blood flow to the right atrium. This may lower the blood pressure, causing dizziness.

Question.1244= An elderly patient with pedal edema is admitted to a medical floor with symptoms of vascular disease. The nurse caring for this patient is aware that edema may result from

Rational.1244= Edema can result from increased capillary pressure or from the passage of protein molecules and fluid from blood into interstitial fluid because of increased capillary permeability. An aneurysm is a localized dilatation of an artery caused by a weakened arterial wall ). Third spacing occurs when fluid is filtered from the plasma and then accumulates in other areas of the body (4).

Question.1241= A patient with cervical cancer enters the hospital to have a cesium implant. Before the insertion of the implant, the nurse tells the patient what to expect while it is in place. Which of the following statements is accurate?

Rational.1241= It is necessary for staff, family and friends, to have decreased exposure to the source of radiation. Nursing care is to be delivered in a timely fashion, with special attention given to physical distance. The patient receiving any type of radiation should not be in any pain as a result of the treatment (1). Bed linens need to be changed as needed, not because of the radioactive implant (3). patients with radioactive implants should be able to at least assist with activities of daily living. patients are encouraged to provide self-care measures to decrease the possibility of complication due to immobility (4).

Question.1239= The nurse caring for a patient with polycythemia vera notes all of the following clinical signs and symptoms. Which of these should be reported to the physician immediately?

Rational.1239= This is a classic clinical manifestation of thrombophlebitis, a serious complication of this blood disorder.

Question.1217= The physician ordered an enema for a patient to relieve gastric distention. The nurse would expect the type of enema ordered would be a (an)

Carminative enema. Rational.1217= This enema is made up of a solution which is 30 mL of magnesium, 60 mL of glycerin, and 90 mL of water. A physiologic normal saline enema is safest for infants and children because of their predisposition to fluid imbalance. A soap suds enema is most frequently used for cleansing the bowel prior to surgery. An oil retention enema is an oil-based solution. It permits administration of a small volume, which is absorbed by the stool. The absorption of the oil softens the stool for easier evacuation.

Question.1211= When teaching a patient to use a walker, the nurse would instruct the patient to move the walker

Rational.1211= The proper distance is 6 inches so that the individual maximizes the safe and effective use of the walker.

coumadin s/e

orange urine

50 GRAMS OF DEXTROSE

4 CALS PER GRAM

lactating mothers

Restricted fat intake. Lactating mothers need 4 to 5 glasses of milk a day. They should never be advised to restrict any nutrient or attempt to diet during lactation.

The nurse can instruct a diabetic patient that blood glucose levels can be moderated by including more

Dietary fiber decreases the absorption of carbohydrates and may affect the release of gastrointestinal hormones, which influence insulin and glucagon secretion. Fiber also increases and prolongs satiety and helps control blood lipids. Omega 3 fatty acids appear to decrease serum triglycerides, but also tend to increase fasting plasma glucose levels.

A male client has just returned to the unit after he had an exploratory laparotomy. His orders include applying a binder to minimize muscular tension over his incision site. The binder that will be used on this client is a

The straight abdominal binder centers support over abdominal structures. It provides continuous wound support and comfort. A stretch net binder is used for support of dressings or surgical sites over the client`s arms or legs. The T binder is applied to facilitate placement of perineal dressings and provide support to perineal muscles and organs. A double T binder is used to support perineal dressings for male patients.

When obtaining a urine sample from a catheter, the period of time for clamping the catheter is

The urinary catheter should be clamped off for short periods of time only, in order to obtain a small sample of urine. Longer periods of time may cause the urine to back up into the bladder and increase the potential for UTIs. Thirty minutes is sufficient to obtain a specimen in a client with a normal urine output.

The normal potassium level is 3.5-5.0 mEq/L. The patient`s potassium level is low, and he needs to replenish what has been lost as a result of taking the Lasix. In addition to taking potassium supplements, the patient should be given a list of the appropriate foods that have an average of 7 mEq potassium per serving. (Fruit, meat, fish, instant coffee, and milk are high in potassium.

electrolytes

normal electrolyte values for an adult are as follows: sodium of 135-145 mEq/L, chloride of 100-106 mEq/L, potassium of 3.5-5.0 mEq/L, and bicarbonate of 22-29 mEq/L.

sed rate

This is a normal sed rate for a female over age 60. Under age 50, normal is 20 mm/hr. If it were increased, it would indicate presence of infection or inflammation, and surgery might have to be postponed.

PT abnormal

Because the patient is not on anticoagulant therapy, the results are abnormal (normal PT is 11-15 seconds). It is important to notify the head nurse or physician before the biopsy; bleeding could be life threatening. The patient will probably be given vitamin K therapy and when the PT results return to the normal range, the procedure can be done. Liver disease likely caused the prolonged PT.

lipid agents

to the sun or photosensitization is a risk for patients taking HMG-COA reductase inhibitors (Mevacor). (1) Niacin is usually given with bile acid sequestrants because they work synergistically. (2) Bleeding from the gums or rectum is a sign of vitamin K deficiency from bile acid sequestrants (Questran).

creatine

The normal serum creatinine level for a male is 0.6-0.9 mg/dL. A patient with a mild degree of renal insufficiency would have a slightly elevated level, which in this case would be 1.7. Levels of 3.3 (2) and 4.0 (1) may be associated with acute or chronic renal failure.

labs for pancreatitis

These elevated serum levels (amylase and lipase) are the hallmark of acute pancreatitis. Increased white blood cell count and serum bilirubin level is also seen with acute pancreatitis. Elevated alkaline phosphatase (4) is found in chronic pancreatitis.

pyloric stenosis distinctive signs

peristalic waves

quickening

17-19 weeks first felt

thrombocytopenia

platlet count 50k or below. Sx petechie,purpura and hematuria

Dornase alfa

reduces viscosity of sputum with CF

blurred vision

advanced indicatior of PIH

suction

mouth then nose of infant

menstrual cycyle regulated

FSH and LH which are produced in the hypothalmus

epidural

vasodilation and blood pooling in extremites. Can lead to hypotension so monitir BP. Immediate TX elevate legs.

Bucks extension

following a hip Fx is to immobalize to relieve muscle spasm at the fx site

pacemaker malfunction

weakness and fatigue R/T hypoxia of the tissues

LVN can't

take admissions

Denver shunt

used for pts with ascites and cirrhosis. diverts fluid from the abd to jugular or vena cava

shunt

should feel warm, hear bruit and feel thrill

chronic hypoxia

R/T copd may stimmulate excessive RBC formation will increase blood viscosity and risk for thrombus

gentamycin

aminoglycide nephrotox!

B12

comes from animal products increase or get b12 shot R/T pernicous anemia

mechanical vent

may cause stress ulcers so maintain Ph above 5

lactalose

decreases NH3 levels with pt s who have hepatic coma

positioning for pancritits

sit up or lean foward to decrease pain

laproscopic shoulder pain

after co2 is injected into the abd main cause shoulder pain after surgery

colchine (for gout) sx of toxicity

diarrhea

aminoglycides

more active when the urine is alkaline so soda bicarb may be given

seconal

barbiturate used for sleep. Monitor for sedation

spironolactone

decrease urine na then causes diuresis

hypertonic fluids

higher in osmotic pressure used for icp because it reduces idema

thiazide diuretics

monitor for k loss monitor muscle weakness leg cramps hypotension

start solids

5-6 months

6 months

sits with minimal support

sterile technique

when skin is broken or STERILE body cavity is entered

surgical asepsis

cath to bladder

after surgery

immediatly assess pulmonary function first thing post op

critical pathway

managment tools developed for particular types of patients . they have interventions outcomes and progress. they are multidisaplinary.

tens

blocks painful stimuli from traveling over small muscle fibers.

osteoarthritis assememt findings

joint pain, crepitus, heberdens nodes

at a 4 y/o wc visit the kid is really small. what is the nurses first action in evaluating his growth

compare with siblings

increased vag drainage

nl during pg

soap enema

nurse should hold about 12 inches above rectum. admin slowly. insert the tube 3-4 inches in rectum, direct the tip towards the umblicus

hospice

inhances the quality of life for terminal ill pt. helps pt and fam to live life to the fullest each day

lasix

because lasix is k wasting the pt needs to eat k foods such as apricots, dates, and citrus foods

a pt is chocking on his lunch and coughing forcefully what should you do

if he is coughing he should be able to dislodge the object if obstruction occurs then do abd thrusts hemi

intermittent cluadication

chronic perhrial vascular disease that reduces 02 to the feet so do meticulus foot care. Bath feet in warm water and dry throughly cut toe nails straight across, wear well fitting shoes, avoid meds unlesed cleared by M.D, stop smoking not cut back!

rubella iz

12-15 months

if 51/50 and client is feeling better answer is:

continue to monitor for suicide

tetany bactrium

clostrdium

late sign of hypoxia

increased heart rate R/T increased energy demands

fluid overload causes

cerebral edema with a pt with meningitis check for icp too

care plan for client in manic state is

listen attentivly to pt request, express wilingness and seriously consider the request, and respond later.

nurse can anticipate a client in the 2nd trimester to be

increased intorspection and general sence of well being

dislodge infants airway

5 back blows followed by 5 chest thrusts

chest tube working correctly

flux with fluid level while breathing. Bubbling indicates an air leak. constant level indicates an obstruction

on g tube first be concerned with

aspiration

while collecting data on unresponsive pt. first check

responsivness

mao

may cause hypotension so nursing dx is r/f injury

group b strep mom

low temp in neonate = signof infection. Place mom and baby in seprate rooms

mom rh neg baby rh pos

give rhogam in 72 hrs

hemangiooma

vascular tumor in baby

2nd trimester behavior

narcissistic and facinated by children

change position first action. variable decels then they might do amniofusion or c sec if does not help

hypogly in the neonate

lathargic

postpartum perineal discomfort

contract your butt before sitting

cna can

bottle feed a 24 hr neonate. no first bath, no diaper changes after circ, no vitals in transitional period

pre term labor nursing intervention

provide adequate hydration to halt contractions

cocaine an pg

cause increased uterine contractlity, and preterm labor. notify neonatologist about pts arrival

contraction breathing

shallow chest breathing

34 weeks pg and in the er complaning of vag bleed nurses first action is

check fhr and maternal bp

vbac incsion

lower uterine transverse is the only one allowed with vbac

pg chocking

provide chest thrusts

heat loss from evaporation with the neonate

drying throughly after bath can prevent this

false labor contractions

usually occur in the abd irregular and typically relived by walking

mom with gdm baby is hyperglycemic then is hypoglycemic in the first 24 hrs what is a nursing intervention

provide frequent early feeding with formula

high folic acid

egg yolks, nuts, seeds and liver

phototheraphy adverse rxn

watery stools

lochia 2nd day post op

red and scant

embryo third week

heart beats

neonate nl resp

30-60

heprin

does not cross placenta

gave birth yesterday the fundus should be

1 fb below umbilicus

molar pg

use bcp for 1 year

placenta previa

softrelaxed non tender uterus

A client with glaucoma has been prescribed Timoptic (timolol) eyedrops. Timoptic should be used with caution in the client with a history of:

Emphysema eta blockers such as timolol (Timoptic) can cause bronchospasms in the client with chronic obstructive lung disease

A client being treated with sodium warfarin has an INR of 9.0. Which intervention would be most important to include in the nursing care plan?

Assess for signs of abnormal bleeding The normal international normalizing ratio (INR) is 2 - 3. A 9 might indicate spontaneous bleeding.

An elderly client asks the nurse how often he will need to receive immunizations against pneumonia. The nurse should tell the client that she will need an immunization against pneumonia:

Every 5 years Immunization against pneumonia is recommended every 5 years for persons over age 65, as well as for those with a chronic illness.

A 45-year-old client returned from a colon resection 2 hours ago. Which vital signs indicate possible hemorrhagic shock?

BP 96/60, heart rate 120

A client is 2 days post-operative colon resection. After a coughing episode, the client's wound eviscerates. Which nursing action is most appropriate?

Cover the wound with a sterile saline-soaked dressing If the client eviscerates, the abdominal content should be covered with a sterile saline-soaked dressing.

do you have trouble controlling upseting thoughts? gathering data on an ocd pt what is most inportant to ask intrusive thoughts repeat over and over

pt is dnr and has cardiac arrest lvn should

assess for signs of death

inserting an ng tube wear

gloves gown mask and goggles

s/s of fat embolism

confusion, agitation, delirum, coma, increased resp, cp, sob, pallor, htn , paetechiae on chest.

liprium

antianxiety used to tx etoh w/d

walker

pick up move forward 10inches then take a few steps to it!

pt after a cataract removal c/o severe pain what lvn to do next

tell RN

dumping syndrome

decrease carbs

acute renal faliure

decreased urinary output

6-12

industry aspires to be best learns social skills, how to finish tasks, sensative about school.

return and turn off feeding. tube feeding at 100ml hr risdual checked its a 90 what to do? stop feeding if risdual is 50% of the volume infused in 1 hr

pku testing

p a source of protein is injested. pref in the hospital in the first 24 hrs repeat in 3 weeks

preshoolers

need to see and play w/medical equiptment

nl cvp

5-10 cm h20

tx for hyperkalemia

decrease k, iv nacl, kayexaltae ,ekg.

tx hypokalemia

give KCL no more then 20 qhr observe i/o, ekg

hyponatremia 135-145nl

weakness, restless,delirium, confusion

ca 4.3-5.3 hypocalcemoa

cramping, tetany

ca 4.3-5.4 hypercalcemia

a/n/v lathargy

tx of metabolic acidosis

na bicarb, na lactate. watch labs. monitor for hyperkalemia and dehydration.i/o.

metabolic alkalosis tx

high k high cl foods. i/o. Kcl. ammonium cl to increase hydrogen ions. labs, monitor for hypokalemia.

resp acidosis tx

all resp stuff to promote drainage, breathing etc. give na hc03,RL, and k

resp alkolosis tx

s/s hyper reflex, muscle twitching, convulsions. use rebreather. monitor k and hco3

ab compatable w/

all blood groups

bladder irrigation

30-50ml

dvt

bed rest 3-4days no pillow under leg . monitor for PE. antiemboletic stocking to the other leg

fat emboli tx

etoh drip, cortisone therapy, deholin to emulsify, lipid lowering drug

mag toxicity first indication

extreme thirst

3.5-5.5

na

135-145

p mva fat emboli

24-48 hrs p

following removal of ng tube for a pt with pancreatitis diet should be

high in carb

diet after cholectomy

low residule and no dairy r/t the moucus it can cause

central line usually

r subclavian

cl

100-106

bicrb

22-29

high gastric acid indicates

duodeal ulcer

sed rate under age 50

20mm/hr

nl PT

11-15 seconds

HMG/COA

teach photsensativity

elsa

verifies hiv dx

male creatinine

0.6-0.9 elevated with renal insuff

nl bun

10-20

neutrophils

first line defense

most common nosocomial infection

c diff

removing isolation gown

untie front waist strings, remove gloves, untie neck

emergency community wide disaster network who is notified first

local health officer commander

rad exposure

1 hr to radiation exposure = 5o RADS

smallpox contraindicated

cardiac, leukemia, lymphoma, PG, burns, hiv, shingles, eczema

anthrax

standard percautions

pneumonic plague

standard and droplet percautions

dm type 1 bg over 250 then

check for ketones teach the pt this

high na in milk

FYI

phenobarb

excreted by the kidneys so good for hepatic coma

pt starts to shiver during a tempid bath

stop as the body is attempting to produce heat

dicumarol

an anti coag so teach pt about this type of drug

colchacine tox

diarrhea first sign

priority assesment for bacterial endocarditis

emboli

eartquake in the hospital

first thing!!! look for instructions. dont call about pts that need help. weird

kid stong by jellyfish first thing

bath in vinigar and apply shaving cream

pt is has severe reaction to meds initial action

place pt supine in shock position

with heating pad first sign of thermal injury

redness to area

proper depth for infant cpr

1/2 to 1 "

earlisest sx of shock

narrowing pluse pressure

radiation beads have follon out of pt you cant find them

let the rn know

nl ms dose

1mg per//ml or 5mg/ml on PCA

after masectomy position pt

semi fowler with affected arm elevated

radiodermititis

avoid all creams or lotions on the area. wash with leukwarm water and mild soap

early warning ca sign

change in bowl habits

wbc nl

5-10k

neomycin preop bowl surg

to sterilize the bowl

cesium needle in cervix

bed rest until it is removed

malignant melonoma

worst prognosis

alkylating chemo drugs work by damaging dna in the cell nuclus

fyi

after abortion pt develops dic what is the most critical intervention

administer ordered meds (hep)

nst is reactive means that

good outcome increased fetal hr w/movment

dm mom ask about breast feeding

its encouraged insulin dose not cross to the milk

jaundice appears

2-3 days of life

primipara

normally go through effacment before dilation

syphillis

may be passed to the fetus after 4 months of pg

lvn care p gen anesthsia

monitor bp q 3-5 min until stable

tetany

uterus contarcted for more then 1 min= can rupture report to md

mag sulfate

assess resp at least 12 per min

dm mom check baby for

hypogly, resp. distress and hypocalcemia

baby chocking or resp distress

pick it up by the feet and head low

post term babys look

small like they have lost wt long nails little subq dry skin

bcp's

inhibit fsh production

small for gestational age infants develop

hypothermia

pg diet

increase iron increase fa and increase cals 300

mastitis

apply heat to the breast

american red cross

clara barton

elizabeth mahoney

first black nurse

managed care

80's to halt rapidly growing health care costs

complemetry care

accupuncture, theraputic tuch and imagry

joint commision

makes sure hospitals meet rigid standards

inferred consent

no consent given by pt but situation is life threating so its inferred the pt wants it

medical asepsis

process to minimize or eliminate organisms that can cause disease

bureaucratic leadership

policy minded

extingushier c

electcal fire

paradoxical responce

opposite effect that you wanted

enteric coated

dose not disolve until reaches the intestines

woodslamp

high pressure mercury lamp long wave ultra violet to detect suprafical fungi and bacterial infx

moist packs applied to the skin

for debridment

vitiligo

lack melanocytes results in white patchy skin

carbuncle

several intr connecting boils in a cluster...yuck

burn types

chemical,electrical, rad or thermal

entire back burn

18%

major burn

includes second degree over 25% of body and inhilation burn injuries

digestion of carbs

begins in the mouth

major fnx of fat

provide energy

albumin and globulin

blood proteins that help keep fluid were it belongs

electrolytes form

acid bases and salts

trace minerals

needed in the body in small amounts like zinc

fat soluble vits

likely to cause tox cuz they accumulate in the body

major intracellular electrolyte

basal cell

non metasta in whites around the head and neck looks like fleshy nodule

saquamous

may mets red scaly patch second most common face lips mouth and rim of ears

malignant melonoma

danger dark pigmented mole occurs near a mole or other dark spot

endocrin consists of

adrenals ovaries testes islets parathyroid pit and thyroid gland

pit gland master gland cuz

it stimulates other endocrin glands to secrete

fbg

65-115 nl

cushings syndrome

excessive adrenalcortico hormone from the adrenal cortex or brain tumor

hypotension, circulatory colapse coma death! Dont stop steroids all at once or an infx can cause this adrenal crisis emergency tx: iv fluids to increase bv, Iv hydrocortisone , recombant position with legs up

tyroid storm tx

hypothermia blanket antithyroid and antiarrythmic meds

myexdema coma

long term untx hypothyroidism

4 main oral anti dm catagories

alphaglucaside inhibators, biguanides, sulfono's, thiazolidenedes

glyburide

sulfon

regular insulin humulin r

short acting

humalog insulin lispro

intramediate acting

lipodystrophy

change insulin sytes often!

hypogl sx

sudden onset of pale moist cool clammy skin w/ sweating shaking and hungur

hypergly sx

SL Ow onset of flushed dry skin, hot drowsy fruity breath, labored breathing

somongi effect

hypogly followed by rebound hyper gly

tx of hyper gly

iv fluids antiemetics and low dose insulin

insulin or oral dm meds dont drink cuz

etoh can cause extra hypogly effect

glucotrol and etoh

disulfaram effect violent, flusshing, throbbing ha, sob and unstable bp

20 grams simple carb =

4 oz OJ

high humidity ant temps 95-100

watch for heat related injuries

muscle cramps, ha, dizzines, weakness and excessive sweating=

heat exhustion tx cool the pt w/o chilling, loosen clothes cold wet compress to the skin water replacment

heat stroke

core temp 105-110 sweatring stops and brain cells FRY!

heat stroke nursing tx

remove clothes wrap client in cold wet sheets, place ice axillary and groin

hypothermia

core 94 or less sx shake clumbsy slow movment arrythmias tx gradual rewarming cardiac monitor warm bevrages when awake

frost bite

blisters some nerve damage increased hypersensativity to cold

anaphlatic SIRES

stabilize,identify,reverse,eleminat,support

s/s of anaphlaxis

hives cold clammy skin dizzy weak low bp nv itchy watery eyes

nervous sytem

communicate and control

frontal lobe

written and motor speech

expressive aphasia

understand written and verbal but can not write or speak it

can not understand writen and verbal

receptive aphasia

nervous sytem aging

short term memory loss and thermoreg

206 bones in body

FYI

rickets

vit d bone formation

foramen

were stuff goes through

cardiovascular aging results in

hematopoiesis

low co2 stimulates breathing

for nl adults

aging of the resp system

increase r/f infx

aging urinary system

nocturia

lens

focuses on light rays

frail elderly

85 and over

maoi

nardil

avoid pickles,aged, smoked, fermeted foods, dry sausage and wine maoi on thymine foods watch for htn crisis sx include occipital ha stiff neck palp sweating dilated pupils photophobia and n

ect adverse effects

ha and myalgia

benzo's

xanax and valium

epse

akathsia

nms cardinal sign

hyperthemia and rigidity

anametadine and cogentin

treat adverse effects of antipycotics

autisim sx

rituals and lack of social interactions

adhd meds

strattera, ritalin and aderall

amniotic fluid

cushing

para

viable newborn or gestation p 20 weeks

adolsent pg complications

pre eclamp, eclamp, spon ab

molar pg early indication

uterus enlarging early

iorn rich

tofu, fortifed cearl, bread and lean meat

accreta placenta

failing to seperate in 20-30 minutes retained placenta

mag sulfate(epson saltsO

prevent and control sz

postpartum hemmorage

notify md and do external uterine massage

rh neg mom rhpos baby

baby will have eretroblastosis fetalis if left untxed

moro reflex

startle through out arms draw up legs

post partum

3-10 days post delivery = mild depression usually resolves in 2 weeks

immediatly report jaundice in the newborn esp, in the first 24 hrs cuz

indignative of pathologic jaundice results from mother building up ab against her own fetus

first meconium

greenish black

nl newborn axillary temp

97.6-98.6

newborn bp

50/30-80/50

newborn apical for 1 min

120-160 and slightly irregular

newborn resp

30-60

2 months

dtap,hib, ipv,pcv

scarlet fever

pinpoint reddots and strawberry tongue

chronic renal failure diet

avoid high k foods, Eat high ca and high fe

cushing diet

increase protein increase k descrease na and cals

addisons diet

increase na decrease k

1 L of water=

1kg

1 cup

240 ml's

1 pint

480 ml

1 quart

960ml's

turgur

forearm and sternum if wrinkled greater then 20 sec poor

stage4 full thickness fascia, involves muscle tendon and bone stage3 full thickness damage and necrosis down to the facia skin breakdown stage 2 epidermis interrupted abrasion or blister stage 1 intact skin nonblanching errathemia

trochlear

downward and inward eye movment

trigeminal

corneal reflex, chewing, face and scalp sensation

vagus

gag reflex, swallowing talking,sensations of the throat and larynx

hypoglossal

tongue moevemnt

battle sign

ecchymosis at mastoid process in basilar skull fx ususally occurs 48hrs p

nystagmus

constant involontary eye movment

hippus

rythmatic and rapid dilation and constriction of the pupil

decorticate

in c sz

generalized absence sz

brief of conciouness or posture

myoclonic generalized sz

repetative muscle contarctions jerking

tonic clonic

stiffness followed by loc and rythmatic contrations of extremites

simple partial sz

focal no loc aura usually

complex partial sz

focal sz w/alteration in loc

afterload

resestance against which the vebtericles must pump when ejecting blood

nl icp

10-15

l side heart failures sx

pulmonary edema, sob, orthopnea,

r side heart failure

dependent edema, hepatomegaly, abd pain, bloating

ra

fatigue anorexia low grade fever joint pain

when is chest tube ready for removal

when suction is removed no flucuation is noted in the water seal chamber

elderly client c PNU first symptom

altered mental status

chemo before bone marrow transplant cuz

to make space for new bone marrow and that all the cancer cells are removed

nl neutrophil count

2200 to 7000mm3

neupogen

colony stimulating factor given p chemo to stim wbc's

nl platlett count

150,000-400,000

s/s of hyponatremia

ha lathargy confusion sz

hyper mg

loss of deep tendon reflexes

demerol metabolites

cause sz's

albumin iv

colliod osmotic pressure

cholinergics

increase ach in the brain good for Alzheimers

chronotropic

change in heart rate

inotropic

force of heart rate

dromotropic

change w/conduction of electrical inpluses

p ng med administration

leave pt sitting for 30 min

IV ntg drip

in glass bottle cuz ntg adhers to plastic

iv acess

insert needle at 30 degrees

phelbitis

remove cath apply warmth moist pack

intr atrial lines

for chemo to give close to tumor

when drawing blood from CVAD

disgard 10 ml's

disconnected CVAD pt c CP first do

turn pt on left side in trendelinburg

Groshong cathether

has no clamp just a 3 way valve at the end

needle to access inplanted port

20 gauge noncoring

flushing Groshong

5-10 mls of NS weekly

RH positive can recieve rh neg bloos

but RH neg cant recieve RH pos If it happens and its admisterd wrong till th eblood back and theMD

type o blood can only recieve

type o blood

neutropenic pt usually be transfused with

granulocytes

unit of whole blood must be used in

4 hrs

Graft vs. host disease

attack on host tissue from lymphocytes. Usually occurs w/comprimised pt

type AB

means no antibodies in the blood

amiodarone and dig

cause increased dig levels

aldactone

k sparing diuretic should not have increase k

electrolyte inbalance w/dig

hypokalemia can cause dig tox

glucose6 phosphate dehydrogenase

if on asa watch for hemolysis of rbcs. Esp c jews

flagly and etoh

= n/v

antacids and dig

can decrease absorption of dig!

desmepression acetate

for hemophilia and von willebrands releases factor 8

timoptic

reduces aqueous humor production

tapazole

tx for hyperthyroid

cerebyx

tx of sz's

s/e of NTG

reflex tachycardia

tetracycline

no if less then 8 cuz it stains teeth

phenergen adverse rxn

tortacollis neck head tilted to one side

trycyclic

adverse rxn= increased temp

body h20

60-70%

main source of energy in a diet

carbs

breast feeding

increase cals by 500

antabuse and drinking etoh

cp, ha, nausea sweating

opioiod w/d sx

tachycardia, tacypnea, htn, mydiasos

thrill

narrow or buldging blood vessel pluse feels like a vibration

passive immunity

administration of ab produced by other people or animals. the pt is exposed to the antigen and produces his own ab

2 y/o

uses 1-2 word sentances

late fetal heart decels

turn on l side give 02

p epidural priority action

monitor bp

evaluation for restrant need

q4h

haldol s/e

hypotension tacycardia

regression

pt's use it to elimimate anxiety

ivp

be NPO

physiological jaundice

on the second or third day decreases by 6th or 7th

renal failure be concerned w/

change in BP

infectious hep

enteric percautions

high k foods

oj, dried dates, milk

position for hypovolemic shock

head on pillow and feet slightly elevated

cobalt implant percations

visitors 1 hr q day exposure, everyone wears a protective shield, rad tx sign on door

fetus in posterior position

back discomfort

expressive aphasia

theraputically encourage to communicate even though not using correcrt words

preeclampsia early signs

htn, proteinurea, and edema

following a hip replacment during first postop hours observe for

hemorrage

infant by 6 months

doubles there wt

sengstaaken blakemore tube for espohgeal varices most important safety intervention

scissors at bedside resp system can be occluded if balloon slips moves up esophagus and pressures trach. cut tube if in distress!

newborns usually sleep

20 out of 24 hrs

GDM most hard to control BS

early post partum R/T the [placenta contains the hormone insulinase which blocks insulin during PG

bethanechol cl

stimulates PNS increases tone and motility of smooth muscles. Give to a pt if they cant pee p a TURP..

preemie

use soft preemee nip and half strength formula then work to full

nl pluse for a 5 y/o

100

pg be sure to take

fe and folacin supplemental

3 month old can

focus eyes on stuff

lactalose

decreas blood ammonia

hypoglycemia in a infant

shrill or high pitched cry

korsakoff syndrome

neuro degeneration R/T vit deficiency

pt in er c DT's orders for IV w/vit b6, valium, or check VS. Which one to do first?

give IV w/ vit b6 first! wow strange!

baby w/spinal bifida most important assesment

measure head circ qd

hirschsprugs diseas sx

fails to pass meconium in 24-48 hrs classic sx

apical on a infant

left mid clavicular, 3rd and 4th intra costal

interventions for a kid with RA

warm comressions and splints at noc

mumps percautions

droplet, direct and indirect contact via the resp.system

position for extrophy of the bladder

side lying to aid in emptying

bronchiolitis priority assesment

flaring nostrils, exp. stridor and wheezing

negativism

struggle by a kid between what they can and can not do

one dram

4 ml's

interssusception

sudden onset of sx

left side heart failure

fatigue, dyspnea, wheezing

menieres disease

extreme vertigo, bedrest.

osteoporosis teaching

diet high in protein, ca, vit d, wt bearing excercise no etoh

low k

skeletal muscle weakness

bucks extension p hip fx

relives muscle spasms

pt wants to leave AMA what to do FIRTS

notify MD, then they will try to make her stay, then you can give AMA form!

breech of duty

care did not meet accepted standards

child had cardiac cath monitor

peripheral pluses for symetry

child admitted with sickle cell crisis what should nurse do first

vs to get a baseline including temp. they are prone to infx

low absolute granulocyte

restrict visitors w/active infx's

cytoxan for chemo monitor for

hematuria which can indicate hemorragic cystitis which is an adverse effect

subtle signs of resp distress in a child/infant

restless, increase resp effort, increase resp,increase pulse, can not calm kid

a child with asthma attack is brought in he has no wheezing!

probabley hypoxic or would be wheezing! emergancy

after shunt placment

bed flat to prevent subdural hematoma

meningitis

droplet spread my resp.

after cleft repair

sit upright for feeding

bactrim

crystalizes the kidneys drink lots of water

child is a home and has scolding burn

immediatly flush site with tempid water to stop progression

glomerulonephritis

monotor bp q 4 h htn major complication

nl urine output for infant

5-10 ml/hr

nephroblastoma

surg in 24-48 hrs after dx then chemo and rad to follow up

infant w/ an overdose of thyroid med

tachycardia, fever, irratiblity,sweating

neonate w/pyloric stenosis

postion on right side

NG tube and complains of nausea

check placment then irragate the tube

gluten free diet

no malted milk, wheat bread or spaghetti

cleft palate surg delayed till

18 months

cleft lip

birth to 3 months

exacerbation of juvenile RA

stress and climate

a child w/ duchennes ms

low cal high protein high fiber R/F constipation

absence sz

brief loss of responsivness with minimal or no alteration in muscle tone

Ritilin adverse rxn

slow growth and ht, sleepy, decreased appitite

do not admin fe with

milk it delays absorption

breast feeding

natural acquired passive immunity

kid w/ cyanotic heart defect what sx

cyanosis, crabby, clubbing, crouching

ultrasound for PG

used at 18-40 weeks

what is priority when feeding a neonate wit a cleft

frequent small feedings

fetal blood ph shows 7.12 what nursing intervention is called for

prep for c section. baby in fetal acidosis

decrement

letting down phase of uterine contractions

to halt uterine contractions give

brethine/trebutaline

monitor i/o with pitocin because

it causes water intoxication leading to sz,coma and death

circulating HCg disapears

8-24 hrs p delivery

letting go phase mom excepts the kid as a

separate individual

neonate temp range

96-97.7

phototherapy to reduce

unconjugated bili

methergine ergot alkaloid given to stimulate

uterine contractions p birth to decrease bleeding

mag sulfate produces smooth muscles depression asses for

post partum hemmorage

p amniotomy priority nursing intervention

asses fetal heart tones

transitional phase of labor

cerv dilation 8-10 contractions 1-2 min apart

hiatal hernia

avoid carbonated drinks R/t to esophageal irratation and increased pain

crohns diet needs

high cal,protein,carbs and low fat

cytoctec for gastric ulcer

monitor for diahrrea

levothroxine

can cause tacycardia indicator of thyroid tox

sinemet is working if pt

decreases tremors

eccrine gland

associated w/body temp regulation

on corticosteriods check

blood glucose

partial thickness burn

fluid filled vesicles

eswl

kindney stones are shattered

a pt w/kidney stones

drink 3 l h20 each day

PTH deficiency abnl serum levels

ca and phospurus

GTT dx w/Dm if greater then

200

radioactive iodine test

increase w/hyperthyroid decrease w/hypo t

levothyroxine

inhances coumadin dose!

addisonian crisis

hyponatremia needs NS

abd rigidity

classic sign of peritonitis

HA after lumbar puncture

increase fluids to help restures CSF volume

greatest r/f femoral fx is

hypovolemia from hemorrhage

temporal lobe

controls speech and hearing

plasmapheresis

ab removed from blood

to prevent vasospasms

give crystalloids

epitaxis

head slightly forward sitting up

breathing w/ emphysema triggered by

low 02 levels

TB drugs effectvness can be indicated when

sputum results show neg for TB

after a cath there is swelling at site

apply firm pressure and have some one call MD

ca channel blocker

decrease bp and hr

if BP below 90 systolic

delay NTG

Growth spurt, fever, and irritability changing to lethargy. The nurse is monitoring a child with hydrocephalus who has a ventriculoperitoneal (VP) shunt. What clinical manifestations will the nurse be most concerned about? he major complications of VP shunts are infection and malfunction. Children can "outgrow" shunts or distal ends can dislodge after growth spurts. Fever can be a sign of an infected shunt, and irritability deteriorating to lethargy could be due to increased intracranial pressure (ICP) from a blocked shunt. Appetite usually decreases with increasing ICP;

A 2-year-old toddler was diagnosed with iron-deficiency anemia. Which of the following statements best describes the anemias of childhood?

The clinical manifestations of anemia are directly related to the decrease in oxygen-carrying capacity of the blood. Clinical manifestations of fatigability, anorexia, weakness, and tachycardia are a result of vitamin B12 and folic acid deficiency. This results in reduced production of red blood cells, and a 2-year-old child will manifest symptoms of this disorder.

When assessing clinical indicators of adequate cardiac output in children, which of the following signs are most important?

Pedal pulses, skin temperature, and capillary refill.

A mother with a 4-month-old infant comes to the clinic for a well-baby examination. The nurse advises the mother to change the formula she is feeding the baby to one that contains iron. The nurse explains that the reason for this is

The infant`s iron source from the mother is depleted. Between 3 and 5 months, the infant has used the iron provided by the mother and requires further supplementation if bottle feeding

Which of the following symptoms is the priority assessment because it is suggestive of a complication of a central nervous system infection?

Separation of cranial sutures Meningitis is a common CNS infection of infancy and early childhood. Increased intracranial pressure, which can accompany meningitis, accounts for separation of the cranial sutures, bulging not depresse.

What anatomical condition must be present in order for an infant with complete transposition of the great vessels to survive at birth?

Large septal defect. Because complete transposition results in two closed blood systems, the child can survive only if a large septal defect is present

Assessing a child with a possible cardiac condition, the nurse knows that a child with a large patent ductus arteriosus would exhibit which of the following symptoms?

Is acyanotic but has difficulty breathing after physical activity.PDA is acyanotic.

Following a saline-induced therapeutic abortion, a patient has developed disseminated intravascular coagulation (DIC). The most critical nursing intervention for this patient is to

Administer ordered medications.In DIC, the patient begins to hemorrhage after the initial hypercoagulability uses up the clotting factors in the blood. Administering heparin, therefore, is a critical nursing intervention. Heparin prevents clot formation and increases available fibrinogen, coagulation factors, and platelets

A patient`s laboratory results indicate a creatinine level of 7 mg/dL. This finding would lead the nurse to place the highest priority on monitoring the patient`s

This elevated creatinine suggests impaired renal function. Monitoring intake and output will provide data related to renal function

A common test used to determine fetal status in the presence of preeclampsia is the nonstress test (NST). If this test is "reactive," the nurse knows that it means

Reactive = good outcome. Increased FHR with movement indicates normal reaction and adequate CNS integration

Which of the following statements is usually true about cervical changes in primiparas

Primiparas normally go through effacement before dilation of the cervix. Multiparas tend to dilate and efface simultaneously.

A patient is 3 days postpartum. Her vital signs are stable; her fundus is 3 fingerbreadths below the umbilicus, and her lochia rubra is moderate. Her breasts are hard and warm to the touch. The nurse would evaluate that the patient

Is normal for 3 days postpartumFrom the assessment findings of the lochia and fundus, the new mother is progressing normally during the postpartum period. The breast signs indicate normal engorgement, which occurs about 3 days after birth. With stable vital signs, infection is not likely to be a problem. Applying warm packs and wearing a nursing bra will reduce discomfort.

Assessing a patient with eclampsia, the nurse knows that a cardinal symptom is

High blood pressure is one of the cardinal symptoms of toxemia or eclampsia, along with excessive weight gain, edema, and albumin in the urine.

An eclamptic patient has been receiving magnesium sulfate IV 2 g/hour. What symptom would indicate that the current dose be continued

A respiratory rate of 16 per minute.The respiratory rate must be maintained at a rate of at least 12 per minute as a precaution against excessive depression of impulses at the myoneural junction

A patient is gravida 3 para 2 and is in a labor room. After a vaginal exam, it is determined that the presenting head is at station +3. The appropriate nursing action is to

Prepare for delivery of the baby.f the head is +3, it is just about crowning, and because the patient is a multipara, it would be reasonable to assume delivery is imminent.

Counseling a patient who is starting to use oral contraception, the nurse explains that birth control pills work by the mechanism of

Birth control pills are small doses of estrogen and progesterone that maintain sufficient levels in the body to inhibit the pituitary from producing the follicle-stimulating hormone.

During a physical exam of an infant with congenital hip dysplasia, the nurse would observe for which of the following characteristics?

Abduction is limited in the affected leg. The nurse would also find asymmetrical gluteal folds and an absent femoral pulse when the affected leg is abducted.

If RhoGAM is given to a mother after giving birth to a healthy baby, the condition that must be present for the globulin to be effective is that the

RhoGAM will not work if there is any titer in the blood; thus, it is important to administer it within 72 hours after delivery or abortion if the mother shows no evidence of antibody production. The mother would be Rh negative and the baby Rh positive for RhoGAM to be needed.

The nurse is doing data collection on a postpartum patient. Suspecting infection, the nurse would assess for

The major symptoms of infection would be rapid pulse, foul-smelling lochia or discharge, and discomfort and tenderness of the abdomen

A patient, 36 weeks pregnant, is having a contraction stress test (oxytocin challenge test). After 35 minutes, her uterus begins to contract, and the nurse observes three 40-second-long contractions in a 10-minute period. She has two contractions within 5 minutes, and her uterus remains contracted after the second contraction. The first nursing action is to

The first action is to turn the Pitocin off. If the fetal heart rate has dropped in response to the prolonged contraction, turning the mother on her side (3) and administering oxygen (2) may be necessary.

A nurse working in a prenatal clinic recognizes that the physician should immediately see any patient who presents with

Blurred vision is an advanced indicator of pregnancy-induced hypertension (PIH) and the physician should see the patient immediately

The nurse is assessing a 75-year-old patient who is taking digitalis. Assessing for digitalis toxicity, the nurse would identify

Anorexia, nausea, vomiting.

NL wbc

5-10 k

A patient has had a partial colectomy because of a diagnosis of cancer. Surgery began at 7:30 AM. She returned to the unit at 1:30 PM. During a 6:00 PM assessment, the nurse observed all of the following. A priority concern that would require the earliest intervention is a

Inability to void after surgery is a common problem resulting from anesthesia or pain medication and requires an early intervention. It is important to be aware of the patient`s output for several reasons: to ensure adequate intake, to detect renal problems, and to assess for blood pressure problems. Solution to this problem is catheterization, based on a physician`s order. The dressing (1) should be closely observed but is not presently a problem. The area on the calf (2) may be developing throbophlebitis and should be reported to the physician immediatel

Administering care to a patient in hypovolemic shock, the sign that the nurse would expect to observe is

n shock, there is decreased blood volume through the kidneys. This is evidenced by a decrease in the amount of urine excreted. The body has numerous compensatory mechanisms that assist in keeping the blood pressure normal for a short time.

A systolic blood pressure of 60 mm Hg or less would indicate shock in which of the following patient age groups?

A systolic blood pressure of 60 mm Hg or less found in children 5 to 12 years old would indicate shock.

The nurse has an order to remove a patient`s nasogastric tube. The correct nursing action related to this procedure would be to

Removing the tube quickly while keeping it pinched lessens the risk of gastric secretions falling into the trachea during removal. Instilling 20-30 mL of air, rather than normal saline, into the tube will also help prevent aspiration of gastric secretions. Unsterile gloves are worn for this procedure.

A patient is about to be discharged on the drug bishydroxycoumarin (Dicumarol).

Dicumarol is an anticoagulant drug and one of the dangers involved is bleeding. Using a safety razor can lead to bleeding through cuts. The drug should be given at the same time daily but not related to meals. Due to danger of bleeding, missed doses should not be made up. The LVN is prepared to do this patient teaching.

The nurse is inserting a Foley catheter into a male patient. How far should the catheter be inserted before inflating the balloon?

7-9 inches.

The priority assessment for a patient with acute infective (bacterial) endocarditis is

While all of the symptoms may be present, the major complication with this condition is that of emboli. If emboli arise in the right heart chambers, they will terminate in the lungs; left chamber emboli may travel anywhere in the arterial tree. The nurse should constantly monitor for this complication.

Basilar crackles are present in a patient`s lungs on auscultation. The nurse knows that these are discrete, noncontinuous sounds that are

Basilar crackles are usually heard during inspiration and are caused by sudden opening of alveoli.

In developing a nursing care plan for a patient with Buerger`s disease, it is important to include

Buerger-Allen exercises improve peripheral arterial circulation which is specific for treatment of Buerger`s disease

The nurse is collecting data on a patient with joint pain. The nurse knows that a patient who is in the early stages of rheumatoid arthritis is most likely to complain of pain, swelling, and limitation of motion in the

Rheumatoid arthritis typically begins with inflammatory changes in the small joints of the hands, wrists, and feet.

The nurse would expect to find an improvement in which of the blood values as a result of dialysis treatment?

High creatinine levels will be decreased. Anemia is a result of decreased production of erythropoietin by the kidney and is not affected by hemodialysis

A 50-year-old patient has a tracheostomy and requires tracheal suctioning. The first intervention in completing this procedure would be to

Before deflating the tracheal cuff, the nurse will apply oral or nasal suction to the airway to prevent secretions from falling into the lungs

The nurse is teaching a type 1 diabetic patient about her diet, which is based on the exchange system. The nurse will know the patient has learned correctly when she says that she can have as much as she wants of

Lettuce contains primarily water and fiber, and is considered a "free food" in the American Dietetic Association exchange lists.

A patient with a bile duct obstruction is jaundiced. The priority intervention to control the itching associated with jaundice is to

Itching is made worse by vasodilation. Tepid water prevents excessive vasodilation

When using nasotracheal suction to clear a patient`s airway of excessive secretions, a principle of the suctioning procedure is to

To prevent trauma to the mucous membranes lining the airway, suction should be applied only while withdrawing the cathete

A patient with an admitting diagnosis of head injury has a Glasgow Coma Score of 3 - 5 - 4. The nurse`s understanding of this test is that the patient

A Glasgow Coma Score of 3 - 5 - 4 means that the patient is able to open his eyes when spoken to and can localize pain, attempting to remove noxious stimuli when motor function is tested. He is not able to follow commands

Part of a plan of care for a patient with increased intracranial pressure is to maintain an adequate airway and to promote gas exchange. To accomplish these goals, an effective nursing action is to

Hypercapnia leads to vasodilation, thus increasing cerebral blood flow and increasing intracranial pressure.

Based on nursing knowledge, the nurse is aware that an epidural hematoma is characterized by

A short period of unconsciousness followed by a lucid period, followed by rapid deterioration. Epidural hematomas classically present with a brief period of unconsciousness, followed by a lucid interval of varying duration, and finally followed by rapid deterioration of the level of consciousness, accompanied by complaints of a severe headache.

A patient is admitted following an automobile accident in which he sustained a contusion. The nurse knows that the significance of a contusion is that

Laceration of the brain may occur. Laceration, a more severe consequence of closed head injury, occurs as the brain tissue moves across the uneven base of the skull in a contusion. Contusion causes cerebral dysfunction, which results in bruising of the brain. A concussion causes transient loss of consciousness and retrograde amnesia, and is generally reversible.

A patient in the early stages of progressive renal failure is admitted to the hospital. The initial assessment will probably reveal

Polyuria, low urine specific gravity, polydipsia.

The nurse will assess for the most significant complication in patients undergoing chronic peritoneal dialysis, which is

Peritonitis is a grave complication with peritoneal dialysis. Hemodialysis may be necessary until infection clears. Excess fluid and protein effluent into the peritoneum also complicate care. Use of aseptic technique is essential

The most important teaching the nurse should do for a patient to have well managed intermittent hemodialysis is

It is essential that the end-stage renal patient adhere to all aspects of the medical regimen. Only excess solutes and fluid are removed with dialysis. Blood pressure management needs to be consistent, not just between treatments, aspects of care concerning concomitant anemia, and phosphate/calcium/vitamin D imbalance, as well as protein restriction and fluid restriction, must be carried out at all times. The dialysis patient continues to be uremic and has multisystem problems that continue despite dialysis.

A 12-year-old patient has just been returned to the unit following a tonsillectomy. A priority nursing intervention during the postoperative period is to

Apple juice or water is given as soon as the patient is awake and not hemorrhaging. Avoidance of citrus juices will prevent irritation of the operative site. The patient should be placed on his abdomen or side to facilitate drainage and prevent aspiration. Ice bags are applied to the neck to prevent edema and bleeding.

A patient with chronic renal failure is on continuous ambulatory peritoneal dialysis (CAPD). Which nursing diagnosis would have the highest priority?

Imbalanced nutrition: less than body requirements. There is a high risk of infection in patients receiving CAPD because microorganisms can enter the body by migrating around, or through, the peritoneal dialysis catheter. They may also enter through contaminated dialysate solutions. The other diagnoses are not life threatening for a patient on CAPD.

Which of the following statements is true of skeletal traction?

Fractures can be reduced because more weight can be used than with skin traction.Because more weight can be applied with skeletal traction, it can be used to reduce fractures and maintain alignment. It is not used commonly in the elderly because of prolonged immobilization. It is not preferred for children because some displacement of fracture fragments is desirable to prevent growth disturbance. Frequently, patients have more mobility than they do with skin traction, because balanced suspension is often incorporated with skeletal traction.

Russell`s traction is easily recognized because it incorporates a

Russell`s traction is a type of skin traction that incorporates a sling under the knee that is connected by a rope to an overhead bar pulley. It is frequently used to treat femoral shaft fractures in the adolescent.

When evaluating all forms of traction, the nurse will check that the direction of pull is controlled by the

Rope/pulley system.

A patient has had a cystectomy and ure-teroileostomy (ileal conduit). The nurse is assigned this patient in the postoperative period. Which of the following observations indicates an unexpected outcome and requires priority care?

Feces in the drainage appliance. The ileal conduit procedure incorporates implantation of the ureters into a portion of the ileum that has been resected from its anatomical position and now functions as a reservoir or conduit for urine. The proximal and distal ileal borders can be resumed. Feces should not be draining from the conduit.

A patient requires that a bronchoscopy procedure be done. Due to his physical condition, he will be awake during the procedure. As part of the pretest teaching, the nurse will instruct him that before the scope insertion, his neck will be positioned so that it is

Hyperextension brings the pharynx into alignment with the trachea and allows the scope to be inserted without trauma.

A patient with chronic lymphocytic leukemia is started on chemotherapy. Monitoring the administration of these drugs, the nurse would suggest dietary guidance to

Consume fluids and foods high in bulk and fiber several hours before the treatment. R/T R/F constipation

Which one of the following conditions could lead to an inaccurate pulse oximetry reading if the sensor is attached to the patient`s ear?

Hypothermia or fever may lead to an inaccurate reading. Artificial nails may distort a reading if a finger probe is used. Vasoconstriction can cause an inaccurate reading of oxygen saturation. Arterial saturations have a close correlation with the reading from the pulse oximeter as long as the arterial saturation is above 70 percen

There is a physician`s order to irrigate a patient`s bladder. The priority nursing measure to ensure patency is to

Irrigate with 20 mL of normal saline to establish patency.

When assessing an ECG, the nurse knows that the P-R interval represents the time it takes for the

Impulse to travel to the ventricles

When a patient has suffered severe burns all over his body, the most effective method of monitoring the cardiovascular system is

Pulmonary artery pressure is the most effective method of monitoring the cardiovascular system for this patient. Patients with a large percentage of burned body surface often do not have an area where a cuff can be applie

A female patient has orders for an oral cholecystogram. Prior to the test, the nursing intervention would be to

Explain that diarrhea may result from the dye tablets.

Knowing that a patient has the diagnosis of heart failure (HF), what symptoms would the nurse assess during data collection?

Cyanosis, crackles, gallop rhythm

The laboratory result that should be monitored regularly in a patient who is receiving gentamicin (Garamycin) is

Platelets.

A patient with thrombophlebitis should be positioned so that his legs are

Elevating the legs about 30 degrees promotes venous return and reduces leg edema. Elevation beyond 45 degrees reduces arterial flow and causes sharp flexion at the hip, thereby reducing venous return. Leaving the legs flat on the bed or dependent promotes edema formation and venous stasis. Patients with arterial, rather than venous, insufficiency benefit from a dependent position.

A hypothyroid patient has orders for all of the following medications. The nurse would evaluate the patient most closely following administration of which medication?

Hypothyroidism reduces the metabolic rate and prolongs the sedative effects of medications. Narcotics are especially dangerous and should be given in smaller doses. The patient must be closely monitored for signs of oversedation and respiratory depression.

A patient with COPD has developed secondary polycythemia. Which nursing diagnosis would be included in the care plan because of the polycythemia?

Chronic hypoxia associated with COPD may stimulate excessive RBC production (polycythemia). This results in increased blood viscosity and the risk of thrombosis. The other nursing diagnoses are not applicable in this situation.

The nurse is teaching a patient with a new colostomy how to apply an appliance to a colostomy. How much skin should remain exposed between the stoma and the ring of the appliance?

A colostomy appliance should be cut to fit the stoma so that there is no pressure placed on the stoma by the appliance and there is a minimum amount of skin exposed to fecal drainage. Leaving 1/8 inch of skin exposed conforms to these criteria.

Which of the following blood chemistry results would the nurse expect to find elevated in a patient with right-sided heart failure?

The liver becomes engorged with blood in right-sided heart failure. Liver function studies, such as the LDH, an enzyme production test for the liver, will be abnormally elevated in 40 percent of the patients. Serum bilirubin is also frequently increased

The treatment prescribed for the burned area of skin before skin grafting can take place will include

In addition to the germicidal soap scrubs, systemic antibiotics are administered to prevent infection of the wound. Silver nitrate is not a common treatment today

When a head injury patient has fluid draining from the left ear, the nurse will immediately position the patient with the head of his bed

It is important to decrease intracranial pressure (head of bed elevated) and to allow for drainage (head turned to left)

The patient has arrived in the recovery room following a lobectomy. As the nurse assigned to care for the patient during the immediate postoperative period, the first intervention will be to

Closed chest drainage is used for lobectomies to reestablish negative pressure in the chest. Because the breathing mechanism operates on the principle of negative pressure, this is an essential action. The other interventions would follow this one. Connect the Pleur-evac to suction.

A nursing care plan for a patient with a suprapubic cystostomy would include

Allowing the patient to void naturally will be done prior to removal of the catheter to ensure adequate emptying of the bladder

A patient admitted for possible bleeding in the cerebrum has vital signs taken every hour to monitor the neurological status. Which of the following neurological checks will give the nurse the best information about the extent of bleeding?

Pupillary checks reflect function of the third cranial nerve, which stretches as it becomes displaced by blood, tumor, etc.

Patient teaching following cataract surgery should include

They must use only one eye at a time to prevent double vision. The function of the lens is that of accommodation, the focusing of near objects on the retina by the lens; therefore, only the remaining lens will function in this capacity, depending on whether a cataract is present.

Preoperative teaching for a patient scheduled for a laryngectomy should include the fact that

The patient will be able to speak again, but it will not be the same as before surgery. Most of the laryngectomy patients will use esophageal speech or a mechanical device for communication. They can usually begin to take oral fluids sometime after 48 hours. They are generally fed by an intravenous or nasogastric tube prior to oral feedings. Because the larynx is removed, it will be impossible to breathe through the nose.

he main complication following a nephrostomy that the nurse must assess for is

While all the other conditions may be complications, bleeding from the site is the main concern. The procedure is done to achieve relief from infection caused by urinary stasis, which may have resulted in kidney congestion.

Hemorrhage is a major complication following oral surgery or radical neck dissection. If this condition occurs, the most immediate nursing intervention would be to

Putting pressure over the vessels in the neck may be lifesaving because a severe blood loss can occur rapidly, leading to shock and death. The surgeon would be notified as soon as possible.

To achieve the desired outcome of fracture healing, which nursing goal should receive the highest priority?

Maintaining the prescribed immobilization and body alignment will keep the fracture fragments in close anatomical proximity, thereby promoting functional fracture healing. This goal should receive the highest priority. The other goals, although applicable in the

The nurse, collecting data for a nursing history from a newly admitted patient, learns that he has a Denver shunt. This suggests that he has a history of

The Denver shunt is a type of peritoneovascular shunt used in the treatment of patients who have cirrhosis with ascites. The shunt diverts ascitic fluid from the abdomen into the jugular vein or the vena cava

The nurse has been teaching a patient to use crutches. Which statement made by the patient indicates a need for more teaching?

The hand grips should be placed so that the elbows are flexed at 20-30 degrees when standing with the crutches. This placement should not be changed as long as the patient continues to need crutches. The other statements indicate effective learning.

The nurse is preparing a patient for a myelogram using metrizamide (Amipaque), a water-soluble contrast material. The nurse will know the patient understands the postmyelogram care regimen when she says

The head must be kept elevated because this drug could provoke a seizure if it reaches the brain in a bolus form. After myelography that uses an oil-based contrast medium (Pantopaque), patients are kept flat. Forcing fluids helps prevent postmyelogram headache by replacing lost spinal fluid. Itching suggests an allergic reaction, while a stiff neck suggests meningeal irritation; neither is an expected response to a myelogram.

The precaution protocol necessary to implement for the biohazard of Pneumonic Plague is

Precautions include Standard plus Droplet (eye protection and surgical mask)suntil 48 s72 hours after antibiotic treatment.

All staff must wear disposable particulate respirators (HEPA filter) when

Staff must wear disposable respirators when there is inadequate room ventilation. If the room has a directional negative-pressure ventilation system, the staff would not be required to wear a HEPA filter mask, even if the patient had TB. These masks are required for droplet transmissionsbased conditions.

The nurse is assigned to care for two patients. One patient has just returned from surgery for an abdominal resection. The second patient is hospitalized with an acute case of tuberculosis. What special precautions should the nurse take when providing care for these two patients?

There are no special precautions; however, the nurse must strictly adhere to barrier nursing principles and the two patients must be treated separately. Providing care to the abdominal surgery patient before the TB patient would be appropriate. Proper handwashing is essential (1), but isolation garb is needed only for the TB patient

When removing an isolation gown, steps the nurse should take would be to

Untie front waist strings, remove gloves, and untie neck ties.

Two major factors that influence whether an infection occurs in an individual are

Inherent health and immunologic status.

Which of the following is a type of transmission-based precaution?

droplet

The nurse should explain to a patient who takes Lasix and has a potassium of 3.2 mEq/L that he should``

The normal potassium level is 3.5-5.0 mEq/L. The patient`s potassium level is low, and he needs to replenish what has been lost as a result of taking the Lasix. In addition to taking potassium supplements, the patient should be given a list of the appropriate foods that have an average of 7 mEq potassium per serving. Eat three servings daily of fruits and meat or fish.`

The premenstrual hemoglobin of a 24-year-old patient with no history of trauma, recent surgery, or hemorrhage is 9.8 g/dL. The nurse interprets that this value is due to

The normal Hgb for a female is > 12-16 g/dL. With the data given, the nurse would suspect anemia Iron-deficiency anemia.

A patient is scheduled for a carotid endarterectomy in 3 days. Which of the following preadmission lab test results must be immediately reported to the physician?

Sodium of 151 mEq/L.The normal electrolyte values for an adult are as follows: sodium of 135-145 mEq/L, chloride of 100-106 mEq/L, potassium of 3.5-5.0 mEq/L, and bicarbonate of 22-29 mEq/L. The serum sodium is the only abnormal value.

At the physician`s office, a patient has a random plasma glucose test. The results were 250 mg/dL. The patient asked the office nurse why the doctor told him to come back the next day to repeat the test. The best Answer is

"This test requires that it be done at least twice for accurate results."The best Answeris to be truthful, but not to frighten the patient by telling him that he may have diabetes (2) (this is the domain of the physician). Levels of > 200 mg/dL on more than one occasion would, however, be diagnostic of diabetes, so the doctor would order at least two tests.

A patient comes to the clinic complaining of a variety of symptoms including pain. The patient has a gastric analysis done and results show that gastric acid is high. This test result would indicate to the nurse that the patient may receive the diagnosis of

High gastric acid levels may indicate a duodenal ulcer.

A 60-year-old patient is admitted to the surgery unit for removal of fibroid tumors. When the nurse checks the lab results for routine blood chemistry, she notes that the sedimentation rate is 29 mm/dL. The appropriate intervention is to

This is a normal sed rate for a female over age 60. Under age 50, normal is 20 mm/hr. If it were increased, it would indicate presence of infection or inflammation, and surgery might have to be postponed

A patient is admitted to the hospital for evaluation. His physician writes in the chart "rule out liver cancer" and schedules a liver biopsy. Before the procedure, the nurse reviews the PT results just returned from the lab: 24 seconds. The nurse also notes that this patient is not on an anticoagulant. The nursing intervention is to

Because the patient is not on anticoagulant therapy, the results are abnormal (normal PT is 11-15 seconds). It is important to notify the head nurse or physician before the biopsy; bleeding could be life threatening. The patient will probably be given vitamin K therapy and when the PT results return to the normal range, the procedure can be done. Liver disease likely caused the prolonged PT.

As part of an annual physical exam, a 60-year-old man has had lab work done. Which of the following serum creatinine levels would indicate that the patient has a mild degree of renal insufficiency?

The normal serum creatinine level for a male is 0.6-0.9 mg/dL. A patient with a mild degree of renal insufficiency would have a slightly elevated level, which in this case would be 1.7. Levels of 3.3 (2) and 4.0 (1) may be associated with acute or chronic renal failure.

A patient with damaged or impaired lungs cannot remove all of the CO2 from the body. When the excess CO2 combines with H2O, it will form

Excess CO2 in the blood, when combined with H2O, forms H2CO3, carbonic acid. Depending on the amount of acid in the blood, the lungs will increase or decrease ventilation to remove excess CO2 (4). The kidneys can excrete or retain H+ (3) and HCO3 (2); thus, the equation representing homeostasis is: CO2 + H2O = H2CO3 = H+ + HCO3.(Lungs) (Kidney)

An 80-year-old patient has been admitted to the hospital with influenza and dehydration. Which of the following blood urea nitrogen (BUN) levels would indicate to the nurse that the patient has received adequate fluid volume replacement?

The normal BUN is 10-20 mg/dL

Which group of cells is the first line of defense against bacterial infection working primarily through phagocytosis?

Neutrophils are the first line of defense against infection. They live in the circulation for about 6 hours after bacteria are ingested. The cells die and become the main component of pus. Monocytes (1) are the second group to defend the body. Platelets (2) are blood components that go to the site of injury and stem blood loss. Basophils (4) release heparin and histamine in areas that are invaded by antigens.

A 53-year-old patient with Crohn`s disease is placed on total parenteral nutrition (TNA). The fluid in the present TNA bottle should be infused by 8 AM. At 7 AM, the nurse observes that it is empty and another TNA bottle has not yet arrived on the unit. The nursing action is to attach the solution nearest a TNA solution which is a bottle of

In order that the patient not experience a sudden drop in blood sugar, the solution nearest most TPN solution concentrations is D10W.

The nurse`s discharge teaching for a patient with acute pancreatitis will include advising him to take a dietary supplement of

Because the patient will be on a low-fat diet to decrease pancreatic activity, he will need supplements of the fat-soluble vitamins

The nurse will know that the patient understands presurgical instructions for hemorrhoid surgery if his diet is

A high-fiber diet produces a soft stool without mechanically irritating the hemorrhoidal area. Foods include bran and complex carbohydrates.

The nurse`s diet instructions for a patient with a colostomy will be

Diets are individualized and patients are generally able to eat the same foods they enjoyed preoperatively. Fresh fruits may cause diarrhea in some, but not all, individuals.

Which of the following statements would be correct when counseling a patient about the postoperative diet he would receive following a simple surgical procedure?

A daily intake of 2800 calories is required for usual/general tissue repair, whereas 6000 calories may be required for extensive tissue repair. Fluid intake is 2000 to 3000 mL/day for uncomplicated surgery. Diet progresses from nothing by mouth the day of surgery to a general diet within a few days.

The nurse will know that the diabetic patient understands his diet when he says that he should obtain the greatest percentage of calories from

The diabetic`s diet should be between 50 and 65 percent carbohydrate calories with only 5 percent of these being simple carbohydrates (sucrose). Fat recommendation is less than 30 percent of calories, and protein should be 0.8 mg/kg/day. complex carbs

A patient with acute pancreatitis required nasogastric intubation due to persistent vomiting and paralytic ileus. Following NG tube removal, the feeding schedule would start with a diet that is

Foods that are high in carbohydrate are given, because those with high protein or fat content stimulate the pancreas. Alcohol is forbidden. There is no need for the patient to be NPO.

A patient with cirrhosis and ascites is placed on a sodium-restricted diet to help control the ascites. In order for this plan to be effective, it is important that the patient also

It is important that fluids be restricted as well, because unrestricted fluid intake leads to a progressive decrease in serum sodium from dilution. Electrolyte imbalance with potential neurologic complications could result.

A patient with a history of pancreatitis should avoid which of the following foods?

Patients with this condition must not consume foods high in fat content because there are inadequate pancreatic enzymes to digest the fat. High fat content also causes pain 2 to 4 hours after ingestion. The suggested diet is high in carbohydrates.

The nurse questions the dietary department about the lunch delivered for a patient with the diagnosis of cirrhosis when she finds on his tray

Ham is high in sodium and can increase fluid retention, leading to edema. Cirrhosis patients are prone to edema as the osmotic pressures change due to a decrease in plasma albumin.

The nurse will know that her teaching has been effective when the patient responds that a low-fiber diet allows the inclusion of

Cooked vegetables and fruits as well as refined breads are included in a low-fiber diet. Bran, fresh fruits, and whole grains and seeds are included in a high-fiber diet.

Patients with hepatitis may have a regular diet ordered, unless they become increasingly symptomatic. The diet will then be modified to decrease the amount of

With liver cell damage, the liver cannot break down and eliminate protein. Protein needs to be decreased until symptoms dissipate.

A pregnant patient comes to the clinic, and the nurse is responsible for nutritional counseling. When the patient says that she has eliminated all salt from her diet, the nurse should respon

Research has indicated that pregnant women require a moderate amount of salt, because it is essential in maintaining increased body fluids needed for adequate placental and renal flow as well as tissue requirements. Highly salted foods should still be avoided

Following surgery, a patient has an IV of D5W to run 50 mL/hr. When the nurse checks his condition for the evening shift, she realizes the IV is 1 hour behind. The first action would be to

Increase the flow so that the loss is made up over the remaining hours in the day. Hmmmmm?

One of the duties of the rehabilitative nurse is to teach the activities of daily living (ADLs) to a patient about to be discharged. One of the most important nursing interventions to accomplish these goals is to

One of the most important principles of teaching is to demonstrate the activity, encourage the patient to perform, and then give positive reinforcement. It is important that the patient learn to do these activities him- or herself

The nurse responsible for administering a thiazide medication to a patient evaluates his recent lab reports, which are K+ 3.0 and NA+ 140. The nurse would

The appropriate intervention is to withhold the thiazide medication (until the nurse receives further orders) and report the K+ level to the physician. Normal K+ is 3.5-5.5 mEq/L. His NA+ level is normal (range 135-145 mEq/L)

A patient scheduled for surgery is given a spinal anesthetic. Immediately following the injection, the nurse will position the patient

Usually, the patient is positioned on the back following the injection. If a high level of anesthesia is desired, the head and shoulders can be lowered to slight Trendelenburg`s. After 20 minutes the anesthetic is set, and the patient can be positioned in any manner.

The nurse is assigned to closely observe a patient for signs of magnesium toxicity following an IV of 4 g magnesium sulfate in 250 mL D5W. The first indication of this condition is

The first sign that the nurse will observe is probably extreme thirst. There will also be a loss of the patellar REFLEX

When a person is experiencing severe stress, the nurse would assess for behaviors such as

Crying and being upset is typical behavior experienced when a person is under stress. Restlessness and anxiety (1) might be present, but they are not typical responses

According to Selye`s stress theory, when the individual is in the alarm phase of the general adaptive syndrome, the body first responds by

Going into shock and countershock.

At which of the following ages would the nurse first expect a child to sit with no support?

nfants begin to sit with support or leaning forward on both hands at 6 months. They sit with minimal or no support between 7 and 8 months. If this milestone does not occur, the infant should be assessed for retardation.

The patient is age 4 and while in the hospital, he becomes very bored. The best activity to implement for this patient is

Fantasy is very active in this stage of development. Puppets would allow for expression of feelings. Also, this activity is more active than TV (1) or books (3) and involves the nurse with the child, which is a positive way of establishing a relationship.

The ultimate outcome, when the grieving process is successfully completed, will be when the bereaved

When the grieving process is completed, the bereaved will no longer feel emotionally dependent on the person who died. They will always feel emotion (1) when thinking of the loved one, but they will be able to realistically recall both the good and bad times. There will always be the need to talk about the loved one (4), even when the grief has been resolved.

In the presence of the RN, a physician asks the LPN to remove the sutures from the incision before the patient is discharged. The initial response to the physician should be

"Please write the order and the sutures will be removed."

The LVN observes the nursing assistant (NA) regulating the IV of an oncology patient receiving morphine sulfate for pain. The LVN is responsible for the patient and has assigned the patient to the NA. The appropriate intervention is to

Ask the NA to meet with the RN and him/her to discuss the responsibility parameters that are appropriate for the NA.

The LVN assigns a patient with uremic frost from renal failure to the UAP. The patient is complaining of dry, itchy skin. To alleviate this problem during bathing, the nurse will instruct the UAP to use

A weak vinegar solution with no soap Vinegar solutions may alleviate itching by dissolving crystal deposits in cutaneous layers and leaving an acid layer on the skin. .

A state's Nurse Practice Act would not include

Difference between RN and LVN functions. Each state has its own Nurse Practice Act for RNs and LVNs. Separately, they are a series of statutes enacted by a state to regulate the practice of nursing in that state. It includes all of these plus education.

The nurse is asked to do a TV commercial for hand lotion. In this commercial, she will wear her nurse`s uniform and advocate the use of this lotion by nurses in their work setting. In doing this, the nurse is violating

The code of ethics is a set of formal guidelines for governing professional action. This situation is not illegalsit is unethical.

The physician wrote a medication order for a patient. The LVN thought the dosage was incorrect. She questioned the physician who said it was all right. Still questioning, she asked the RN, who said it was all right. The LVN gave the medicine, and the patient died from an overdose. Who is liable?

Both the physician and the nurse who gave the medication.

The primary purpose and criteria of licensure is to

The primary purpose of licensing nurses, both RN and LVN, is to safeguard the public by determining that the nurse is a safe and competent practitioner.

A nursing assessment on a male patient indicates that he is showing evidence of increased intracranial pressure. The first nursing action is to

As the PaCO2 increases in the cerebral tissues, blood rushes to the area and this further increases the intracranial pressure. Decreasing the PaCO2, accomplished by breathing deeper and more slowly, will decrease the intracranial blood flow, thus decreasing intracranial pressure.

The nurse is caring for a patient undergoing chemotherapy for cancer. One of the goals of care is to discuss possible side effects of this therapy, one of which is

Alopecia, or hair loss, will probably occur caused by damage to the rapidly dwindling cells of hair follicles. Hair loss begins 2 to 3 weeks after therapy begins and continues through the course of therapy. The other side effects listed do not occur, as nausea, anorexia and diarrhea are common.

A neighbor asks the nurse to look at her 3-year-old child`s rash. Her face, neck and chest are covered with a maculopapular rash. She appears feverish and her nose and eyes are "running." There are small red spots with bluish-white centers on the mucosa of her mouth. The nurse recognizes that these manifestations are most likely caused by

Rubeola is a highly contagious virus. It is more severe than rubella or roseola because of the complications. The virus is transmitted by a cough or sneeze.

A patient is several days postoperative following a right hip replacement with a prosthesis. Assisting the patient to sit in a chair, the nurse will use

The ball in socket position is maintained by minimizing hip flexion (60 degrees or less). This is done with wheelchair and commode extenders, high chairs, and proper bed positions.

A patient has sustained a femoral shaft fracture and is being treated with skeletal traction using balanced suspension with a Thomas splint and Pearson attachment. The goal of maintaining optimum positioning will be accomplished by

It is important that the established angle between the affected thigh and the bed be maintained. The patient can usually have the head of the bed flat or elevated and the lower leg can be exercised, then rest in the Pearson attachment. If the patient migrates toward the head or foot of the bed, then the angle between the thigh and bed would be altered, so it is important that adequate countertraction be maintained.

Which one of the following conditions contraindicates giving morphine?

Morphine causes spasm of the gallbladder and the common bile duct, thus worsening the pain if stones are present. The spasm and resulting inflammation can also block the pancreatic outlet (ampulla of Vater). Meperidine (Demerol) would be a safe choice for pain relief

A patient has been in a motor vehicle accident and has received cervical and spinal stabilization. He is alert and oriented with no evidence of head injury. He develops lower extremity paralysis on the same side as the wound and loses pain and temperature sensation on the side opposite the injury. Based on the preceding information, this type of spinal injury response would be referred to as

Brown-Squard syndrome is caused by hemisection of the spinal cord. Clinical manifestations include paralysis below the level of injury on the same side as the lesion, and the loss of the perception of pain and temperature below the level of injury on the opposite side of the lesion. A total transection would lead to paralysis with no sensation

Apnea of prematurity is defined as a pause

Apnea of prematurity is defined as: 1. cessation of breathing for 20 seconds or longer. 2. cessation of breathing for less than 20 seconds,but associated with cyanosis, bradycardia or limpness

The nurse is interviewing a patient who has been addicted to opiates. The patient tells the nurse that he cannot understand why his drug screening test was positive for opiates, as he is not using the drugs now. Which question would help the nurse understand why the patient`s urine test was positive for opiates?

Ingestion of ibuprofen may cause a false positive on a urine screen for opiates

The father of a 24-hour-old baby boy asks how to hold his son, Christopher, who was circumcised 15 minutes ago. The nurse should respond

"There are a variety of ways to hold a baby. Let me show you a few that I find comforting to newborns."

Which of the following data place a patient at risk for developing pregnancy-induced hypertension (PIH)?

Primigravid women or women pregnant for the first time after an abortion are more susceptible to PIH than are multiparous women.

A 54-year-old female patient is 3 days postoperative following abdominal surgery. The lab results indicate that her white blood cell count is 8,000/cu mm. After analyzing this lab report, the nursing action is to

Do nothing because this is a normal count.This is a normal cell count (the range is 4500 to 11,000/cu mm) so the nurse would do nothing except note that it is normal.

When educating the patient about prevention of coronary heart disease and about risk factors, which of the following would the nurse describe as a predisposing pathologic factor?

CHD is significantly higher in persons with pathologic conditions such as hypertension, hyperlipidemia, and glucose intolerance (such as diabetes mellitus). Gender is an unmodifiable risk factor. Smoking and stress are considered modifiable risk factors.

During visiting hours, a patient the nurse is caring for becomes very agitated and angry with his visitor. The most effective nursing approach to this patient is to

Approach your patient in a warm, supportive manner and assist him to explore his feelings

The nurse is teaching the family of a patient with adrenocortical insufficiency. It is important to assess coping skills and family functioning of patients with this condition because

Clinical findings in Addison`s disease reflect involvement in most body systems including psychologic and behavioral changes.

When planning meals with a patient who is hemodialyzed three times per week, which of the following food selections would indicate the patient understands the dietary guidelines

Apple juice is low in phosphorus, sodium, potassium, and protein. The patient in renal failure must avoid these nutrients, because the damaged kidney cannot clear them. Bananas are high in potassium. Red meat contains protein and phosphorus. Legumes contain phosphorus.

A quadriplegic patient tells the nurse that he believes he is experiencing an episode of autonomic hyperreflexia (dysreflexia). The first nursing intervention is to

Elevate his head as high as possible.Blood pressure can become dangerously elevated during an episode of dysreflexia and can cause cerebral and retinal hemorrhages. Elevating the head will help prevent these complications and should be the nurse`s first action. Identifying the precipitant is useful in terminating the episode by removing the noxious stimulus which provoked the exaggerated autonomic response. A full bladder may precipitate dysreflexia and emptying the bladder would be appropriate if it was the precipitant. The blood pressure and pulse should be monitored throughout the episode of dysreflexia.

Dry heat (AquaK pad) is ordered for a patient with the diagnosis of right leg thrombophlebitis. The orders read: apply to patient`s right calf continuously for 20 hours. The nurse would expect that this treatment would

Because the affected veins are inflamed, the use of heat is indicated to help relieve the inflammation and venospasm

The most important consideration when working with or teaching older adult patients is to evaluate their

The developmental stage is very important when developing plans and when considering the health needs of the individual patient. Occupation, social relationships, and economic status should also be considered in working with the elderly, but these factors are not as important.

A patient is admitted to the labor room with contractions that are about 4 to 5 minutes apart and 30 seconds long. The couple have attended Lamaze childbirth classes. During transition, the nurse should support a breathing pattern that is

Shallow chest breathing, slightly faster than a normal rate, is the pattern used most in transition. Slow breathing, while effective for some women during transition, is usually most effective in earlier labor

When catheterization is performed to obtain a urine specimen for culture from a young child, the procedure would include

Urinary catheters come in a variety of sizes however infants have urethras that are too small even for the smallest urinary catheters. For these infants, 5 Fr and 8 Fr feeding tubes are used to obtain a specimen. All catheters must be sterile and well lubricated.

Infants with sickle cell disease rarely show any symtpoms before 4 months of age because

Fetal hemoglobin resists sickling.

The primary purpose for adult patients having executed a valid living will is that it

Protects health care providers who abide by its provisions against being charged with criminal negligence.

A 29 year old is in the labor room and has been there for 6 hours. It is the change of shift and, when the nurse is receiving report, a nurse`s aide rushes in and says that the patient is delivering. As the nurse enters the room, the baby`s head is crowning. The first nursing action is to

Support the baby`s head with a clean or sterile towel.

When instructing a group of mothers-to-be, which of the following would the nurse note as the virus that most commonly causes acute diarrhea?

Rotavirus is the most common virus causing diarrhea in young children and accounts for 50% of the admissions for dehydration and diarrhea.

A nurse is in a medication-oriented group therapy session with patients who are all on medication therapy. One of the patients says, "I`m sick and tired of taking these drugs." Many group members express similar feelings. In addition to discussing the patients` feelings and the importance of and reasons for medication therapy, the group should have a goal that patients will

Work closely with staff when they discontinue medications.

In a generalized seizure the airway may be compromised during the period of the

During the tonic phase of a seizure, the individual loses consciousness, and continuous muscular contractions occur. If the tonic phase is prolonged, the person may be unable to breathe because the respiratory muscles are contracted. This can result in hypoxia. During the clonic phase, there is a rhythmic jerking of extremities. The postictal phase is the postseizure phase when the person is relaxed.

You might also like